Med Surg II Exam 3 Practice Questions

अब Quizwiz के साथ अपने होमवर्क और परीक्षाओं को एस करें!

Which of the following is often the earliest sign of malignant hyperthermia?

tachycardia

Select the nutrient that is important for postoperative wound healing because it helps form collagen.

vit C

The blood type A positive signifies the presence of the type A __________. a. antigen b. antibody c. platelet d. clotting factor

a. antigen

White blood cells are produced by the __________. a. bone marrow b. lymphatic system c. spleen and thymus d. liver and pancreas

a. bone marrow

A nurse provides care on a bone marrow transplant unit and is preparing a female client for a hematopoietic stem cell transplantation (HSCT) the following day. What information should the nurse emphasize to the client's family and friends?

"Do not visit if you've had a recent infection."

A patient with cancer who developed neutropenia several days ago has consequently been placed in a single-bed room that has positive pressure. His daughter has just come to visit her father after arriving from her home in another state and has asked you for his room number. You notice that the daughter has reddened eyes, sniffles, and a dry cough. What instruction should you provide to the daughter?

"Even though it might be difficult, it's best for your father's health if you get well before visiting him in person, since he's so vulnerable right now."

A nurse is caring for a client who is receiving chemotherapy and has a platelet count of 30,000/mm3. Which statement by the client indicates a need for additional teaching?

"I floss my teeth every morning."

Which statement by a client undergoing external radiation therapy indicates the need for further teaching?

"I'm worried I'll expose my family members to radiation."

A client has been hospitalized for a wedge resection of the left lower lung lobe after a routine chest x-ray shows carcinoma. The client states that he is anxious and asks if he can smoke. Which statement by the nurse would be most therapeutic?

"You are anxious about the surgery. Do you see smoking as helping?"

A young female client has received chemotherapeutic medications and asks about any effects the treatments will have related to her sexual health. The most appropriate statement by the nurse is

"You will need to practice birth control measures."

A patient is learning about his new diagnosis of asthma with the asthma nurse. What medication has the ability to prevent the onset of acute asthma exacerbations? A) Diphenhydramine (Benadryl) B) Montelukast (Singulair) C) Albuterol sulfate (Ventolin) D) Epinephrine

Ans: B

A patient with a family history of allergies has suffered an allergic response based on a genetic predisposition. This atopic response is usually mediated by what immunoglobulin? A) Immunoglobulin A B) Immunoglobulin M C) Immunoglobulin G D) Immunoglobulin E

Ans: D

Which action prevents bacterial contamination during a blood transfusion? 1. Administering the blood within 4 hours of obtaining from blood bank 2. Making certain the blood product and patient's blood type match 3. Washing hands thoroughly before hanging the blood 4. Using medical asepsis when handling the blood

1. Administering the blood within 4 hours of obtaining from blood bank

Which is a type IV hypersensitivity reaction? 1. Allograft rejection 2. Allergic rhinitis 3. Wheal and flare 4. Atopic allergy

1. Allograft rejection

Which is a clinical example of a type II hypersensitivity reaction? 1. Blood transfusion reaction 2. Rash from poison ivy 3. Serum sickness 4. Hay fever

1. Blood transfusion reaction

A patient is being treated for the effects of a longstanding vitamin B12 deficiency. What aspect of the patient's health history would most likely predispose her to this deficiency? A) The patient has irregular menstrual periods. B) The patient is a vegan. C) The patient donated blood 60 days ago. D) The patient frequently smokes marijuana.

Ans: B

A patient is receiving a transfusion of packed red blood cells. Shortly after initiation of the transfusion, the patient begins to exhibit signs and symptoms of a transfusion reaction. The patient is suffering from which type of hypersensitivity? A) Anaphylactic (type 1) B) Cytotoxic (type II) C) Immunecomplex (type III) D) Delayed type (type IV)

Ans: B

A patient on the oncology unit is receiving carmustine, a chemotherapy agent, and the nurse is aware that a significant side effect of this medication is thrombocytopenia. Which symptom should the nurse assess for in patients at risk for thrombocytopenia? A) Interrupted sleep pattern B) Hot flashes C) Epistaxis (nose bleed) D) Increased weight

Ans:C

A nurse is teaching a community class about how to decrease the risk of cancer. Which food should the nurse recommend? a. Low-fat hot dogs b. Smoked ham c. Oranges d. Medium-rare steak

c. oranges

A bone marrow transplant that uses a donor's bone marrow is referred to as __________. a. allogeneic b. autologous c. stem cell d. sibling

c. stem cell

Which manifestations are typical of type I systemic anaphylaxis? 1. Dyspnea, wheezing, laryngeal edema 2. Itching, hypertension, tachycardia 3. Labored breathing, rales, hives 4. Nausea, diaphoresis, chest pain

1. Dyspnea, wheezing, laryngeal edema

Which clinical manifestations are associated with circulatory overload in patients receiving blood? 1. Hypertension, chest tightness, jugular vein distention 2. Fever, abdominal cramping, increased urine output 3. Flushing, tachycardia, hypotension 4. Nausea, vomiting, low back pain

1. Hypertension, chest tightness, jugular vein distention

1The patient at greatest risk for suppressed immunity is the patient who: 1. Is receiving chemotherapy for cancer 2. Is recovering from an appendectomy 3. Has been exposed to chickenpox 4. Has a history of allergies

1. Is receiving chemotherapy for cancer

The nurse is performing the shift assessment of a postsurgical patient. The nurse finds his mental status, level of consciousness, speech, and orientation are intact and at baseline, but the patient tells you he is very anxious. What should the nurse do next? A) Assess the patients oxygen levels. B) Administer antianxiety medications. C) Page the patients the physician. D) Initiate a social work referral.

A) Assess the patients oxygen levels.

The perioperative nurse is providing care for a patient who is recovering on the postsurgical unit following a transurethral prostate resection (TUPR). The patient is reluctant to ambulate, citing the need to recover in bed. For what complication is the patient most at risk? A) Atelectasis B) Anemia C) Dehydration D) Peripheral edema

A) Atelectasis

The intraoperative nurse is transferring a patient from the OR to the PACU after replacement of the right knee. The patient is a 73-year-old woman. The nurse should prioritize which of the following actions? A) Keeping the patient sterile B) Keeping the patient restrained C) Keeping the patient warm D) Keeping the patient hydrated

C) Keeping the patient warm

Which is the appropriate therapy for serum sickness (type III hypersensitivity)? 1. Intravenous gamma globulins 2. Antihistamines and salicylates 3. Oral glucocorticosteroids 4. Epinephrine and steroids

2. Antihistamines and salicylates

Patients with contact dermatitis are taught to: 1. Use antibiotic creams 2. Avoid scratching the lesions 3. Coat their hands with lanolin 4. Soak their hands in mild detergent

2. Avoid scratching the lesions

A patient is receiving the first two ordered units of PRBCs. Shortly after the initiation of the transfusion, the patient complains of chills and experiences a sharp increase in temperature. What is the nurse's priority action? A) Position the patient in high Fowler's. B) Discontinue the transfusion. C) Auscultate the patient's lungs. D) Obtain a blood specimen from the patient.

Ans: B

Persons with B-cell deficiencies experience reduced: 1. Ability to stop the immune response 2. Numbers of circulating antibodies 3. Production of white blood cells 4. Myoglobulin levels

2. Numbers of circulating antibodies

Which nursing actions take priority when a person begins experiencing chills, tachycardia, and back pain during a transfusion? 1. Provide warm blankets and take the patient's temperature. 2. Administer oxygen and raise the head of the bed. 3. Stop the transfusion and notify the physician. 4. Assess vital signs and notify the laboratory.

3. Stop the transfusion and notify the physician.

Clinical manifestations commonly associated with chronic fatigue syndrome (CFS) include: 1. Chronic infections 2. Insomnia and sore throat 3. Rash and hyperexcitability 4. Dysrhythmias and hypertension

2. Insomnia and sore throat

Multiple myeloma is characterized by: 1. Hypokalemia and leukopenia 2. Hypercalcemia and hypouricemia 3. Hypocalcemia and hyperuricemia 4. Thrombocytopenia and hyperkalemia

2. Hypercalcemia and hypouricemia

The nurse is working with the interdisciplinary team to care for a patient who has recently been diagnosed with severe combined immunodeficiency disease (SCID). What treatment is likely of most benefit to this patient? A) Combined radiotherapy and chemotherapy B) Antibiotic therapy C) Hematopoietic stem cell transplantation (HSCT) D) Treatment with colony-stimulating factors (CSFs)

Ans: C

The nurse's review of a patient's most recent blood work reveals a significant increase in the number of band cells. The nurse's subsequent assessment should focus on which of the following? A) Respiratory function B) Evidence of decreased tissue perfusion C) Signs and symptoms of infection D) Recent changes in activity tolerance

Ans: C

Which of the following circumstances would most clearly warrant autologous blood donation? A) The patient has type-O blood. B) The patient has sickle cell disease or a thalassemia. C) The patient has elective surgery pending. D) The patient has hepatitis C.

Ans: C

Which drug is used during initial treatment for systemic anaphylaxis? 1. Dobutamine 250 mg IV 2. Diphenhydramine 50 mg IM 3. 1:1000 epinephrine 0.3 ml SC 4. Methylprednisolone 300 mg IV

3. 1:1000 epinephrine 0.3 ml SC

Hyposensitization therapy is most successful when used for persons who are allergic to: 1. Snake venom and foods 2. Shellfish and bee stings 3. Pollens and house dust 4. Trees and grasses

3. Pollens and house dust

Which laboratory findings will likely be encountered when caring for patients with autoimmune disorders? 1. Reduced red blood cell count 2. Elevated total serum complement levels 3. Presence of antinuclear or autoantibodies 4. Decreased erythrocyte sedimentation rate (ESR)

3. Presence of antinuclear or autoantibodies

Because cachexia is common in patients with metastases what is the nurse sure to assess on this patient? A) Sensory function B) Cranial nerves C) Nutritional status D) Motor function

Ans: C Feedback: Nutritional status is assessed, because cachexia (weak and emaciated condition) is common in patients with metastases.

Which are diagnostic tests for immunodeficiency? 1. Antibody titer, bone scan 2. White blood cell count, hematocrit 3. Erythrocyte sedimentation rate, antibody titer 4. Complete blood count, serologic culture

3. Erythrocyte sedimentation rate, antibody titer

A nurse is providing health education regarding self-care to a patient with an immunodeficiency. What teaching point should the nurse emphasize? A) The importance of aggressive treatment of acne B) The importance of avoiding alcohol-based cleansers C) The need to keep fingernails and toenails closely trimmed D) The need for thorough oral hygiene

Ans: D

Persons visiting an immunodeficient patient are advised to: 1. Leave if the patient begins coughing or sneezing 2. Make sure their immunizations are up to date 3. Wash their hands before entering the room 4. Wear gloves before touching the patient

3. Wash their hands before entering the room

Which nursing action facilitates effective airway clearance during an allergic reaction? 1.Turing the patient to the side 2.Increasing the patient's fluid intake 3.Placing the patient in a high Fowler's position 4.Percussing the patient's back during coughing episodes

3.Placing the patient in a high Fowler's position

Which objective data support the presence of hypersensitivity? 1. Reported exposure to a known allergen 2. Complaints of chest tightness 3. Familial history of allergies 4. Audible wheezing

4. Audible wheezing

Which blood can the person with Rh-negative, type O blood receive safely? 1. A, B, AB, or O 2. A, B, or AB 3. AB or O 4. O

4. O

Which blood product can be safely administered to a patient with a history of febrile nonhemolytic reactions? 1. Whole blood 2. Packed red blood cells 3 .Fresh frozen red blood cells 4. Leukocyte-poor red blood cells

4. Leukocyte-poor red blood cells

Which is the best diet for a person with multiple myeloma? 1. High-phosphate, low-magnesium diet 2. High-potassium, high-fluid diet 3. High-calcium, low-purine diet 4. Low-calcium, low-purine diet

4. Low-calcium, low-purine diet

Which is an appropriate gift for a neutropenic patient? 1. Potted mums 2. Fresh-cut flowers 3. A basket of fresh fruit 4. Flavored bottled water

4. Flavored bottled water

A patient is scheduled for a splenectomy. During discharge education, what teaching point should the nurse prioritize? A) The importance of adhering to prescribed immunosuppressant therapy B) The need to report any signs or symptoms of infection promptly C) The need to ensure adequate folic acid, iron, and vitamin B12 intake D) The importance of limiting activity postoperatively to prevent hemorrhage

Ans: B

A nurse is witnessing a patient sign the consent form for surgery. After the patient signs the consent form, the patient starts asking questions regarding the risks and benefits of a surgical procedure. What action by the nurse is most appropriate? a)Place the consent form in the patient's medical record. b)Notify the nurse manager of the patient's questions. c)Request that the surgeon come and answer the questions. d)Answer the patient's questions.

:c)Request that the surgeon come and answer the questions. Explanation: It is the physician's responsibility to provide information pertaining to risks and benefits of surgery. It is not the responsibility of the nurse or nurse manager to discuss risks and benefits. The consent form should not be placed in the medical record until all questions are answered fully for the patient.

During a mumps outbreak at a local school, a patient, who is a school teacher, is exposed. She has previously been immunized for mumps. What type of immunity does she possess? A) Acquired immunity B) Natural immunity C) Phagocytic immunity D) Humoral immunity

A (Feedback: Acquired immunity usually develops as a result of prior exposure to an antigen, often through immunization. When the body is attacked by bacteria, viruses, or other pathogens, it has three means of defense. The first line of defense, the phagocytic immune response, involves the WBCs that have the ability to ingest foreign particles. A second protective response is the humoral immune response, which begins when the B lymphocytes transform themselves into plasma cells that manufacture antibodies. The natural immune response system is rapid, nonspecific immunity present at birth.)

An infection control nurse is presenting an inservice reviewing the immune response. The nurse describes the clumping effect that occurs when an antibody acts like a cross-link between two antigens. What process is the nurse explaining? A) Agglutination B) Cellular immune response C) Humoral response D) Phagocytic immune response

A (Feedback: Agglutination refers to the clumping effect occurring when an antibody acts as a cross-link between two antigens. This takes place within the context of the humoral immune response, but is not synonymous with it. Cellular immune response, the immune system's third line of defense, involves the attack of pathogens by T-cells. The phagocytic immune response, or immune response, is the system's first line of defense, involving white blood cells that have the ability to ingest foreign particles.)

The nurse is providing care for a patient who has multiple sclerosis. The nurse recognizes the autoimmune etiology of this disease and the potential benefits of what treatment? A) Stem cell transplantation B) Serial immunizations C) Immunosuppression D) Genetic engineering

A (Feedback: Clinical trials using stem cells are under way in patients with a variety of disorders having an autoimmune component, including multiple sclerosis. Immunizations and genetic engineering are not used to treat multiple sclerosis. Immunosuppression would exacerbate symptoms of MS)

A patient with a history of dermatitis takes corticosteroids on a regular basis. The nurse should assess the patient for which of the following complications of therapy? A) Immunosuppression B) Agranulocytosis C) Anemia D) Thrombocytopenia

A (Feedback: Corticosteroids such as prednisone can cause immunosuppression. Corticosteroids do not typically cause agranulocytosis, anemia, or low platelet counts.)

A gardener sustained a deep laceration while working and requires sutures. The patient is asked about the date of her last tetanus shot, which is over 10 years ago. Based on this information, the patient will receive a tetanus immunization. The tetanus injection will allow for the release of what? A) Antibodies B) Antigens C) Cytokines D) Phagocytes

A (Feedback: Immunizations activate the humoral immune response, culminating in antibody production. Antigens are the substances that induce the production of antibodies. Immunizations do not prompt cytokine or phagocyte production)

A patient has undergone treatment for septic shock and received high doses of numerous antibiotics during the course of treatment. When planning the patient's subsequent care, the nurse should be aware of what potential effect on the patient's immune function? A) Bone marrow suppression B) Uncontrolled apoptosis C) Thymus atrophy D) Lymphoma

A (Feedback: Large doses of antibiotics can precipitate bone marrow suppression, affecting immune function. Antibiotics are not noted to cause apoptosis, thymus atrophy, or lymphoma.)

A patient is vigilant in her efforts to "take good care of herself" but is frustrated by her recent history of upper respiratory infections and influenza. What aspect of the patient's lifestyle may have a negative effect on immune response? A) The patient works out at the gym twice daily. B) The patient does not eat red meats. C) The patient takes over-the-counter dietary supplements. D) The patient sleeps approximately 6 hours each night.

A (Feedback: Rigorous exercise or competitive exercise—usually considered a positive lifestyle factor—can be a physiologic stressor and cause negative effects on immune response. The patient's habits around diet and sleep do not present obvious threats to immune function.)

A woman has been diagnosed with breast cancer and is being treated aggressively with a chemotherapeutic regimen. As a result of this regimen, she has an inability to fight infection due to the fact that her bone marrow is unable to produce a sufficient amount of what? A) Lymphocytes B) Cytoblasts C) Antibodies D) Capillaries

A (Feedback: The white blood cells involved in immunity (including lymphocytes) are produced in the bone marrow. Cytoblasts are the protoplasm of the cell outside the nucleus. Antibodies are produced by lymphocytes, but not in the bone marrow. Capillaries are small blood vessels)

A patient's exposure to which of the following microorganisms is most likely to trigger a cellular response? A) Herpes simplex B) Staphylococcus aureus C) Pseudomonas aeruginosa D) Beta hemolytic Streptococcus

A (Feedback: Viral, rather than bacterial antigens, induce a cellular response.)

A surgical patient has just been admitted to the unit from PACU with patient-controlled analgesia (PCA). The nurse should know that the requirements for safe and effective use of PCA include what? A) A clear understanding of the need to self-dose B) An understanding of how to adjust the medication dosage C) A caregiver who can administer the medication as ordered D) An expectation of infrequent need for analgesia

A) A clear understanding of the need to self-dose

An adult patient has just been admitted to the PACU following abdominal surgery. As the patient begins to awaken, he is uncharacteristically restless. The nurse checks his skin and it is cold, moist, and pale. The nurse concerned the patient may be at risk for what? A) Hemorrhage and shock B) Aspiration C) Postoperative infection D) Hypertension and dysrhythmias

A) Hemorrhage and shock

A home health nurse will soon begin administering IVIG to a new patient on a regular basis. What teaching should the nurse provide to the patient? A) The need for a sterile home environment B) Complementary alternatives to IVIG C) Expected benefits and outcomes of the treatment D) Technique for managing and monitoring daily fluid intake

Ans: C

The nurse is caring for an 88-year-old patient who is recovering from an ileac-femoral bypass graft. The patient is day 2 postoperative and has been mentally intact, as per baseline. When the nurse assesses the patient, it is clear that he is confused and has been experiencing disturbed sleep patterns and impaired psychomotor skills. What should the nurse suspect is the problem with the patient? A) Postoperative delirium B) Postoperative dementia C) Senile dementia D) Senile confusion

A) Postoperative delirium

The perioperative nurse is preparing to discharge a female patient home from day surgery performed under general anesthetic. What instruction should the nurse give the patient prior to the patient leaving the hospital? A) The patient should not drive herself home. B) The patient should take an OTC sleeping pill for 2 nights. C) The patient should attempt to eat a large meal at home to aid wound healing. D) The patient should remain in bed for the first 48 hours postoperative.

A) The patient should not drive herself home.

The nurse is discharging a patient home from an outpatient surgery center. The nurse has reviewed all of the discharge instructions with the patient and her caregiver. What else should the nurse do before discharging the patient from the facility? Select all that apply. A) Provide all discharge instructions in writing. B) Provide the nurses or surgeons contact information. C) Give prescriptions to the patient. D) Irrigate the patients incision and perform a sterile dressing change. E) Administer a bolus dose of an opioid analgesic.

A, B, C

The PACU nurse is caring for a 45-year-old male patient who had a left lobectomy. The nurse is assessing the patient frequently for airway patency and cardiovascular status. The nurse should know that the most common cardiovascular complications seen in the PACU include what? Select all that apply. A) Hypotension B) Hypervolemia C) Heart murmurs D) Dysrhythmias E) Hypertension

A, D, E

A patient's blood pressure in the postanesthesia care unit (PACU) has dropped from an admission blood pressure of 140/86 to 102/60 with a pulse change of 70 to 96. SpO2 is 92% on 3 L of oxygen. In which order should the nurse take these actions? a. Increase the IV infusion rate. b. Assess the patient's dressing. c. Increase the oxygen flow rate. d. Check the patient's temperature

ANS: A, C, B, D The first nursing action should be to increase the IV infusion rate. Because the most common cause of hypotension is volume loss, the IV rate should be increased. The next action should be to increase the oxygen flow rate to maximize oxygenation of hypoperfused organs. Because hemorrhage is a common cause of postoperative volume loss, the nurse should check the dressing. Finally, the patient's temperature should be assessed to determine the effects of vasodilation caused by rewarming.

While ambulating in the room, a patient complains of feeling dizzy. In what order will the nurse accomplish the following activities? a. Have the patient sit down in a chair. b. Give the patient something to drink. c. Take the patient's blood pressure (BP). d. Notify the patient's health care provider

ANS: A, C, B, D The first priority for the patient with syncope is to prevent a fall, so the patient should be assisted to a chair. Assessment of the BP will determine whether the dizziness is due to orthostatic hypotension, which occurs because of hypovolemia. Increasing the fluid intake will help prevent orthostatic dizziness. Because this is a common postoperative problem that is usually resolved through nursing measures such as increasing fluid intake and making position changes more slowly, there is no urgent need to notify the health care provider

The nurse is evaluating the client's risk for cancer. What lifestyle change should the nurse recommend?

eats red meat such as steaks or hamburgers every day

When caring for a patient the second postoperative day after abdominal surgery for removal of a large pancreatic cyst, the nurse obtains an oral temperature of 100.8° F. Which action should the nurse take first? a. Have the patient use the incentive spirometer. b. Assess the surgical incision for redness and swelling. c. Administer the ordered PRN acetaminophen (Tylenol). d. Ask the health care provider to prescribe a different antibiotic.

ANS: A A temperature of 100.8° F in the first 48 hours is usually caused by atelectasis, and the nurse should have the patient cough and deep breathe. This problem may be resolved by nursing intervention, and therefore notifying the health care provider is not necessary. Acetaminophen will reduce the temperature, but it will not resolve the underlying respiratory congestion. Because a wound infection does not usually occur before the third postoperative day, a wound infection is not a likely source of the elevated temperature

A patient has developed severe contact dermatitis with burning, itching, cracking, and peeling of the skin on her hands. What should the nurse teach the patient to do? A) Wear powdered latex gloves when in public. B) Wash her hands with antibacterial soap every few hours. C) Maintain room temperature at 75°F to 80°F whenever possible. D) Keep her hands well-moisturized at all times.

Ans: D

The nurse assesses a patient who had a total abdominal hysterectomy 2 days ago. Which information about the patient is most important to communicate to the health care provider? a. The right calf is swollen, warm, and painful. b. The patient's temperature is 100.3° F (37.9° C). c. The 24-hour oral intake is 600 mL greater than the total output. d. The patient complains of abdominal pain at level 6 (0 to 10 scale) when ambulating.

ANS: A The calf pain, swelling, and warmth suggest that the patient has a deep vein thrombosis, which will require the health care provider to order diagnostic tests and/or anticoagulants. Because the stress response causes fluid retention for the first 2 to 5 days postoperatively, the difference between intake and output is expected. A temperature elevation to 100.3° F on the second postoperative day suggests atelectasis, and the nurse should have the patient deep breathe and cough. Pain with ambulation is normal, and the nurse should administer the ordered analgesic before patient activities

A patient lives with a diagnosis of sickle cell anemia and receives frequent blood transfusions. The nurse should recognize the patient's consequent risk of what complication of treatment? A) Hyopvolemia B) Vitamin B12 deficiency C) Thrombocytopenia D) Iron overload

Ans: D

A postoperative patient has not voided for 8 hours after return to the clinical unit. Which action should the nurse take first? a. Perform a bladder scan. b. Encourage increased oral fluid intake. c. Assist the patient to ambulate to the bathroom. d. Insert a straight catheter as indicated on the PRN order.

ANS: A The initial action should be to assess the bladder for distention. If the bladder is distended, providing the patient with privacy (by walking with them to the bathroom) will be helpful. Because of the risk for urinary tract infection, catheterization should only be done after other measures have been tried without success. There is no indication to notify the surgeon about this common postoperative problem unless all measures to empty the bladder are unsuccessful

The nurse working in the postanesthesia care unit (PACU) notes that a patient who has just been transported from the operating room is shivering and has a temperature of 96.5° F (35.8° C). Which action should the nurse take? a. Cover the patient with a warm blanket and put on socks. b. Notify the anesthesia care provider about the temperature. c. Avoid the use of opioid analgesics until the patient is warmer. d. Administer acetaminophen (Tylenol) 650 mg suppository rectally.

ANS: A The patient assessment indicates the need for active rewarming. There is no indication of a need for acetaminophen. Opioid analgesics may help reduce shivering. Because hypothermia is common in the immediate postoperative period, there is no need to notify the anesthesia care provider, unless the patient continues to be hypothermic after active rewarming

After receiving change-of-shift report about these postoperative patients, which patient should the nurse assess first? a. Obese patient who had abdominal surgery 3 days ago and whose wound edges are separating b. Patient who has 30 mL of sanguineous drainage in the wound drain 10 hours after hip replacement surgery c. Patient who has bibasilar crackles and a temperature of 100°F (37.8°C) on the first postoperative day after chest surgery d. Patient who continues to have incisional pain 15 minutes after hydrocodone and acetaminophen (Vicodin) administration

ANS: A The patient's history and assessment suggests possible wound dehiscence, which should be reported immediately to the surgeon. Although the information about the other patients indicates a need for ongoing assessment and/or possible intervention, the data do not suggest any acute complications. Small amounts of red drainage are common in the first postoperative hours. Bibasilar crackles and a slightly elevated temperature are common after surgery, although the nurse will need to have the patient cough and deep breathe. Oral medications typically take more than 15 minutes for effective pain relief

A patient who had knee surgery received intramuscular ketorolac (Toradol) 30 minutes ago and continues to complain of pain at a level of 7 (0 to 10 scale). Which action is best for the nurse to take at this time? a. Administer the prescribed PRN IV morphine sulfate. b. Notify the health care provider about the ongoing knee pain. c. Reassure the patient that postoperative pain is expected after knee surgery. d. Teach the patient that the effects of ketorolac typically last about 6 to 8 hours.

ANS: A The priority at this time is pain relief. Concomitant use of opioids and nonsteroidal antiinflammatory drugs (NSAIDs) improves pain control in postoperative patients. Patient teaching and reassurance are appropriate, but should be done after the patient's pain is relieved. If the patient continues to have pain after the morphine is administered, the health care provider should be notified

A patient who has received a heart transplant is taking cyclosporine, an immunosuppressant. What should the nurse emphasize during health education about infection prevention? A) Eat a high-calorie, high-protein diet. B) Limit physical activity in order to conserve energy. C) Take prophylactic antibiotics as ordered. D) Perform frequent handwashing.

Ans: D

On admission of a patient to the postanesthesia care unit (PACU), the blood pressure (BP) is 122/72. Thirty minutes after admission, the BP falls to 114/62, with a pulse of 74 and warm, dry skin. Which action by the nurse is most appropriate? a. Increase the IV fluid rate. b. Continue to take vital signs every 15 minutes. c. Administer oxygen therapy at 100% per mask. d. Notify the anesthesia care provider (ACP) immediately.

ANS: B A slight drop in postoperative BP with a normal pulse and warm, dry skin indicates normal response to the residual effects of anesthesia and requires only ongoing monitoring. Hypotension with tachycardia and/or cool, clammy skin would suggest hypovolemic or hemorrhagic shock and the need for notification of the ACP, increased fluids, and high-concentration oxygen administration

A nurse assists a patient on the first postoperative day to ambulate, cough, deep breathe, and turn. Which action by the nurse is most helpful? a. Teach the patient to fully exhale into the incentive spirometer. b. Administer ordered analgesic medications before these activities. c. Ask the patient to state two possible complications of immobility. d. Encourage the patient to state the purpose of splinting the incision.

ANS: B An important nursing action to encourage these postoperative activities is administration of adequate analgesia to allow the patient to accomplish the activities with minimal pain. Even with motivation provided by proper teaching, positive reinforcement, and concern about complications, patients will have difficulty if there is a great deal of pain involved with these activities. When using an incentive spirometer, the patient should be taught to inhale deeply, rather than exhale into the spirometer to promote lung expansion and prevent atelectasis

The nurse reviews the laboratory results for a patient on the first postoperative day after a hiatal hernia repair. Which finding would indicate to the nurse that the patient is at increased risk for poor wound healing? a. Potassium 3.5 mEq/L b. Albumin level 2.2 g/dL c. Hemoglobin 11.2 g/dL d. White blood cells 11,900/µL

ANS: B Because proteins are needed for an appropriate inflammatory response and wound healing, the low serum albumin level (normal level 3.5 to 5.0 g/dL) indicates a risk for poor wound healing. The potassium level is normal. Because a small amount of blood loss is expected with surgery, the hemoglobin level is not indicative of an increased risk for wound healing. WBC count is expected to increase after surgery as a part of the normal inflammatory response

A patient who has begun to awaken after 30 minutes in the postanesthesia care unit (PACU) is restless and shouting at the nurse. The patient's oxygen saturation is 96%, and recent laboratory results are all normal. Which action by the nurse is most appropriate? a. Increase the IV fluid rate. b. Assess for bladder distention. c. Notify the anesthesia care provider (ACP). d. Demonstrate the use of the nurse call bell button.

ANS: B Because the patient's assessment indicates physiologic stability, the most likely cause of the patient's agitation is emergence delirium, which will resolve as the patient wakes up more fully. The nurse should look for a cause such as bladder distention. Although hypoxemia is the most common cause, the patient's oxygen saturation is 96%. Emergence delirium is common in patients recovering from anesthesia, so there is no need to notify the ACP. Orientation of the patient to bed controls is needed, but is not likely to be effective until the effects of anesthesia have resolved more completely

A patient who is just waking up after having hip replacement surgery is agitated and confused. Which action should the nurse take first? a. Administer the ordered opioid. b. Check the oxygen (O2) saturation. c. Take the blood pressure and pulse. d. Apply wrist restraints to secure IV lines.

ANS: B Emergence delirium may be caused by a variety of factors. However, the nurse should first assess for hypoxemia. The other actions also may be appropriate, but are not the best initial action

An older patient is being discharged from the ambulatory surgical unit following left eye surgery. The patient tells the nurse, "I do not know if I can take care of myself with this patch over my eye." Which action by the nurse is most appropriate? a. Refer the patient for home health care services. b. Discuss the specific concerns regarding self-care. c. Give the patient written instructions regarding care. d. Assess the patient's support system for care at home.

ANS: B The nurse's initial action should be to assess exactly the patient's concerns about self-care. Referral to home health care and assessment of the patient's support system may be appropriate actions but will be based on further assessment of the patient's concerns. Written instructions should be given to the patient, but these are unlikely to address the patient's stated concern about self-care

An older patient who had knee replacement surgery 2 days ago can only tolerate being out of bed with physical therapy twice a day. Which collaborative problem should the nurse identify as a priority for this patient? a. Potential complication: hypovolemic shock b. Potential complication: venous thromboembolism c. Potential complication: fluid and electrolyte imbalance d. Potential complication: impaired surgical wound healing

ANS: B The patient is older and relatively immobile, which are two risk factors for development of deep vein thrombosis. The other potential complications are possible postoperative problems, but they are not supported by the data about this patient

In the postanesthesia care unit (PACU), a patient's vital signs are blood pressure 116/72, pulse 74, respirations 12, and SpO2 91%. The patient is sleepy but awakens easily. Which action should the nurse take first? a. Place the patient in a side-lying position. b. Encourage the patient to take deep breaths. c. Prepare to transfer the patient to a clinical unit. d. Increase the rate of the postoperative IV fluids.

ANS: B The patient's borderline SpO2 and sleepiness indicate hypoventilation. The nurse should stimulate the patient and remind the patient to take deep breaths. Placing the patient in a lateral position is needed when the patient first arrives in the PACU and is unconscious. The stable blood pressure and pulse indicate that no changes in fluid intake are required. The patient is not fully awake and has a low SpO2, indicating that transfer from the PACU to a clinical unit is not appropriate

Which action could the postanesthesia care unit (PACU) nurse delegate to unlicensed assistive personnel (UAP) who help with the transfer of a patient to the clinical unit? a. Clarify the postoperative orders with the surgeon. b. Help with the transfer of the patient onto a stretcher. c. Document the appearance of the patient's incision in the chart. d. Provide hand off communication to the surgical unit charge nurse.

ANS: B The scope of practice of UAP includes repositioning and moving patients under the supervision of a nurse. Providing report to another nurse, assessing and documenting the wound appearance, and clarifying physician orders with another nurse require registered-nurse (RN) level education and scope of practice

Patient teaching regarding infection prevention for the patient with an immunodeficiency includes which of the following guidelines? A) Cook all food thoroughly. B) Refrain from using creams or emollients on skin. C) Maintain contact only with individuals who have recently been vaccinated. D) Take OTC vitamin supplements consistently.

Ans: A

A patient's T-tube is draining dark green fluid after gallbladder surgery. What action by the nurse is the most appropriate? a. Notify the patient's surgeon. b. Place the patient on bed rest. c. Document the color and amount of drainage. d. Irrigate the T-tube with sterile normal saline.

ANS: C A T-tube normally drains dark green to bright yellow drainage, so no action other than to document the amount and color of the drainage is needed. The other actions are not necessary

The nasogastric (NG) tube is removed on the second postoperative day, and the patient is placed on a clear liquid diet. Four hours later, the patient complains of sharp, cramping gas pains. What action by the nurse is the most appropriate? a. Reinsert the NG tube. b. Give the PRN IV opioid. c. Assist the patient to ambulate. d. Place the patient on NPO status.

ANS: C Ambulation encourages peristalsis and the passing of flatus, which will relieve the patient's discomfort. If distention persists, the patient may need to be placed on NPO status, but usually this is not necessary. Morphine administration will further decrease intestinal motility. Gas pains are usually caused by trapping of flatus in the colon, and reinsertion of the NG tube will not relieve the pains

The nurse is caring for a patient who has a diagnosis of paroxysmal nocturnal hemoglobinuria. When planning this patient's care, the nurse should recognize the patient's heightened risk of what complication? A) Venous thromboembolism B) Acute respiratory distress syndrome (ARDS) C) Myocardial infarction D) Hypertensive urgency

Ans: A

A patient is transferred from the postanesthesia care unit (PACU) to the clinical unit. Which action by the nurse on the clinical unit should be performed first? a. Assess the patient's pain. b. Orient the patient to the unit. c. Take the patient's vital signs. d. Read the postoperative orders.

ANS: C Because the priority concerns after surgery are airway, breathing, and circulation, the vital signs are assessed first. The other actions should take place after the vital signs are obtained and compared with the vital signs before transfer

The nurse is caring for a patient the first postoperative day following a laparotomy for a small bowel obstruction. The nurse notices new bright-red drainage about 5 cm in diameter on the dressing. Which action should the nurse take first? a. Reinforce the dressing. b. Apply an abdominal binder. c. Take the patient's vital signs. d. Recheck the dressing in 1 hour for increased drainage.

ANS: C New bright-red drainage may indicate hemorrhage, and the nurse should initially assess the patient's vital signs for tachycardia and hypotension. The surgeon should then be notified of the drainage and the vital signs. The dressing may be changed or reinforced, based on the surgeon's orders or institutional policy. The nurse should not wait an hour to recheck the dressing

A postoperative patient has a nursing diagnosis of ineffective airway clearance. The nurse determines that interventions for this nursing diagnosis have been successful if which is observed? a. Patient drinks 2 to 3 L of fluid in 24 hours. b. Patient uses the spirometer 10 times every hour. c. Patient's breath sounds are clear to auscultation. d. Patient's temperature is less than 100.4° F orally.

ANS: C One characteristic of ineffective airway clearance is the presence of adventitious breath sounds such as rhonchi or crackles, so clear breath sounds are an indication of resolution of the problem. Spirometer use and increased fluid intake are interventions for ineffective airway clearance but may not improve breath sounds in all patients. Elevated temperature may occur with atelectasis, but a normal or near-normal temperature does not always indicate resolution of respiratory problems

An experienced nurse orients a new nurse to the postanesthesia care unit (PACU). Which action by the new nurse, if observed by the experienced nurse, indicates that the orientation was successful? a. The new nurse assists a nauseated patient to a supine position. b. The new nurse positions an unconscious patient supine with the head elevated. c. The new nurse turns an unconscious patient to the side upon arrival in the PACU. d. The new nurse places a patient in the Trendelenburg position when the blood pressure drops.

ANS: C The patient should initially be positioned in the lateral "recovery" position to keep the airway open and avoid aspiration. The Trendelenburg position is avoided because it increases the work of breathing. The patient is placed supine with the head elevated after regaining consciousness

A nurse is caring for a patient with a phagocytic cell disorder. The patient states, "My specialist says that I will likely be cured after I get my treatment tomorrow." To what treatment is the patient most likely referring? A) Treatment with granulocyte-macrophage colony-stimulating factor (GM-CSF) B) Hematopoietic stem cell transplantation C) Treatment with granulocyte colony-stimulating factor (G-CSF) D) Brachytherapy

Ans: B

A nurse is caring for a teenage girl who has had an anaphylactic reaction after a bee sting. The nurse is providing patient teaching prior to the patient's discharge. In the event of an anaphylactic reaction, the nurse informs the patient that she should self-administer epinephrine in what site? A) Forearm B) Thigh C) Deltoid muscle D) Abdomen

Ans: B

A nurse is preparing to discharge a patient with an immunodeficiency. When preparing the patient for self-infusion of IVIG in the home setting, what education should the nurse prioritize? A) Sterile technique for establishing a new IV site B) Signs and symptoms of adverse reactions C) Formulas for calculating daily doses D) Technique for adding medications to the IVIG

Ans: B

A nurse knows of several patients who have achieved adequate control of their allergy symptoms using over-the-counter antihistamines. Antihistamines would be contraindicated in the care of which patient? A) A patient who has previously been treated for tuberculosis B) A pregnant woman at 30 weeks' gestation C) A patient who is on estrogen-replacement therapy D) A patient with a severe allergy to eggs

Ans: B

The nurse assesses a patient on the second postoperative day after abdominal surgery to repair a perforated duodenal ulcer. Which finding is most important for the nurse to report to the surgeon? a. Tympanic temperature 99.2° F (37.3° C) b. Fine crackles audible at both lung bases c. Redness and swelling along the suture line d. 200 mL sanguineous fluid in the wound drain

ANS: D Wound drainage should decrease and change in color from sanguineous to serosanguineous by the second postoperative day. The color and amount of drainage for this patient are abnormal and should be reported. Redness and swelling along the suture line and a slightly elevated temperature are normal signs of postoperative inflammation. Atelectasis is common after surgery. The nurse should have the patient cough and deep breathe, but there is no urgent need to notify the surgeon

A patient with a diagnosis of renal cell carcinoma is being treated with chemotherapy. During a previous round of chemotherapy, the patient's tumor responded well to treatment but the chemotherapy caused intense nausea and vomiting. How should the patient's potential nausea and vomiting be addressed during this current round of treatment?

Administer antiemetics in anticipation of the patient's nausea.

A patient diagnosed with common variable immune deficiency (CVID) has been admitted to the acute medicine unit. When reviewing this patient's laboratory findings, the nurse should prioritize what values? A) Creatinine and blood urea nitrogen (BUN) B) Hemoglobin and vitamin B12 C) Sodium, potassium and magnesium D) D-dimer and c-reactive protein

Ans: B

A 59-year-old male patient is scheduled for a hemorrhoidectomy. The OR nurse should anticipate assisting the other team members with positioning the patient in what manner? A) Dorsal recumbent position B) Trendelenburg position C) Sims position D) Lithotomy position

Ans: Lithotomy position D Feedback: The lithotomy position is used for nearly all perineal, rectal, and vaginal surgical procedures. The Sims or lateral position is used for renal surgery and the Trendelenburg position usually is used for surgery on the lower abdomen and pelvis. The usual position for surgery, called the dorsal recumbent position, is flat on the back, but this would be impracticable for rectal surgery.

A 5-year-old boy has been diagnosed with a severe food allergy. What is an important parameter to address when educating the parents of this child about his allergy and care? A) Wear a medical identification bracelet. B) Know how to use the antihistamine pen. C) Know how to give injections of lidocaine. D) Avoid live attenuated vaccinations.

Ans: A

A patient is being treated in the ICU after a medical error resulted in an acute hemolytic transfusion reaction. What was the etiology of this patient's adverse reaction? A) Antibodies to donor leukocytes remained in the blood. B) The donor blood was incompatible with that of the patient. C) The patient had a sensitivity reaction to a plasma protein in the blood. D) The blood was infused too quickly and overwhelmed the patient's circulatory system.

Ans: B

A home health nurse is caring for a patient who has an immunodeficiency. What is the nurse's priority action to help ensure successful outcomes and a favorable prognosis? A) Encourage the patient and family to be active partners in the management of the immunodeficiency. B) Encourage the patient and family to manage the patient's activity level and activities of daily living effectively. C) Make sure that the patient and family understand the importance of monitoring fluid balance. D) Make sure that the patient and family know how to adjust dosages of the medications used in treatment.

Ans: A

A man suffers a leg wound which causes minor blood loss. As a result of bleeding, the process of primary hemostasis is activated. What occurs in primary hemostasis? A) Severed blood vessels constrict. B) Thromboplastin is released. C) Prothrombin is converted to thrombin. D) Fibrin is lysed.

Ans: A

A nurse at a blood donation clinic has completed the collection of blood from a woman. The woman states that she feels lightheaded and she appears visibly pale. What is the nurse's most appropriate action? A) Help her into a sitting position with her head lowered below her knees. B) Administer supplementary oxygen by nasal prongs. C) Obtain a full set of vital signs. D) Inform a physician or other primary care provider.

Ans: A

A nurse has asked the nurse educator if there is any way to predict the severity of a patient's anaphylactic reaction. What would be the nurse's best response? A) "The faster the onset of symptoms, the more severe the reaction." B) "The reaction will be about one-third more severe than the patient's last reaction to the same antigen." C) "There is no way to gauge the severity of a patient's anaphylaxis, even if it has occurred repeatedly in the past." D) "The reaction will generally be slightly less severe than the last reaction to the same antigen."

Ans: A

A nurse is admitting a patient with an immunodeficiency to the medical unit. In planning the care of this patient, the nurse should assess for what common sign of immunodeficiency? A) Chronic diarrhea B) Hyperglycemia C) Rhinorrhea D) Contact dermatitis

Ans: A

A patient has been admitted to the emergency department with signs of anaphylaxis following a bee sting. The nurse knows that if this is a true allergic reaction the patient will present with what alteration in laboratory values? A) Increased eosinophils B) Increased neutrophils C) Increased serum albumin D) Decreased blood glucose

Ans: A

A patient has been diagnosed with a lymphoid stem cell defect. This patient has the potential for a problem involving which of the following? A) Plasma cells B) Neutrophils C) Red blood cells D) Platelets

Ans: A

A patient has sought care, stating that she developed hives overnight. The nurse's inspection confirms the presence of urticaria. What type of allergic hypersensitivity reaction has the patient developed? A) Type I B) Type II C) Type III D) Type IV

Ans: A

A patient is admitted for the treatment of a primary immunodeficiency and intravenous immunoglobulin (IVIG) is ordered. What should the nurse monitor for as a potential adverse effect of IVIG administration? A) Anaphylaxis B) Hypertension C) Hypothermia D) Joint pain

Ans: A

A patient is receiving a blood transfusion and complains of a new onset of slight dyspnea. The nurse's rapid assessment reveals bilateral lung crackles and elevated BP. What is the nurse's most appropriate action? A) Slow the infusion rate and monitor the patient closely. B) Discontinue the transfusion and begin resuscitation. C) Pause the transfusion and administer a 250 mL bolus of normal saline. D) Discontinue the transfusion and administer a beta-blocker, as ordered.

Ans: A

A patient with multiple food and environmental allergies tells the nurse that he is frustrated and angry about having to be so watchful all the time and wonders if it is really worth it. What would be the nurse's best response? A) "I can only imagine how you feel. Would you like to talk about it?" B) "Let's find a quiet spot and I'll teach you a few coping strategies." C) "That's the same way that most patients who have a chronic illness feel." D) "Do you think that maybe you could be managing things more efficiently?"

Ans: A

A patient's most recent blood work reveals low levels of albumin. This assessment finding should suggest the possibility of what nursing diagnosis? A) Risk for imbalanced fluid volume related to low albumin B) Risk for infection related to low albumin C) Ineffective tissue perfusion related to low albumin D) Impaired skin integrity related to low albumin

Ans: A

A school nurse is caring for a child who appears to be having an allergic response. What should be the initial action of the school nurse? A) Assess for signs and symptoms of anaphylaxis. B) Assess for erythema and urticaria. C) Administer an OTC antihistamine. D) Administer epinephrine.

Ans: A

After the completion of testing, a child's allergies have been attributed to her family's cat. When introducing the family to the principles of avoidance therapy, the nurse should promote what action? A) Removing the cat from the family's home B) Administering OTC antihistamines to the child regularly C) Keeping the cat restricted from the child's bedroom D) Maximizing airflow in the house

Ans: A

An office worker takes a cupcake that contains peanut butter. He begins wheezing, with an inspiratory stridor and air hunger and the occupational health nurse is called to the office. The nurse should recognize that the worker is likely suffering from which type of hypersensitivity? A) Anaphylactic (type 1) B) Cytotoxic (type II) C) Immune complex (type III) D) Delayed-type (type IV)

Ans: A

An older adult client is exhibiting many of the characteristic signs and symptoms of iron deficiency. In addition to a complete blood count, what diagnostic assessment should the nurse anticipate? A) Stool for occult blood B) Bone marrow biopsy C) Lumbar puncture D) Urinalysis

Ans: A

Family members of an immunocompromised patient have asked the nurse why antibiotics are not being given to the patient in order to prevent infection. How should the nurse best respond? A) "Using antibiotics to prevent infections can cause the growth of drug-resistant bacteria." B) "If an antibiotic is given to prevent a bacterial infection, the patient is at risk of a viral infection." C) "Antibiotics can never prevent an infection; they can only cure an infection that is fully developed." D) "Antibiotics cannot resolve infections in people who are immunocompromised."

Ans: A

The nurse is caring for a patient with an immunodeficiency who has experienced sudden malaise. The nurse's colleague states, "I'm pretty sure that it's not an infection, because the most recent blood work looks fine." What principle should guide the nurse's response to the colleague? A) Immunodeficient patients will usually exhibit subtle and atypical signs of infection. B) Infections in immunodeficient patients have a slower onset but a more severe course. C) Laboratory blood work is often inaccurate in immunodeficient patients. D) Immunodeficient patients do not develop symptoms of infection.

Ans: A

The nurse is planning the care of a patient who has a diagnosis of atopic dermatitis, which commonly affects both of her hands and forearms. What risk nursing diagnosis should the nurse include in the patient's care plan? A) Risk for Disturbed Body Image Related to Skin Lesions B) Risk for Disuse Syndrome Related to Dermatitis C) Risk for Ineffective Role Performance Related to Dermatitis D) Risk for Self-Care Deficit Related to Skin Lesions

Ans: A

The nurse is preparing to administer a unit of platelets to an adult patient. When administering this blood product, which of the following actions should the nurse perform? A) Administer the platelets as rapidly as the patient can tolerate. B) Establish IV access as soon as the platelets arrive from the blood bank. C) Ensure that the patient has a patent central venous catheter. D) Aspirate 10 to 15 mL of blood from the patient's IV immediately following the transfusion.

Ans: A

The nurse is providing care for a patient who has experienced a type I hypersensitivity reaction. What condition is an example of such a reaction? A) Anaphylactic reaction after a bee sting B) Skin reaction resulting from adhesive tape C) Myasthenia gravis D) Rheumatoid arthritis

Ans: A

The nurse's brief review of a patient's electronic health record indicates that the patient regularly undergoes therapeutic phlebotomy. Which of the following rationales for this procedure is most plausible? A) The patient may chronically produce excess red blood cells. B) The patient may frequently experience a low relative plasma volume. C) The patient may have impaired stem cell function. D) The patient may previously have undergone bone marrow biopsy.

Ans: A

A nurse on the oncology unit is caring for a patient with an astrocytoma. The patient has just been told that the tumor is growing very fast. The patient asks the nurse how these tumors grow. What would be the nurse's best response? A) "Astrocytomas infiltrate the surrounding neural connective tissue." B) "Astrocytomas grow by invading the surrounding gray matter." C) "Astrocytomas grow by invading the surrounding white matter." D) "Astrocytomas spread down the spinal cord." al structures. Astrocytomas invade both the gray and white matter indiscriminately, and will grow down the spinal cord if they spread far enough. These answers are not the most correct.

Ans: A Feedback: Usually, astrocytomas spread by infiltrating into the surrounding neural connective tissue, and therefore cannot be totally removed without causing considerable damage to vital structures. Astrocytomas invade both the gray and white matter indiscriminately, and will grow down the spinal cord if they spread far enough. These answers are not the most correct.

You are providing preoperative teaching to a patient scheduled for hip replacement surgery in 1 month. During the preoperative teaching, the patient gives you a list of medications she takes, the dosage, and frequency. Which of the following interventions provides the patient with the most accurate information? A) Instruct the patient to stop taking St. John's wort at least 2 weeks prior to surgery due to its interaction with anesthetic agents. B) Instruct the patient to continue taking ephedrine prior to surgery due to its beneficial effect on blood pressure. C) Instruct the patient to discontinue Synthroid due to its effect on blood coagulation and the potential for heart dysrhythmias. D) Instruct the patient to continue any herbal supplements unless otherwise instructed, and inform the patient that these supplements have minimal effect on the surgical procedure.

Ans: A Feedback: Because of the potential effects of herbal medications on coagulation and potential lethal interactions with other medications, the nurse must ask surgical patients specifically about the use of these agents, document their use, and inform the surgical team and anesthesiologist, anesthetist, or nurse anesthetist. Currently, it is recommended that the use of herbal products be discontinued at least 2 weeks before surgery. Patients with uncontrolled thyroid disorders are at risk for thyrotoxicosis and respiratory failure. The administration of Synthroid is imperative in the preoperative period. The use of ephedrine in the preoperative phase can cause hypertension and should be avoided.

An OR nurse will be participating in the intraoperative phase of a patient's kidney transplant. What action will the nurse prioritize in this aspect of nursing care? A) Monitoring the patient's physiologic status B) Providing emotional support to family C) Maintaining the patient's cognitive status D) Maintaining a clean environment

Ans: A Feedback: During the intraoperative phase, the nurse is responsible for physiologic monitoring. The intraoperative nurse cannot support the family at this time and the nurse is not responsible for maintaining the patient's cognitive status. The intraoperative nurse maintains an aseptic, not clean, environment.

A clinic nurse is conducting a preoperative interview with an adult patient who will soon be scheduled to undergo cardiac surgery. What interview question most directly addresses the patient's safety? A) What prescription and nonprescription medications do you currently take? B) Have you previously been admitted to the hospital, either for surgery or for medical treatment? C) How long do you expect to be at home recovering after your surgery? D) Would you say that you tend to eat a fairly healthy diet?

Ans: A Feedback: It is imperative to know a preoperative patient's current medication regimen, including OTC medications and supplements. None of the other listed questions directly addresses an issue with major safety implications.

The nurse is performing a preadmission assessment of a patient scheduled for a bilateral mastectomy. Of what purpose of the preadmission assessment should the nurse be aware? A) Verifies completion of preoperative diagnostic testing B) Discusses and reviews patient's health insurance coverage C) Determines the patient's suitability as a surgical candidate D) Informs the patient of need for postoperative transportation

Ans: A Feedback: Purposes of preadmission testing (PAT) include verifying completion of preoperative diagnostic testing. The nurse's role in PAT does not normally involve financial considerations or addressing transportation. The physician determines the patient's suitability for surgery.

One of the things a nurse has taught to a patient during preoperative teaching is to have nothing by mouth for the specified time before surgery. The patient asks the nurse why this is important. What is the most appropriate response for the patient? A) You will need to have food and fluid restricted before surgery so you are not at risk for choking. B) The restriction of food or fluid will prevent the development of pneumonia related to decreased lung capacity. C) The presence of food in the stomach interferes with the absorption of anesthetic agents. D) By withholding food for 8 hours before surgery, you will not develop constipation in the postoperative period.

Ans: A Feedback: The major purpose of withholding food and fluid before surgery is to prevent aspiration. There is no scientific basis for withholding food and the development of pneumonia or interference with absorption of anesthetic agents. Constipation in patients in the postoperative period is related to the anesthesia, not from withholding food or fluid in the hours before surgery.

The admitting nurse in a short-stay surgical unit is responsible for numerous aspects of care. What must the nurse verify before the patient is taken to the preoperative holding area? A) That preoperative teaching was performed B) That the family is aware of the length of the surgery C) That follow-up home care is not necessary D) That the family understands the patient will be discharged immediately after surgery.

Ans: A Feedback: The nurse needs to be sure that the patient and family understand that the patient will first go to the preoperative holding area before going to the OR for the surgical procedure and then will spend some time in the PACU before being discharged home with the family later that day. Other preoperative teaching content should also be verified and reinforced, as needed. The nurse should ensure that any plans for follow-up home care are in place.

A nurse is assessing a patient with an acoustic neuroma who has been recently admitted to an oncology unit. What symptoms is the nurse likely to find during the assessment? A) Loss of hearing, tinnitus, and vertigo B) Loss of vision, change in mental status, and hyperthermia C) Loss of hearing, increased sodium retention, and hypertension D) Loss of vision, headache, and tachycardia

Ans: A Feedback: An acoustic neuroma is a tumor of the eighth cranial nerve, the cranial nerve most responsible for hearing and balance. The patient with an acoustic neuroma usually experiences loss of hearing, tinnitus, and episodes of vertigo and staggering gait. Acoustic neuromas do not cause loss of vision, increased sodium retention, or tachycardia.

A 16-year-old male patient presents at the free clinic with complaints of impotency. Upon physical examination the physician finds hypogonadism. What would the nurse know is the suspected diagnosis? A) Prolactinoma B) Angioma C) Glioma D) Adrenocorticotropic hormone (ACTH)-producing adenoma

Ans: A Feedback: Male patients with prolactinomas may present with impotence and hypogonadism. An adrenocorticotropic hormone (ACTH)-producing adenoma would cause acromegaly. There is not enough information in the scenario to know if the tumor is an angioma, glioma, or neuroma.

A patient has been admitted to the Neuro ICU with a diagnosis of a brain tumor. The patient is scheduled to have a tumor resection/removal in the morning. What is an important part of the initial assessment on this patient? A) The gag reflex B) Ability to chew C) Sensory perception D) Corneal reflex

Ans: A Feedback: Preoperatively, the gag reflex and ability to swallow are evaluated. In patients with diminished gag response, care includes teaching the patient to direct food and fluids toward the unaffected side, having the patient sit upright to eat, offering a semisoft diet, and having suction readily available. The ability to chew and the corneal reflex would be assessed and so would sensory perception on the face, but none of them are more important than the gag reflex.

An OR nurse is participating in an interdisciplinary audit of infection control practices in the surgical department. The nurse should know that a basic guideline for maintaining surgical asepsis is what? A) Sterile surfaces or articles may touch other sterile surfaces. B) Sterile supplies can be used on another patient if the packages are intact. C) The outer lip of a sterile solution is considered sterile. D) The scrub nurse may pour a sterile solution from a nonsterile bottle.

Ans: A Feedback: Basic guidelines for maintaining sterile technique include that sterile surfaces or articles may touch other sterile surfaces only. The other options each constitute a break in sterile technique.

A circulating nurse provides care in a surgical department that has multiple surgeries scheduled for the day. The nurse should know to monitor which patient most closely during the intraoperative period because of the increased risk for hypothermia? A) A 74-year-old woman with a low body mass index B) A 17-year-old boy with traumatic injuries C) A 45-year-old woman having an abdominal hysterectomy D) A 13-year-old girl undergoing craniofacial surgery

Ans: A Feedback: Elderly patients are at greatest risk during surgical procedures because they have an impaired ability to increase their metabolic rate and impaired thermoregulatory mechanisms, which increase susceptibility to hypothermia. The other patients are likely at a lower risk.

The nurse is preparing an elderly patient for a scheduled removal of orthopedic hardware, a procedure to be performed under general anesthetic. For which adverse effect should the nurse most closely monitor the patient? A) Hypothermia B) Pulmonary edema C) Cerebral ischemia D) Arthritis

Ans: A Feedback: Inadvertent hypothermia may occur as a result of a low temperature in the OR, infusion of cold fluids, inhalation of cold gases, open body wounds or cavities, decreased muscle activity, advanced age, or the pharmaceutical agents used (e.g., vasodilators, phenothiazines, general anesthetics). The anesthetist monitors for pulmonary edema and cerebral ischemia. Arthritis is not an adverse effect of surgical anesthesia.

The OR nurse acts in the circulating role during a patients scheduled cesarean section. For what task is this nurse solely responsible? A) Performing documentation B) Estimating the patients blood loss C) Setting up the sterile tables D) Keeping track of drains and sponges

Ans: A Feedback: Main responsibilities of the circulating nurse include verifying consent; coordinating the team; and ensuring cleanliness, proper temperature and humidity, lighting, safe function of equipment, and the availability of supplies and materials. The circulating nurse monitors aseptic practices to avoid breaks in technique while coordinating the movement of related personnel as well as implementing fire safety precautions. The circulating nurse also monitors the patient and documents specific activities throughout the operation to ensure the patients safety and well-being. Estimating the patients blood loss is the surgeons responsibility; setting up the sterile tables is the responsibility of the first scrub; and keeping track of the drains and sponges is the joint responsibility of the circulating nurse and the scrub nurse.

A patient who underwent a bowel resection to correct diverticula suffered irreparable nerve damage. During the case review, the team is determining if incorrect positioning may have contributed to the patients nerve damage. What surgical position places the patient at highest risk for nerve damage? A) Trendelenburg B) Prone C) Dorsal recumbent D) Lithotomy

Ans: A Feedback: Shoulder braces must be well padded to prevent irreparable nerve injury, especially when the Trendelenburg position is necessary. The other listed positions are less likely to cause nerve injury.

The perioperative nurse has completed the presurgical assessment of an 82-year-old female patient who is scheduled for a left total knee replacement. When planning this patients care, the nurse should address the consequences of the patients aging cardiovascular system. These include an increased risk of which of the following? A) Hypervolemia B) Hyponatremia C) Hyperkalemia D) Hyperphosphatemia

Ans: A Feedback: The aging heart and blood vessels have decreased ability to respond to stress. Reduced cardiac output and limited cardiac reserve make the elderly patient vulnerable to changes in circulating volume and blood oxygen levels. There is not an increased risk for hypopnea, hyperkalemia, or hyperphosphatemia because of an aging cardiovascular system.

A patient waiting in the presurgical holding area asks the nurse, Why exactly do they have to put a breathing tube into me? My surgery is on my knee. What is the best rationale for intubation during a surgical procedure that the nurse should describe? A) The tube provides an airway for ventilation. B) The tube protects the patients esophagus from trauma. C) The patient may receive an antiemetic through the tube. D) The patients vital signs can be monitored with the tube.

Ans: A Feedback: The anesthetic is administered and the patients airway is maintained through an intranasal intubation, oral intubation, or a laryngeal mask airway. The tube also helps protect aspiration of stomach contents. The tube does not protect the esophagus. Because the tube goes into the lungs, no medications are given through the tube. The patients vital signs are not monitored through the tube.

The circulating nurse in an outpatient surgery center is assessing a patient who is scheduled to receive moderate sedation. What principle should guide the care of a patient receiving this form of anesthesia? A) The patient must never be left unattended by the nurse. B) The patient should begin a course of antiemetics the day before surgery. C) The patient should be informed that he or she will remember most of the procedure. D) The patient must be able to maintain his or her own airway.

Ans: A Feedback: The patient receiving moderate sedation should never be left unattended. The patients ability to maintain his or her airway depends on the level of sedation. The administration of moderate sedation is not a counter indication for giving an antiemetic. The patient receiving moderate sedation does not remember most of the procedure.

The OR nurse is participating in the appendectomy of a 20 year-old female patient who has a dangerously low body mass index. The nurse recognizes the patients consequent risk for hypothermia. What action should the nurse implement to prevent the development of hypothermia? A) Ensure that IV fluids are warmed to the patients body temperature. B) Transfuse packed red blood cells to increase oxygen carrying capacity. C) Place warmed bags of normal saline at strategic points around the patients body. D) Monitor the patients blood pressure and heart rate vigilantly.

Ans: A Feedback: Warmed IV fluids can prevent the development of hypothermia. Applying warmed bags of saline around the patient is not common practice. The patient is not transfused to prevent hypothermia. Blood pressure and heart rate monitoring are important, but do not relate directly to the risk for hypothermia.

A patient has been brought to the emergency department by EMS after being found unresponsive. Rapid assessment reveals anaphylaxis as a potential cause of the patient's condition. The care team should attempt to assess for what potential causes of anaphylaxis? Select all that apply. A) Foods B) Medications C) Insect stings D) Autoimmunity E) Environmental pollutants

Ans: A, B, C

A nurse in the preoperative holding area is admitting a woman prior to reduction mammoplasty. What should the nurse include in the care given to this patient? Select all that apply. A) Establishing an IV line B) Verifying the surgical site with the patient C) Taking measures to ensure the patient's comfort D) Applying a grounding device to the patient E) Preparing the medications to be administered in the OR

Ans: A, B, C Feedback: In the holding area, the nurse reviews charts, identifies patients, verifies surgical site and marks site per institutional policy, establishes IV lines, administers medications, if prescribed, and takes measures to ensure each patient's comfort. A nurse in the preoperative holding area does not prepare medications to be administered by anyone else. A grounding device is applied in the OR.

The nurse is preparing to send a patient to the OR for a scheduled surgery. What should the nurse ensure is on the chart when it accompanies the patient to surgery? Select all that apply. A) Laboratory reports B) Nurses' notes C) Verification form D) Social work assessment E) Dietician's assessment

Ans: A, B, C Feedback: The completed chart (with the preoperative checklist and verification form) accompanies the patient to the OR with the surgical consent form attached, along with all laboratory reports and nurses' records. Any unusual last-minute observations that may have a bearing on anesthesia or surgery are noted prominently at the front of the chart. The social work and dietician's assessments are not normally necessary when the patient goes to surgery.

An intraoperative nurse is applying interventions that will address surgical patients risks for perioperative positioning injury. Which of the following factors contribute to this increased risk for injury in the intraoperative phase of the surgical experience? Select all that apply. A) Absence of reflexes B) Diminished ability to communicate C) Loss of pain sensation D) Nausea resulting from anesthetic E) Reduced blood pressure

Ans: A, B, C Feedback: Loss of pain sense, reflexes, and ability to communicate subjects the intraoperative patient to possible injury. Nausea and low blood pressure are not central factors that contribute to this risk, though they are adverse outcomes.

A patient has a documented history of allergies presents to the clinic. She states that she is frustrated by her chronic nasal congestion, anosmia (inability to smell) and inability to concentrate. The nurse should identify which of the following nursing diagnoses? A) Deficient Knowledge of Self-Care Practices Related to Allergies B) Ineffective Individual Coping with Chronicity of Condition and Need for Environmental Modification C) Acute Confusion Related to Cognitive Effects of Allergic Rhinitis D) Disturbed Body Image Related to Sequelae of Allergic Rhinitis

Ans: B

The nurse is planning the care of a patient who has type 1 diabetes and who will be undergoing knee replacement surgery. This patient's care plan should reflect an increased risk of what postsurgical complications? Select all that apply. A) Hypoglycemia B) Delirium C) Acidosis D) Glucosuria E) Fluid overload

Ans: A, C, D Feedback: Hypoglycemia may develop during anesthesia or postoperatively from inadequate carbohydrates or excessive administration of insulin. Hyperglycemia, which can increase the risk for surgical wound infection, may result from the stress of surgery, which can trigger increased levels of catecholamine. Other risks are acidosis and glucosuria. Risks of fluid overload and delirium are not normally increased.

When creating plans of nursing care for patients who are undergoing surgery using general anesthetic, what nursing diagnoses should the nurse identify? Select all that apply. A) Disturbed sensory perception related to anesthetic B) Risk for impaired nutrition: less than body requirements related to anesthesia C) Risk of latex allergy response related to surgical exposure D) Disturbed body image related to anesthesia E) Anxiety related to surgical concerns

Ans: A, C, E Feedback: Based on the assessment data, some major nursing diagnoses may include the following: anxiety related to surgical or environmental concerns, risk of latex allergy response due to possible exposure to latex in the OR environment, risk for perioperative positioning injury related to positioning in the OR, risk for injury related to anesthesia and surgical procedure, or disturbed sensory perception (global) related to general anesthesia or sedation. Malnutrition and disturbed body image are much less likely.

The nurse is applying standard precautions in the care of a patient who has an immunodeficiency. What are key elements of standard precautions? Select all that apply. A) Using appropriate personal protective equipment B) Placing patients in negative-pressure isolation rooms C) Placing patients in positive-pressure isolation rooms D) Using safe injection practices E) Performing hand hygiene

Ans: A, D, E

Through the process of hematopoiesis, stem cells differentiate into either myeloid or lymphoid stem cells. Into what do myeloid stem cells further differentiate? Select all that apply. A) Leukocytes B) Natural killer cells C) Cytokines D) Platelets E) Erythrocytes

Ans: A, D, E

The surgical nurse is preparing to send a patient from the presurgical area to the OR and is reviewing the patient's informed consent form. What are the criteria for legally valid informed consent? Select all that apply. A) Consent must be freely given. B) Consent must be notarized. C) Consent must be signed on the day of surgery. D) Consent must be obtained by a physician. E) Signature must be witnessed by a professional staff member.

Ans: A, D, E Feedback: Valid consent must be freely given, without coercion. Consent must be obtained by a physician and the patient's signature must be witnessed by a professional staff member. It does not need to be signed on the same day as the surgery and it does not need to be notarized.

A 20-year-old patient with an immunodeficiency is admitted to the unit with an acute episode of upper airway edema. This is the fifth time in the past 3 months that the patient has had such as episode. As the nurse caring for this patient, you know that the patient may have a deficiency of what? A) Interferons B) C1esterase inhibitor C) IgG D) IgA

Ans: B

A 6-month-old infant has been diagnosed with X-linked agammaglobulinemia and the parents do not understand why their baby did not develop an infection during the first months of life. The nurse should describe what phenomenon? A) Cell-mediated immunity in infants B) Passive acquired immunity C) Phagocytosis D) Opsonization

Ans: B

A child has been diagnosed with a severe walnut allergy after suffering an anaphylactic reaction. What is a priority for health education? A) The need to begin immunotherapy as soon as possible B) The need for the parents to carry an epinephrine pen C) The need to vigilantly maintain the child's immunization status D) The need for the child to avoid all foods that have a high potential for allergies

Ans: B

A client's health history reveals daily consumption of two to three bottles of wine. The nurse should plan assessments and interventions in light of the patient's increased risk for what hematologic disorder? A) Leukemia B) Anemia C) Thrombocytopenia D) Lymphoma

Ans: B

A nurse at an allergy clinic is providing education for a patient starting immunotherapy for the treatment of allergies. What education should the nurse prioritize? A) The importance of scheduling appointments for the same time each month B) The importance of keeping appointments for desensitization procedures C) The importance of avoiding antihistamines for the duration of treatment D) The importance of keeping a diary of reactions to the immunotherapy

Ans: B

A nurse has admitted a patient diagnosed with severe combined immunodeficiency disease (SCID) to the unit. The patient's orders include IVIG. How will the patient's dose of IVIG be determined? A) The patient will receive 25 to 50 mg/kg of body weight. B) The dose will be determined by the patient's response. C) The dose will be determined by body surface area. D) The patient will receive a one-time bolus followed by 100- to 150-mg doses.

Ans: B

A nurse has created a plan of care for an immunodeficient patient, specifying that care providers take the patient's pulse and respiratory rate for a full minute. What is the rationale for this aspect of care? A) Respirations affect heart rate in immunodeficient patients. B) These patients' blunted inflammatory responses can cause subtle changes in status. C) Hemodynamic instability is one of the main complications of immunodeficiency. D) Immunodeficient patients are prone to ventricular tachycardia and atrial fibrillation.

Ans: B

A nurse has participated in organizing a blood donation drive at a local community center. Which of the following individuals would most likely be disallowed from donating blood? A) A man who is 81 years of age B) A woman whose blood pressure is 88/51 mm Hg C) A man who donated blood 4 months ago D) A woman who has type 1 diabetes

Ans: B

A nurse is caring for a patient who has an immunodeficiency. What assessment finding should prompt the nurse to consider the possibility that the patient is developing an infection? A) Uncharacteristic aggression B) Persistent diarrhea C) Pruritis (itching) D) Constipation

Ans: B

A patient is brought to the emergency department (ED) in a state of anaphylaxis. What is the ED nurse's priority for care? A) Monitor the patient's level of consciousness. B) Protect the patient's airway. C) Provide psychosocial support. D) Administer medications as ordered.

Ans: B

A patient on the medical unit is receiving a unit of PRBCs. Difficult IV access has necessitated a slow infusion rate and the nurse notes that the infusion began 4 hours ago. What is the nurse's most appropriate action? A) Apply an icepack to the blood that remains to be infused. B) Discontinue the remainder of the PRBC transfusion and inform the physician. C) Disconnect the bag of PRBCs, cool for 30 minutes and then administer. D) Administer the remaining PRBCs by the IV direct (IV push) route.

Ans: B

A patient undergoing a hip replacement has autologous blood on standby if a transfusion is needed. What is the primary advantage of autologous transfusions? A) Safe transfusion for patients with a history of transfusion reactions B) Prevention of viral infections from another person's blood C) Avoidance of complications in patients with alloantibodies D) Prevention of alloimmunization

Ans: B

A patient was prescribed an oral antibiotic for the treatment of sinusitis. The patient has now stopped, stating she developed a rash shortly after taking the first dose of the drug. What is the nurse's most appropriate response? A) Encourage the woman to continue with the medication while monitoring her skin condition closely. B) Refer the woman to her primary care provider to have the medication changed. C) Arrange for the woman to go to the nearest emergency department. D) Encourage the woman to take an OTC antihistamine with each dose of the antibiotic.

Ans: B

A patient with severe environmental allergies is scheduled for an immunotherapy injection. What should be included in teaching the patient about this treatment? A) The patient will be given a low dose of epinephrine before the treatment. B) The patient will remain in the clinic to be monitored for 30 minutes following the injection. C) Therapeutic failure occurs if the symptoms to the allergen do not decrease after 3 months. D) The allergen will be administered by the peripheral intravenous route.

Ans: B

A patient's decline in respiratory and renal function has been attributed to Goodpasture syndrome, which is a type II hypersensitivity reaction. What pathologic process underlies the patient's health problem? A) Antigens have bound to antibodies and formed inappropriate immune complexes. B) The patient's body has mistakenly identified a normal constituent of the body as foreign. C) Sensitized T cells have caused cell and tissue damage. D) Mast cells have released histamines that directly cause cell lysis.

Ans: B

A patient's electronic health record states that the patient receives regular transfusions of factor IX. The nurse would be justified in suspecting that this patient has what diagnosis? A) Leukemia B) Hemophilia C) Hypoproliferative anemia D) Hodgkin's lymphoma

Ans: B

A young couple visits the nurse practitioner stating that they want to start a family. The husband states that his brother died of a severe infection at age 6 months. He says he never knew what was wrong but his mother had him undergo blood testing as a child. Based on these statements, what health problem should the nurse practitioner suspect? A) Severe neutropenia B) X-linked agammaglobulinemia C) Drug-induced thrombocytopenia D) Aplastic anemia

Ans: B

An individual has accidentally cut his hand, immediately initiating the process of hemostasis. Following vasoconstriction, what event in the process of hemostasis will take place? A) Fibrin will be activated at the bleeding site. B) Platelets will aggregate at the injury site. C) Thromboplastin will form a clot. D) Prothrombin will be converted to thrombin.

Ans: B

The nurse caring for a patient receiving a transfusion notes that 15 minutes after the infusion of packed red blood cells (PRBCs) has begun, the patient is having difficulty breathing and complains of severe chest tightness. What is the most appropriate initial action for the nurse to take? A) Notify the patient's physician. B) Stop the transfusion immediately. C) Remove the patient's IV access. D) Assess the patient's chest sounds and vital signs.

Ans: B

The nurse in an allergy clinic is educating a new patient about the pathology of the patient's health problem. What response should the nurse describe as a possible consequence of histamine release? A) Constriction of small venules B) Contraction of bronchial smooth muscle C) Dilation of large blood vessels D) Decreased secretions from gastric and mucosal cells

Ans: B

The nurse is describing normal RBC physiology to a patient who has a diagnosis of anemia. The nurse should explain that the RBCs consist primarily of which of the following? A) Plasminogen B) Hemoglobin C) Hematocrit D) Fibrin

Ans: B

The nurse is describing the role of plasminogen in the clotting cascade. Where in the body is plasminogen present? A) Myocardial muscle tissue B) All body fluids C) Cerebral tissue D) Venous and arterial vessel walls

Ans: B

The nurse is providing care for an older adult who has a hematologic disorder. What age-related change in hematologic function should the nurse integrate into care planning? A) Bone marrow in older adults produces a smaller proportion of healthy, functional blood cells. B) Older adults are less able to increase blood cell production when demand suddenly increases. C) Stem cells in older adults eventually lose their ability to differentiate. D) The ratio of plasma to erythrocytes and lymphocytes increases with age.

Ans: B

The parents of a child with muscular dystrophy have the desire to gain additional information regarding services available to children and families affected by this condition. They call the clinic nurse and ask for advice. The nurse's best response is to provide information to the parents related to what? A) Spiritual counseling B) The Muscular Dystrophy Association C) Programs offered by the local school district D) The American Academy of Pediatrics

Ans: B Feedback: While all options will serve to provide a varying level of support to the child and the family, the Muscular Dystrophy Association will provide information specific to programs of patient services and clinical care, research initiatives, and professional and public education.

A junior nursing student is having an observation day in the operating room. Early in the day, the student tells the OR nurse that her eyes are swelling and she is having trouble breathing. What should the nurse suspect? A) Cytotoxic reaction due to contact with the powder in the gloves B) Immune complex reaction due to contact with anesthetic gases C) Anaphylaxis due to a latex allergy D) Delayed reaction due to exposure to cleaning products

Ans: C

The nurse is performing a preoperative assessment on a patient going to surgery. The patient informs the nurse that he drinks approximately two bottles of wine each day and has for the last several years. What postoperative difficulties can the nurse anticipate for this patient? A) Alcohol withdrawal syndrome immediately following surgery B) Alcohol withdrawal syndrome 2 to 4 days after his last alcohol drink C) Alcohol withdrawal syndrome upon administration of general anesthesia D) Alcohol withdrawal syndrome 1 week after his last alcohol drink

Ans: B Feedback: Alcohol withdrawal syndrome may be anticipated between 48 and 96 hours after alcohol withdrawal and is associated with a significant mortality rate when it occurs postoperatively.

A nurse is providing preoperative teaching to a patient who will soon undergo a cardiac bypass. The nurse's teaching plan includes exercises of the extremities. What is the purpose of teaching a patient leg exercises prior to surgery? A) Leg exercises increase the patient's muscle mass postoperatively. B) Leg exercises improve circulation and prevent venous thrombosis. C) Leg exercises help to prevent pressure sores to the sacrum and heels. D) Leg exercise help increase the patient's level of consciousness after surgery.

Ans: B Feedback: Exercise of the extremities includes extension and flexion of the knee and hip joints (similar to bicycle riding while lying on the side) unless contraindicated by type of surgical procedure (e.g., hip replacement). When the patient does leg exercises postoperatively, circulation is increased, which helps to prevent blood clots from forming. Leg exercises do not prevent pressure sores to the sacrum, or increase the patient's level of consciousness. Leg exercises have the potential to increase strength and mobility, but are unlikely to make a change to muscle mass in the short term.

During the care of a preoperative patient, the nurse has given the patient a preoperative benzodiazepine. The patient is now requesting to void. What action should the nurse take? A) Assist the patient to the bathroom. B) Offer the patient a bedpan or urinal. C) Wait until the patient gets to the operating room and is catheterized. D) Have the patient go to the bathroom.

Ans: B Feedback: If a preanesthetic medication is administered, the patient is kept in bed with the side rails raised because the medication can cause lightheadedness or drowsiness. If a patient needs to void following administration of a sedative, the nurse should offer the patient a urinal. The patient should not get out of bed because of the potential for lightheadedness.

You are caring for an 88-year-old woman who is scheduled for a right mastectomy. You know that elderly patients are frequently more anxious prior to surgery than younger patients. What would you increase with this patient to decrease her anxiety? A) Analgesia B) Therapeutic touch C) Preoperative medication D) Sleeping medication the night before surgery

Ans: B Feedback: Older patients report higher levels of preoperative anxiety; therefore, the nurse should be prepared to spend additional time, increase the amount of therapeutic touch utilized, and encourage family members to be present to decrease anxiety. For most patients, nonpharmacologic interventions should be attempted before administering medications.

The nurse is providing preoperative teaching to a patient scheduled for surgery. The nurse is instructing the patient on the use of deep breathing, coughing, and the use of incentive spirometry when the patient states, I don't know why you're focusing on my breathing. My surgery is on my hip, not my chest. What rationale for these instructions should the nurse provide? A) To prevent chronic obstructive pulmonary disease (COPD) B) To promote optimal lung expansion C) To enhance peripheral circulation D) To prevent pneumothorax

Ans: B Feedback: One goal of preoperative nursing care is to teach the patient how to promote optimal lung expansion and consequent blood oxygenation after anesthesia. COPD is not a realistic risk and pneumothorax is also unlikely. Breathing exercises do not primarily affect peripheral circulation.

The policies and procedures on a preoperative unit are being amended to bring them closer into alignment with the focus of the Surgical Care Improvement Project (SCIP). What intervention most directly addresses the priorities of the SCIP? A) Actions aimed at increasing participation of families in planning care B) Actions aimed at preventing surgical site infections C) Actions aimed at increasing interdisciplinary collaboration D) Actions aimed at promoting the use of complementary and alternative medicine (CAM)

Ans: B Feedback: SCIP identifies performance measures aimed at preventing surgical complications, including venous thromboembolism (VTE) and surgical site infections (SSI). It does not explicitly address family participation, interdisciplinary collaboration, or CAM.

You are the nurse caring for an unconscious trauma victim who needs emergency surgery. The patient is a 55-year-old man with an adult son. He is legally divorced and is planning to be remarried in a few weeks. His parents are at the hospital with the other family members. The physician has explained the need for surgery, the procedure to be done, and the risks to the children, the parents, and the fiancé. Who should be asked to sign the surgery consent form? A) The fiancé B) The son C) The physician, acting as a surrogate D) The patient's father

Ans: B Feedback: The patient personally signs the consent if of legal age and mentally capable. Permission is otherwise obtained from a surrogate, who most often is a responsible family member (preferably next of kin) or legal guardian. In this instance, the child would be the appropriate person to ask to sign the consent form as he is the closest relative at the hospital. The fiancé is not legally related to him as the marriage has not yet taken place. The father would only be asked to sign the consent if no children were present to sign. The physician would not sign if family members were available.

A 77-year-old man's coronary artery bypass graft has been successful and discharge planning is underway. When planning the patient's subsequent care, the nurse should know that the postoperative phase of perioperative nursing ends at what time? A) When the patient is returned to his room after surgery B) When a follow-up evaluation in the clinical or home setting is done C) When the patient is fully recovered from all effects of the surgery D) When the family becomes partly responsible for the patient's care

Ans: B Feedback: The postoperative phase begins with the admission of the patient to the PACU and ends with a follow-up evaluation in the clinical setting or home.

A nurse caring for a patient who has an immunosuppressive disorder knows that continual monitoring of the patient is critical. What is the primary rationale behind the need for continual monitoring? A) So that the patient's functional needs can be met immediately B) So that medications can be given as ordered and signs of adverse reactions noted C) So that early signs of impending infection can be detected and treated D) So that the nurse's documentation can be thorough and accurate

Ans: C

A patient is on call to the OR for an aortobifemoral bypass and the nurse administers the ordered preoperative medication. After administering a preoperative medication to the patient, what should the nurse do? A) Encourage light ambulation. B)Place the bed in a low position with the side rails up. C) Tell the patient that he will be asleep before he leaves for surgery. D) Take the patient's vital signs every 15 minutes.

Ans: B Feedback: When the preoperative medication is given, the bed should be placed in low position with the side rails raised. The patient should not get up without assistance. The patient may not be asleep, but he may be drowsy. Vital signs should be taken before the preoperative medication is given; vital signs are not normally required every 15 minutes after administration.

The nurse is preparing a patient for surgery. The patient states that she is very nervous and really does not understand what the surgical procedure is for or how it will be performed. What is the most appropriate nursing action for the nurse to take? A) Have the patient sign the informed consent and place it in the chart. B) Call the physician to review the procedure with the patient. C) Explain the procedure clearly to the patient and her family. D) Provide the patient with a pamphlet explaining the procedure.

Ans: B Feedback: While the nurse may ask the patient to sign the consent form and witness the signature, it is the surgeon's responsibility to provide a clear and simple explanation of what the surgery will entail prior to the patient giving consent. The surgeon must also inform the patient of the benefits, alternatives, possible risks, complications, disfigurement, disability, and removal of body parts as well as what to expect in the early and late postoperative periods. The nurse clarifies the information provided, and, if the patient requests additional information, the nurse notifies the physician. The consent formed should not be signed until the patient understands the procedure that has been explained by the surgeon. The provision of a pamphlet will benefit teaching the patient about the surgical procedure, but will not substitute for the information provided by the physician.

A patient has just returned to the unit from the PACU after surgery for a tumor within the spine. The patient complains of pain. When positioning the patient for comfort and to reduce injury to the surgical site, the nurse will position to patient in what position? A) In the lithotomy position B) In a flat side-lying position C) In the Trendelenberg position D) In the reverse Trendelenberg position

Ans: B Feedback: After spinal surgery, the bed is usually kept flat initially. The side-lying position is usually the most comfortable because this position imposes the least pressure on the surgical site. The lithotomy position, Trendelenberg position, and reverse Trendelenberg position are inappropriate for this patient.

A 13-year-old patient is admitted to the adolescent unit with a suspected brain tumor. The patient asks the nurse which diagnostic test is the most helpful in the diagnosis of brain tumors. What is the nurse's best response? A) Computed tomography (CT) scan B) Magnetic resonance imaging (MRI) C) Brain biopsy D) Blood work with (ACTH) levels

Ans: B Feedback: An MRI is the most helpful in the diagnosis of brain tumors. Its use has resulted in the detection of smaller lesions; it is particularly helpful in detecting tumors in the brainstem and pituitary regions, where bone interferes with CT. A brain biopsy and blood work with ACTH levels do not diagnose brain tumors.

You are caring for a patient scheduled for a cervical discectomy tomorrow. You are presenting pre-operative teaching to the patient. Your teaching includes what potential complications? A) Damage to the vocal cords B) Hematoma at the surgical site C) Airway edema D) Hemorrhage

Ans: B Feedback: Based on all the assessment data, the potential complications may include hematoma at the surgical site, resulting in cord compression and neurologic deficit; and recurrent or persistent pain after surgery.

A patient diagnosed with a pituitary adenoma has arrived on your unit. Based upon your initial assessment, the patient is most likely to exhibit what clinical manifestations? A) Decreased intracranial pressure B) Headache C) Hyperthalamic disorders D) Restlessness

Ans: B Feedback: Pressures from pituitary adenomas may cause headaches, visual dysfunction, and hypothalamic disorders. Restlessness is not a typical manifestation of pituitary adenomas. Pressure from pituitary adenomas would increase ICP and they would not cause hyperthalmic disorders.

A 25-year-old female with brain metastases asks the nurse about the length of time she has to live. Based upon the fact that the patient is not receiving treatment for the brain metastases, the nurse's best response is that patients with this condition generally have a mean survival time of what? A) 2 weeks B) 1 month C) 6 months D) 1 year

Ans: B Feedback: The median survival time for patients with no treatment for brain metastases is 1 month; with corticosteroid treatment alone it is 2 months; radiation therapy extends the median survival to 3 to 6 months. Therefore, options A, C, and D are incorrect.

The OR will be caring for a patient who will receive a transsacral block. For what patient would the use of a transsacral block be appropriate for pain control? A) A middle-aged man who is scheduled for a thoracotomy B) An older adult man who will undergo an inguinal hernia repair C) A 50-year-old woman who will be having a reduction mammoplasty D) A child who requires closed reduction of a right humerus fracture

Ans: B Feedback: A transsacral block produces anesthesia for the perineum and lower abdomen. Both a thoracotomy and breast reduction are in the chest region, and a transsacral block would not provide pain control for these procedures. A closed reduction of a right humerus is a procedure on the right arm, and a transsacral block would not provide pain control.

The anesthetist is coming to the surgical admissions unit to see a patient prior to surgery scheduled for tomorrow morning. Which of the following is the priority information that the nurse should provide to the anesthetist during the visit? A) Last bowel movement B) Latex allergy C) Number of pregnancies D) Difficulty falling asleep

Ans: B Feedback: Due to the increased number of patients with latex allergies, it is essential to identify this allergy early on so precautions can be taken in the OR. The anesthetist should be informed of any allergies. This is a priority over pregnancy history, insomnia, or recent bowel function, though some of these may be relevant.

A nurse has included the nursing diagnosis of Risk for Latex Allergy Response in a patient's plan of care. The presence of what chronic health problem would most likely prompt this diagnosis? A) Herpes simplex B) HIV C) Spina bifida D) Hypogammaglobulinemia

Ans: C

The nurse knows that elderly patients are at higher risk for complications and adverse outcomes during the intraoperative period. What is the best rationale for this phenomenon? A) The elderly patient has a more angular bone structure than a younger person. B) The elderly patient has reduced ability to adjust rapidly to emotional and physical stress. C) The elderly patient has impaired thermoregulatory mechanisms, which increase susceptibility to hyperthermia. D) The elderly patient has an impaired ability to decrease his or her metabolic rate.

Ans: B Feedback: Factors that affect the elderly surgical patient in the intraoperative period include the following: impaired ability to increase metabolic rate and impaired thermoregulatory mechanisms increase susceptibility to hypothermia. Bone loss (25% in women, 12% in men) necessitates careful manipulation and positioning during surgery. Reduced ability to adjust rapidly to emotional and physical stress influences surgical outcomes and requires meticulous observation of vital functions. Older adults do not have more angular bones than younger people.

The patients surgery is nearly finished and the surgeon has opted to use tissue adhesives to close the surgical wound. This requires the nurse to prioritize assessments related to what complication? A) Hypothermia B) Anaphylaxis C) Infection D) Malignant hyperthermia

Ans: B Feedback: Fibrin sealants are used in a variety of surgical procedures, and cyanoacrylate tissue adhesives are used to close wounds without the use of sutures. These sealants have been implicated in allergic reactions and anaphylaxis. There is not an increased risk of malignant hyperthermia, hypothermia, or infection because of the use of tissue adhesives.

The intraoperative nurse is implementing a care plan that addresses the surgical patients risk for vomiting. Interventions that address the potential for vomiting reduce the risk of what subsequent surgical complication? A) Impaired skin integrity B) Hypoxia C) Malignant hyperthermia D) Hypothermia

Ans: B Feedback: If the patient aspirates vomitus, an asthma-like attack with severe bronchial spasms and wheezing is triggered. Pneumonitis and pulmonary edema can subsequently develop, leading to extreme hypoxia. Vomiting can cause choking, but the question asks about aspirated vomitus. Malignant hyperthermia is an adverse reaction to anesthesia. Aspirated vomitus does not cause hypothermia. Vomiting does not result in impaired skin integrity.

The surgical patient is a 35-year-old woman who has been administered general anesthesia. The nurse recognizes that the patient is in stage II (the excitement stage) of anesthesia. Which intervention would be most appropriate for the nurse to implement during this stage? A) Rub the patients back. B) Restrain the patient. C) Encourage the patient to express feelings. D) Stroke the patients hand.

Ans: B Feedback: In stage II, the patient may struggle, shout, or laugh. The movements of the patient may be uncontrolled, so it is essential the nurse help to restrain the patient for safety. None of the other listed actions protects the patients safety.

Verification that all required documentation is completed is an important function of the intraoperative nurse. The intraoperative nurse should confirm that the patients accompanying documentation includes which of the following? A) Discharge planning B) Informed consent C) Analgesia prescription D) Educational resources

Ans: B Feedback: It is important to review the patients record for the following: correct informed surgical consent, with patients signature; completed records for health history and physical examination; results of diagnostic studies; and allergies (including latex). Discharge planning records and prescriptions are not normally necessary. Educational resources would not be included at this stage of the surgical process.

The OR nurse is taking the patient into the OR when the patient informs the operating nurse that his grandmother spiked a 104F temperature in the OR and nearly died 15 years ago. What relevance does this information have regarding the patient? A) The patient may be experiencing presurgical anxiety. B) The patient may be at risk for malignant hyperthermia. C) The grandmothers surgery has minimal relevance to the patients surgery. D) The patient may be at risk for a sudden onset of postsurgical infection.

Ans: B Feedback: Malignant hyperthermia is an inherited muscle disorder chemically induced by anesthetic agents. Identifying patients at risk is imperative because the mortality rate is 50%. The patients anxiety is not relevant, the grandmothers surgery is very relevant, and all patients are at risk for hypothermia.

An OR nurse is teaching a nursing student about the principles of surgical asepsis as a requirement in the restricted zone of the operating suite. What personal protective equipment should the nurse wear at all times in the restricted zone of the OR? A) Reusable shoe covers B) Mask covering the nose and mouth C) Goggles D) Gloves

Ans: B Feedback: Masks are worn at all times in the restricted zone of the OR. Shoe covers are worn one time only; goggles and gloves are worn as required, but not necessarily at all times.

While the surgical patient is anesthetized, the scrub nurse hears a member of the surgical team make an inappropriate remark about the patients weight. How should the nurse best respond? A) Ignore the comment because the patient is unconscious. B) Discourage the colleague from making such comments. C) Report the comment immediately to a supervisor. D) Realize that humor is needed in the workplace.

Ans: B Feedback: Patients, whether conscious or unconscious, should not be subjected to excess noise, inappropriate conversation, or, most of all, derogatory comments. The nurse must act as an advocate on behalf of the patient and discourage any such remarks. Reporting to a supervisor, however, is not likely necessary.

A patient has been admitted with a phagocytic cell disorder and the nurse is reviewing the most common health problems that accompany these disorders. The nurse should identify which of the following? Select all that apply. A) Inflammatory bowel disease B) Chronic otitis media C) Cutaneous abscesses D) Pneumonia E) Cognitive deficits

Ans: B, C, D

A child has been transported to the emergency department (ED) after a severe allergic reaction. The ED nurse is evaluating the patient's respiratory status. How should the nurse evaluate the patient's respiratory status? Select all that apply. A) Facilitate lung function testing. B) Assess breath sounds. C) Measure the child's oxygen saturation by oximeter. D) Monitor the child's respiratory pattern. E) Assess the child's respiratory rate.

Ans: B, C, D, E

A child is undergoing testing for food allergies after experiencing unexplained signs and symptoms of hypersensitivity. What food items would the nurse inform the parents are common allergens? A) Citrus fruits and rice B) Root vegetables and tomatoes C) Eggs and wheat D) Hard cheeses and vegetable oils

Ans: C

A nurse is admitting an adolescent patient with a diagnosis of ataxia-telangiectasis. Which of the following nursing diagnoses should the nurse include in the patient's plan of care? A) Fatigue Related to Pernicious Anemia B) Risk for Constipation Related to Decreased Gastric Motility C) Risk for Falls Due to Loss of Muscle Coordination D) Disturbed Kinesthetic Sensory Perception Related to Vascular Changes

Ans: C

A nurse is aware of the need to assess patients' risks for anaphylaxis. What health care procedure constitutes the highest risk for anaphylaxis? A) Administration of the measles-mumps-rubella (MMR) vaccine B) Rapid administration of intravenous fluids C) Computed tomography with contrast solution D) Administration of nebulized bronchodilators

Ans: C

A nurse would identify that a colleague needs additional instruction on standard precautions when the colleague exhibits which of the following behaviors? A) The nurse wears face protection, gloves, and a gown when irrigating a wound. B) The nurse washes the hands with a waterless antiseptic agent after removing a pair of soiled gloves. C) The nurse puts on a second pair of gloves over soiled gloves while performing a bloody procedure. D) The nurse places a used needle and syringe in the puncture-resistant container without capping the needle.

Ans: C

A patient has been living with seasonal allergies for many years, but does not take antihistamines, stating, "When I was young I used to take antihistamines, but they always put me to sleep." How should the nurse best respond? A) "Newer antihistamines are combined with a stimulant that offsets drowsiness." B) "Most people find that they develop a tolerance to sedation after a few months." C) "The newer antihistamines are different than in years past, and cause less sedation." D) "Have you considered taking them at bedtime instead of in the morning?"

Ans: C

A patient has been scheduled for a bone marrow biopsy and admits to the nurse that she is worried about the pain involved with the procedure. What patient education is most accurate? A) "You'll be given painkillers before the test, so there won't likely be any pain?" B) "You'll feel some pain when the needle enters your skin, but none when the needle enters the bone because of the absence of nerves in bone." C) "Most people feel some brief, sharp pain when the needle enters the bone." D) "I'll be there with you, and I'll try to help you keep your mind off the pain."

Ans: C

A patient has come to the OB/GYN clinic due to recent heavy menstrual flow. Because of the patient's consequent increase in RBC production, the nurse knows that the patient may need to increase her daily intake of what substance? A) Vitamin E B) Vitamin D C) Iron D) Magnesium

Ans: C

A patient has presented with signs and symptoms that are consistent with contact dermatitis. What aspect of care should the nurse prioritize when working with this patient? A) Promoting adequate perfusion in affected regions B) Promoting safe use of topical antihistamines C) Identifying the offending agent, if possible D) Teaching the patient to safely use an EpiPen

Ans: C

A patient with a diagnosis of common variable immunodeficiency begins to develop thick, sticky, tenacious sputum. The patient has a history of episodes of pneumonia at least one time per year for the last 10 years. What does the nurse suspect the patient is developing? A) Pulmonary edema B) A pulmonary neoplasm C) Bronchiectasis D) Emphysema

Ans: C

A patient with a diagnosis of primary immunodeficiency informs the nurse that he has been experiencing a new onset of a dry cough and occasional shortness of breath. After determining that the patient's vital signs are within reference ranges, what action should the nurse take? A) Administer a nebulized bronchodilator. B) Perform oral suctioning. C) Assess the patient for signs and symptoms of infection. D) Teach the patient deep breathing and coughing exercises.

Ans: C

A patient with a hematologic disorder asks the nurse how the body forms blood cells. The nurse should describe a process that takes place where? A) In the spleen B) In the kidneys C) In the bone marrow D) In the liver

Ans: C

A patient's low hemoglobin level has necessitated transfusion of PRBCs. Prior to administration, what action should the nurse perform? A) Have the patient identify his or her blood type in writing. B) Ensure that the patient has granted verbal consent for transfusion. C) Assess the patient's vital signs to establish baselines. D) Facilitate insertion of a central venous catheter.

Ans: C

A patient's primary immunodeficiency disease is characterized by the inability of white blood cells to initiate an inflammatory response to infectious organisms. What is this patient's most likely diagnosis? A) Chronic granulomatous disease B) Wiskott-Aldrich syndrome C) Hyperimmunoglobulinemia E syndrome D) Common variable immunodeficiency

Ans: C

A teenager is diagnosed with cellulitis of the right knee and fails to respond to oral antibiotics. He then develops osteomyelitis of the right knee, prompting a detailed diagnostic workup that reveals a phagocytic disorder. This patient faces an increased risk of what complication? A) Thrombocytopenia B) HIV/AIDS C) Neutropenia D) Hemophilia

Ans: C

An immunocompromised patient is being treated in the hospital. The nurse's assessment reveals that the patient's submandibular lymph nodes are swollen, a finding that represents a change from the previous day. What is the nurse's most appropriate action? A) Administer a PRN dose of acetaminophen as ordered. B) Monitor the patient's vital signs q2h for the next 24 hours. C) Inform the patient's primary care provider of this finding. D) Implement standard precautions in the patient's care.

Ans: C

Fresh-frozen plasma (FFP) has been ordered for a hospital patient. Prior to administration of this blood product, the nurse should prioritize what patient education? A) Infection risks associated with FFP administration B) Physiologic functions of plasma C) Signs and symptoms of a transfusion reaction D) Strategies for managing transfusion-associated anxiety

Ans: C

The nurse educator is differentiating primary immunodeficiency diseases from secondary immunodeficiencies. What is the defining characteristic of primary immunodeficiency diseases? A) They require IVIG as treatment. B) They are the result of intrauterine infection. C) They have a genetic origin. D) They are communicable.

Ans: C

The nursing instructor is talking with the junior nursing class about glial cell tumors. The instructor tells the students that the most common type of glioma is an astrocytoma and that astrocytomas are graded from I to IV, indicating the degree of malignancy. What would the instructor tell the students the grade is based on? A) Cellular density, number of cells, and appearance B) Size of cells, number of cells, and appearance C) Cellular density, cell mitosis, and appearance D) Cell mitosis, size of cells, and appearance

Ans: C Feedback: Astrocytomas are the most common type of glioma and are graded from I to IV, indicating the degree of malignancy. The grade is based on cellular density, cell mitosis, and appearance. Usually, these tumors spread by infiltrating into the surrounding neural connective tissue and therefore cannot be totally removed without causing considerable damage to vital structures. The grading of the tumor is not based on the size of the cells or the number of cells.

You are covering patients for a nurse who is at dinner. One of those patients had a cervical diskectomy earlier today. The patient calls you to her room and tells you she is having severe pain and that it came on suddenly. What would you do? A) Call for an entubation tray B) Take the bandage off C) Call the surgeon D) Increase their pain medicine

Ans: C Feedback: If the patient experiences a sudden increase in pain, extrusion of the graft may have occurred, requiring reoperation. A sudden increase in pain should be promptly reported to the surgeon. You would not call for an entubation tray; if the procedure was an anterior cervical diskectomy the entubation tray should already be in the room. Taking the bandage off is only done at the surgeon's order, as is increasing the pain medicine.

The oncology staff educator is precepting a graduate nurse. They are admitting a 77-year-old female patient with a diagnosis of stage IV astrocytoma. The graduate nurse asks how long the lady has known she has cancer. What would be the staff educator's best response? A) "She hasn't known very long because she is pretty good at hiding things that change from her family." B) "She hasn't known very long because an early sign is personality change and elderly people don't know that their personality has changed." C) "She hasn't known very long because early signs and symptoms of intracranial tumors can be overlooked in the elderly because people think they are the result of normal aging." D) "She hasn't known very long because elderly people have a tendency not to go to the doctor when early signs and symptoms appear."

Ans: C Feedback: In elderly patients, early signs and symptoms of intracranial tumors can be easily overlooked or incorrectly attributed to cognitive and neurologic changes associated with normal aging.

The nurse is writing a care plan for a patient with brain metastases. The nurse decides that an appropriate nursing diagnoses is "Anxiety related to lack of control over the health care needs and situation." In establishing this plan of care for the patient, the nurse will identify which measure as appropriate for the care of this patient? A) The patient will receive anti-anxiety medications every 4 hours. B) The patient's family will be instructed on measures to implement when providing care for the patient. C) The patient will be encouraged to verbalize concerns related to the disease and its treatment. D) The patient will begin a busy schedule of therapy, so that he or she will forget about the anxiety.

Ans: C Feedback: Patients need the opportunity to exercise some control over their situation. A sense of mastery can be gained as they learn to understand the disease and its treatment and how to deal with their feelings. Distraction, assuming care responsibilities, and administering medications will not allow the patient to gain some control over their situation or discuss their feelings.

The nurse is checking the informed consent for a 17-year-old who has just been married and expecting her first child. She is scheduled for a cesarean section. She is still living with her parents and is on her parents' health insurance. When obtaining informed consent for the cesarean section, who is legally responsible for signing? A) Her parents B) Her husband C) The patient D) The obstetrician

Ans: C Feedback: An emancipated minor (married or independently earning his or her own living) may sign his or her own consent form. In this case, the patient is the only person who can provide consent unless she would be neurologically incapacitated or incompetent, in which case her husband would need to provide consent.

The clinic nurse is doing a preoperative assessment of a patient who will be undergoing outpatient cataract surgery with lens implantation in 1 week. While taking the patient's medical history, the nurse notes that this patient had a kidney transplant 8 years ago and that the patient is taking immunosuppressive drugs. For what is this patient at increased risk when having surgery? A) Rejection of the kidney B) Rejection of the implanted lens C) Infection D) Adrenal storm

Ans: C Feedback: Because patients who are immunosuppressed are highly susceptible to infection, great care is taken to ensure strict asepsis. The patient is unlikely to experience rejection or adrenal storm.

A patient is admitted to the ED complaining of severe abdominal pain, stating that he has been vomiting coffee-ground like emesis. The patient is diagnosed with a perforated gastric ulcer and is informed that he needs surgery. When can the patient most likely anticipate that the surgery will be scheduled? A) Within 24 hours B) Within the next week C) Without delay because the bleed is emergent D) As soon as all the day's elective surgeries have been completed

Ans: C Feedback: Emergency surgeries are unplanned and occur with little time for preparation for the patient or the perioperative team. An active bleed is considered an emergency, and the patient requires immediate attention because the disorder may be life threatening. The surgery would not likely be deferred until after elective surgeries have been completed.

The ED nurse is caring for an 11-year-old brought in by ambulance after having been hit by a car. The child's parents are thought to be en route to the hospital but have not yet arrived. No other family members are present and attempts to contact the parents have been unsuccessful. The child needs emergency surgery to save her life. How should the need for informed consent be addressed? A) A social worker should temporarily sign the informed consent. B) Consent should be obtained from the hospital's ethics committee. C) Surgery should be done without informed consent. D) Surgery should be delayed until the parents arrive.

Ans: C Feedback: In an emergency, it may be necessary for the surgeon to operate as a lifesaving measure without the patient's informed consent. However, every effort must be made to contact the patient's family. In such a situation, contact can be made by electronic means. In this scenario, the surgery is considered lifesaving, and the parents are on their way to the hospital and not available. A delay would be unacceptable. Neither a social worker nor a member of the ethics committee may sign.

A patient is scheduled for a bowel resection in the morning and the patient's orders include a cleansing enema tonight. The patient wants to know why this is necessary. The nurse should explain that the cleansing enema will have what therapeutic effect? A) Preventing aspiration of gastric contents B) Preventing the accumulation of abdominal gas postoperatively C) Preventing potential contamination of the peritoneum D) Facilitating better absorption of medications

Ans: C Feedback: The administration of a cleansing enema will allow for satisfactory visualization of the surgical site and to prevent trauma to the intestine or contamination of the peritoneum by feces. It will have no effect on aspiration of gastric contents or the absorption of medications. The patient should expect to develop gas in the postoperative period.

The nurse is caring for a trauma victim in the ED who will require emergency surgery due to injuries. Before the patient leaves the ED for the OR, the patient goes into cardiac arrest. The nurse assists in the successful resuscitation and proceeds to release the patient to the OR staff. When can the ED nurse perform the preoperative assessment? A) When he or she has the opportunity to review the patient's electronic health record B) When the patient arrives in the OR C) When assisting with the resuscitation D) Preoperative assessment is not necessary in this case

Ans: C Feedback: The only opportunity for preoperative assessment may take place at the same time as resuscitation in the ED. Preoperative assessment is necessary, but the nurse could not normally enter the OR to perform this assessment. The health record is an inadequate data source.

In anticipation of a patient's scheduled surgery, the nurse is teaching her to perform deep breathing and coughing to use postoperatively. What action should the nurse teach the patient? A) The patient should take three deep breaths and cough hard three times, at least every 15 minutes for the immediately postoperative period. B) The patient should take three deep breaths and exhale forcefully and then take a quick short breath and cough from deep in the lungs. C) The patient should take a deep breath in through the mouth and exhale through the mouth, take a short breath, and cough from deep in the lungs. D) The patient should rapidly inhale, hold for 30 seconds or as long as possible, and exhale slowly.

Ans: C Feedback: The patient assumes a sitting position to enhance lung expansion. The nurse then demonstrates how to take a deep, slow breath and how to exhale slowly. After practicing deep breathing several times, the patient is instructed to breathe deeply, exhale through the mouth, take a short breath, and cough from deep in the lungs.

The nurse is preparing a patient for surgery prior to her hysterectomy without oophorectomy. The nurse is witnessing the patient's signature on a consent form. Which comment by the patient would best indicate informed consent? A) I know I'll be fine because the physician said he has done this procedure hundreds of times. B) I know I'll have pain after the surgery but they'll do their best to keep it to a minimum. C) The physician is going to remove my uterus and told me about the risk of bleeding. D) Because the physician isn't taking my ovaries, I'll still be able to have children.

Ans: C Feedback: The surgeon must inform the patient of the benefits, alternatives, possible risks, complications, disfigurement, disability, and removal of body parts as well as what to expect in the early and late postoperative periods. The nurse clarifies the information provided, and, if the patient requests additional information, the nurse notifies the physician. In the correct response, the patient is able to tell the nurse what will occur during the procedure and the associated risks. This indicates the patient has a sufficient understanding of the procedure to provide informed consent. Clarification of information given may be necessary, but no additional information should be given. The other listed statements do not reflect an understanding of the surgery to be performed.

The nurse is caring for a patient who is admitted to the ER with the diagnosis of acute appendicitis. The nurse notes during the assessment that the patient's ribs and xiphoid process are prominent. The patient states she exercises two to three times daily and her mother indicates that she is being treated for anorexia nervosa. How should the nurse best follow up these assessment data? A) Inform the postoperative team about the patient's risk for wound dehiscence. B) Evaluate the patient's ability to manage her pain level. C) Facilitate a detailed analysis of the patient's electrolyte levels. D) Instruct the patient on the need for a high-sodium diet to promote healing.

Ans: C Feedback: The surgical team should be informed about the patient's medical history regarding anorexia nervosa. Any nutritional deficiency, such as malnutrition, should be corrected before surgery to provide adequate protein for tissue repair. The electrolyte levels should be evaluated and corrected to prevent metabolic abnormalities in the operative and postoperative phase. The risk of wound dehiscence is more likely associated with obesity. Instruction on proper nutrition should take place in the postoperative period, and a consultation should be made with her psychiatric specialist. Evaluation of pain management is always important, but not particularly significant in this scenario.

The nurse is doing a preoperative assessment of an 87-year-old man who is slated to have a right lung lobe resection to treat lung cancer. What underlying principle should guide the nurse's preoperative assessment of an elderly patient? A) Elderly patients have a smaller lung capacity than younger patients. B) Elderly patients require higher medication doses than younger patients. C) Elderly patients have less physiologic reserve than younger patients. D) Elderly patients have more sophisticated coping skills than younger patients.

Ans: C Feedback: The underlying principle that guides the preoperative assessment, surgical care, and postoperative care is that elderly patients have less physiologic reserve (the ability of an organ to return to normal after a disturbance in its equilibrium) than do younger patients. Elderly patients do not have larger lung capacities than younger patients. Elderly patients cannot necessarily cope better than younger patients and they often require lower doses of medications.

The nurse is caring for a patient who is experiencing pain and anxiety following his prostatectomy. Which intervention will likely best assist in decreasing the patient's pain and anxiety? A) Administration of NSAIDs rather than opioids B) Allowing the patient to increase activity C) Use of guided imagery along with pain medication D) Use of deep breathing and coughing exercises

Ans: C Feedback: The use of guided imagery will enhance pain relief and assist in reduction of anxiety. It may be combined with analgesics. Deep breathing and the increase in activity may produce increased pain. Replacing opioids with NSAIDs may cause an increase in pain.

The nurse is caring for a patient newly diagnosed with a primary brain tumor. The patient asks the nurse where his tumor came from. What would be the nurse's best response? A) Your tumor originated from somewhere outside the CNS. B) Your tumor is pituitary in origin. C) Your tumor originated from cells and structures within the brain. D) Your tumor is from nerve tissue somewhere in your body.

Ans: C Feedback: Primary brain tumors originate from cells and structures within the brain. Secondary brain tumors are metastatic tumors that originate somewhere else in the body. The scenario does not indicate that the patient's tumor is a pituitary tumor or a neuroma; therefore options B and D are incorrect.

A patient is scheduled for surgery the next day and the different phases of the patients surgical experience will require input from members of numerous health disciplines. How should the patients care best be coordinated? A) By planning care using a surgical approach B) By identifying the professional with the most knowledge of the patient C) By implementing an interdisciplinary approach to care D) By using the nursing process to guide all aspects of care and treatment

Ans: C Feedback: An interdisciplinary approach involving the surgeon, anesthesiologist or anesthetist, and nurse is best. This is superior to each of the other listed options.

As an intraoperative nurse, you know that the patients emotional state can influence the outcome of his or her surgical procedure. How would you best reinforce the patients ability to influence outcome? A) Teach the patient strategies for distraction. B) Pair the patient with another patient who has better coping strategies. C) Incorporate cultural and religious considerations, as appropriate. D) Give the patient antianxiety medication.

Ans: C Feedback: Because the patients emotional state remains a concern, the care initiated by preoperative nurses is continued by the intraoperative nursing staff that provides the patient with information and reassurance. The nurse supports coping strategies and reinforces the patients ability to influence outcomes by encouraging active participation in the plan of care incorporating cultural, ethnic, and religious considerations, as appropriate. Buddying a patient is normally inappropriate and distraction may or may not be effective. Nonpharmacologic measures should be prioritized.

A patient will be undergoing a total hip arthroplasty later in the day and it is anticipated that the patient may require blood transfusion during surgery. How can the nurse best ensure the patients safety if a blood transfusion is required? A) Prime IV tubing with a unit of blood and keep it on hold. B) Check that the patients electrolyte levels have been assessed preoperatively. C) Ensure that the patient has had a current cross-match. D) Keep the blood on standby and warmed to body temperature.

Ans: C Feedback: Few patients undergoing an elective procedure require blood transfusion, but those undergoing high-risk procedures may require an intraoperative transfusion. The circulating nurse anticipates this need, checks that blood has been cross-matched and held in reserve, and is prepared to administer blood. Storing the blood at body temperature or in IV tubing would result in spoilage and potential infection.

You are caring for a male patient who has had spinal anesthesia. The patient is under a physicians order to lie flat postoperatively. When the patient asks to go to the bathroom, you encourage him to adhere to the physicians order. What rationale for complying with this order should the nurse explain to the patient? A) Preventing the risk of hypotension B) Preventing respiratory depression C) Preventing the onset of a headache D) Preventing pain at the lumbar injection site

Ans: C Feedback: Lying flat reduces the risk of headache after spinal anesthesia. Hypotension and respiratory depression may be adverse effects of spinal anesthesia associated with the spread of the anesthetic, but lying flat does not help reduce these effects. Pain at the lumbar injection site typically is not a problem.

As an intraoperative nurse, you are the advocate for each of the patients who receives care in the surgical setting. How can you best exemplify the principles of patient advocacy? A) By encouraging the patient to perform deep breathing preoperatively B) By limiting the patients contact with family members preoperatively C) By maintaining each of your patients privacy D) By eliciting informed consent from patients

Ans: C Feedback: Patient advocacy in the OR entails maintaining the patients physical and emotional comfort, privacy, rights, and dignity. Deep breathing is not necessary before surgery and obtaining informed consent is the purview of the physician. Family contact should not be limited.

Maintaining an aseptic environment in the OR is essential to patient safety and infection control. When moving around surgical areas, what distance must the nurse maintain from the sterile field? A) 2 feet B) 18 inches C) 1 foot D) 6 inches

Ans: C Feedback: Sterile areas must be kept in view during movement around the area. At least a 1-foot distance from the sterile field must be maintained to prevent inadvertent contamination.

The OR nurse is providing care for a 25-year-old major trauma patient who has been involved in a motorcycle accident. The nurse should know that the patient is at increased risk for what complication of surgery? A) Respiratory depression B) Hypothermia C) Anesthesia awareness D) Moderate sedation

Ans: C Feedback: The Joint Commission has issued an alert regarding the phenomenon of patients being partially awake while under general anesthesia (referred to as anesthesia awareness). Patients at greatest risk of anesthesia awareness are cardiac, obstetric, and major trauma patients. This patient does not likely face a heightened risk of respiratory depression or hypothermia. Moderate sedation is not a complication.

The perioperative nurse is constantly assessing the surgical patient for signs and symptoms of complications of surgery. Which symptom should first signal to the nurse the possibility that the patient is developing malignant hyperthermia? A) Increased temperature B) Oliguria C) Tachycardia D) Hypotension

Ans: C Feedback: The initial symptoms of malignant hyperthermia are related to cardiovascular and musculoskeletal activity. Tachycardia (heart rate greater than 150 beats per minute) is often the earliest sign. Oliguria, hypotension, and increased temperature are later signs of malignant hyperthermia.

A patient who is scheduled for a skin test informs the nurse that he has been taking corticosteroids to help control his allergy symptoms. What nursing intervention should the nurse implement? A) The patient should take his corticosteroids regularly prior to testing. B) The patient should only be tested for grass, mold, and dust initially. C) The nurse should have an emergency cart available in case of anaphylaxis during the test. D) The patient's test should be cancelled until he is off his corticosteroids.

Ans: D

A 68-year-old patient is scheduled for a bilateral mastectomy. The OR nurse has come out to the holding area to meet the patient and quickly realizes that the patient is profoundly anxious. What is the most appropriate intervention for the nurse to apply? A) Reassure the patient that modern surgery is free of significant risks. B) Describe the surgery to the patient in as much detail as possible. C) Clearly explain any information that the patient seeks. D) Remind the patient that the anesthetic will render her unconscious.

Ans: C Feedback: The nurse can alleviate anxiety by supplying information as the patient requests it. The nurse should not assume that every patient wants as much detail as possible and false reassurance must be avoided. Reminding the patient that she will be unconscious is unlikely to reduce anxiety.

The circulating nurse will be participating in a 78-year-old patients total hip replacement. Which of the following considerations should the nurse prioritize during the preparation of the patient in the OR? A) The patient should be placed in Trendelenburg position. B) The patient must be firmly restrained at all times. C) Pressure points should be assessed and well padded. D) The preoperative shave should be done by the circulating nurse.

Ans: C Feedback: The vascular supply should not be obstructed by an awkward position or undue pressure on a body part. During surgical procedures, the patient is at risk for impairment of skin integrity due to a stationary position and immobility. An elderly patient is at an increased risk of injury and impaired skin integrity. A Trendelenburg position is not indicated for this patient. Once anesthetized for a total hip replacement, the patient cannot move; restraints are not necessary. A preoperative shave is not performed; excess hair is removed by means of a clipper.

The nurse is performing wound care on a 68-year-old postsurgical patient. Which of the following practices violates the principles of surgical asepsis? A) Holding sterile objects above the level of the nurses waist B) Considering a 1 inch (2.5 cm) edge around the sterile field as being contaminated C) Pouring solution onto a sterile field cloth D) Opening the outermost flap of a sterile package away from the body

Ans: C Feedback: Whenever a sterile barrier is breached, the area must be considered contaminated. Pouring solution onto a sterile field cloth violates surgical asepsis because moisture penetrating the cloth can carry microorganisms to the sterile field via capillary action. The other options are practices that help ensure surgical asepsis.

The PACU nurse is caring for a patient who has been deemed ready to go to the postsurgical floor after her surgery. What would the PACU nurse be responsible for reporting to the nurse on the floor? Select all that apply. A) The names of the anesthetics that were used B) The identities of the staff in the OR C) The patient's preoperative level of consciousness D) The presence of family and/or significant others E) The patient's full name

Ans: C, D, E Feedback: The PACU nurse is responsible for informing the floor nurse of the patient's intraoperative factors (e.g., insertion of drains or catheters, administration of blood or medications during surgery, or occurrence of unexpected events), preoperative level of consciousness, presence of family and/or significant others, and identification of the patient by name. The PACU nurse does not tell which anesthetic was used, only the type and amount used. The PACU nurse does not identify the staff that was in the OR with the patient.

The pathophysiology instructor is discussing neoplasms with the pre-nursing students. The instructor explains that the effects of neoplasms are caused by the compression and infiltration of normal tissue. The physiologic changes that result cause what pathophysiologic events? (Mark all that apply.) A) Fatigue B) Slurred speech C) Increased ICP D) Focal neurologic signs E) Altered pituitary function

Ans: C, D, E Feedback: The effects of neoplasms are caused by the compression and infiltration of tissue. A variety of physiologic changes result, causing any or all of the following pathophysiologic events: increased intracranial pressure (ICP) and cerebral edema, seizure activity and focal neurologic signs, hydrocephalus, and altered pituitary function.

A home health nurse is reinforcing health education with a patient who is immunosuppressed and his family. What statement best suggests that the patient has understood the nurse's teaching? A) "My family needs to understand when I can go get the seasonal flu shot." B) "I need to know how to treat my infections in a home setting." C) "I need to understand how to give my platelet transfusions." D) "My family needs to understand that I'll probably need lifelong treatment."

Ans: D

A nurse educator is explaining that patients with primary immunodeficiencies are living longer than in past decades because of advances in medical treatment. This increased longevity is associated with an increased risk of what? A) Chronic obstructive pulmonary disease B) Dementia C) Pulmonary fibrosis D) Cancer

Ans: D

A nurse is caring for a patient who has allergic rhinitis. What intervention would be most likely to help the patient meet the goal of improved breathing pattern? A) Teach the patient to take deep breaths and cough frequently. B) Use antihistamines daily throughout the year. C) Teach the patient to seek medical attention at the first sign of an allergic reaction. D) Modify the environment to reduce the severity of allergic symptoms.

Ans: D

A nurse is caring for a patient who undergoing preliminary testing for a hematologic disorder. What sign or symptom most likely suggests a potential hematologic disorder? A) Sudden change in level of consciousness (LOC) B) Recurrent infections C) Anaphylaxis D) Severe fatigue

Ans: D

A nurse is educating a patient about the role of B lymphocytes. The nurse's description will include which of the following physiologic processes? A) Stem cell differentiation B) Cytokine production C) Phagocytosis D) Antibody production

Ans: D

A nurse is planning the care of a patient who requires immunosuppression to ensure engraftment of depleted bone marrow during a transplantation procedure. What is the most important component of infection control in the care of this patient? A) Administration of IVIG B) Antibiotic administration C) Appropriate use of gloves and goggles D) Thorough and consistent hand hygiene

Ans: D

A nurse is preparing a patient for allergy skin testing. Which of the following precautionary steps is most important for the nurse to follow? A) The patient must not have received an immunization within 7 days. B) The nurse should administer albuterol 30 to 45 minutes prior to the test. C) Prophylactic epinephrine should be administered before the test. D) Emergency equipment should be readily available.

Ans: D

A patient's diagnosis of atrial fibrillation has prompted the primary care provider to prescribe warfarin (Coumadin), an anticoagulant. When assessing the therapeutic response to this medication, what is the nurse's most appropriate action? A) Assess for signs of myelosuppression. B) Review the patient's platelet level. C) Assess the patient's capillary refill time. D) Review the patient's international normalized ratio (INR).

Ans: D

A patient's wound has begun to heal and the blood clot which formed is no longer necessary. When a blood clot is no longer needed, the fibrinogen and fibrin will be digested by which of the following? A) Plasminogen B) Thrombin C) Prothrombin D) Plasmin

Ans: D

An adolescent patient's history of skin hyperreactivity and inflammation has been attributed to atopic dermatitis. The nurse should recognize that this patient consequently faces an increased risk of what health problem? A) Bronchitis B) Systemic lupus erythematosus (SLE) C) Rheumatoid arthritis D) Asthma

Ans: D

An interdisciplinary team has been commissioned to create policies and procedures aimed at preventing acute hemolytic transfusion reactions. What action has the greatest potential to reduce the risk of this transfusion reaction? A) Ensure that blood components are never infused at a rate greater than 125 ml/hr. B) Administer prophylactic antihistamines prior to all blood transfusions. C) Establish baseline vital signs for all patients receiving transfusions. D) Be vigilant in identifying the patient and the blood component.

Ans: D

IVIG has been ordered for the treatment of a patient with an immunodeficiency. Which of the following actions should the nurse perform before administering this blood product? A) Ensure that the patient has a patent central line. B) Ensure that the IVIG is appropriately mixed with normal saline. C) Administer furosemide before IVIG to prevent hypervolemia. D) Weigh the patient before administration to verify the correct dose.

Ans: D

The home health nurse is assessing a patient who is immunosuppressed following a liver transplant. What is the most essential teaching for this patient and the family? A) How to promote immune function through nutrition B) The importance of maintaining the patient's vaccination status C) How to choose antibiotics based on the patient's symptoms D) The need to report any slight changes in the patient's health status

Ans: D

The nurse educating a patient with anemia is describing the process of RBC production. When the patient's kidneys sense a low level of oxygen in circulating blood, what physiologic response is initiated? A) Increased stem cell synthesis B) Decreased respiratory rate C) Arterial vasoconstriction D) Increased production of erythropoietin

Ans: D

The nurse is caring for a patient who has developed scar tissue in many of the areas that normally produce blood cells. What organs can become active in blood cell production by the process of extramedullary hematopoiesis? A) Spleen and kidneys B) Kidneys and pancreas C) Pancreas and liver D) Liver and spleen

Ans: D

The nurse is creating a care plan for a patient suffering from allergic rhinitis. Which of the following outcomes should the nurse identify? A) Appropriate use of prophylactic antibiotics B) Safe injection of corticosteroids C) Improved skin integrity D) Improved coping with lifestyle modifications

Ans: D

The nurse is planning the care of a patient with a nutritional deficit and a diagnosis of megaloblastic anemia. The nurse should recognize that this patient's health problem is due to what? A) Production of inadequate quantities of RBCs B) Premature release of immature RBCs C) Injury to the RBCs in circulation D) Abnormalities in the structure and function RBCs

Ans: D

The nurse is preparing to administer IVIG to a patient who has an immunodeficiency. What nursing guideline should the nurse apply? A) Do not exceed an infusion rate of 300 mL/hr. B) Slow the infusion rate if the patient exhibits signs of a transfusion reaction. C) Weigh the patient immediately after the infusion is complete. D) Administer pretreatment medications as ordered 30 minutes prior to infusion.

Ans: D

The nurse is providing care for a patient who has a diagnosis of hereditary angioedema. When planning this patient's care, what nursing diagnosis should be prioritized? A) Risk for Infection Related to Skin Sloughing B) Risk for Acute Pain Related to Loss of Skin Integrity C) Risk for Impaired Skin Integrity Related to Cutaneous Lesions D) Risk for Impaired Gas Exchange Related to Airway Obstruction

Ans: D

The nurse is providing health education to the parents of a toddler who has been diagnosed with food allergies. What should the nurse teach this family about the child's health problem? A) "Food allergies are a life-long condition, but most families adjust quite well to the necessary lifestyle changes." B) "Consistent use of over-the-counter antihistamines can often help a child overcome food allergies." C) "Make sure that you carry a steroid inhaler with you at all times, especially when you eat in restaurants." D) "Many children outgrow their food allergies in a few years if they avoid the offending foods."

Ans: D

The parents of a 1-month-old infant bring their child to the pediatrician with symptoms of congestive heart failure. The infant is ultimately diagnosed with DiGeorge syndrome. What will prolong this infant's survival? A) Stem cell transplantation B) Long-term antibiotics C) Chemotherapy D) Thymus gland transplantation

Ans: D

Which of the following individuals would be the most appropriate candidate for immunotherapy? A) A patient who had an anaphylactic reaction to an insect sting B) A child with allergies to eggs and dairy C) A patient who has had a positive tuberculin skin test D) A patient with severe allergies to grass and tree pollen

Ans: D

A 37-year-old male is brought to the clinic by his wife because the patient is experiencing loss of motor function and sensation. The physician suspects the patient has a spinal cord tumor and hospitalizes him for diagnostic testing. In preparation for diagnostic studies, the nurse will inform the patient that the most commonly used study to diagnose spinal cord compression from a tumor is what? A) An x-ray B) An ultrasound C) A CT scan D) An MRI scan

Ans: D Feedback: The MRI scan is the most commonly used diagnostic procedure. It is the most sensitive diagnostic tool that is particularly helpful in detecting epidural spinal cord compression and vertebral bone metastases. Therefore options A, B, and C are incorrect.

The nurse is caring for a hospice patient who is scheduled for a surgical procedure to reduce the size of his spinal tumor in an effort to relieve his pain. The nurse should plan this patient care with the knowledge that his surgical procedure is classified as which of the following? A) Diagnostic B) Laparoscopic C)Curative D) Palliative

Ans: D Feedback: A patient on hospice will undergo a surgical procedure only for palliative care to reduce pain, but it is not curative. The reduction of tumor size to relieve pain is considered a palliative procedure. A laparoscopic procedure is a type of surgery that is utilized for diagnostic purposes or for repair. The excision of a tumor is classified as curative. This patient is not having the tumor removed, only the size reduced.

The nurse is creating the care plan for a 70-year-old obese patient who has been admitted to the postsurgical unit following a colon resection. This patient's age and increased body mass index mean that she is at increased risk for what complication in the postoperative period? A) Hyperglycemia B) Azotemia C) Falls D) Infection

Ans: D Feedback: Like age, obesity increases the risk and severity of complications associated with surgery. During surgery, fatty tissues are especially susceptible to infection. In addition, obesity increases technical and mechanical problems related to surgery. Therefore, dehiscence (wound separation) and wound infections are more common. A postoperative patient who is obese will not likely be at greater risk for hyperglycemia, azotemia, or falls.

The nurse is caring for a patient in the postoperative period following an abdominal hysterectomy. The patient states, I don't want to use my pain meds because they'll make me dependent and I won't get better as fast. Which response is most important when explaining the use of pain medication? A) You will need the pain medication for at least 1 week to help in your recovery. What do you mean you feel you won't get better faster? B) Pain medication will help to decrease your pain and increase your ability to breath. Dependency is a risk with pain medication, but you are young and won't have any problems. C) Pain medication can be given by mouth to prevent the risk of dependency that you are worried about. The pain medication has not been shown to affect your risk of a slowed recovery. D) You will move more easily and heal more quickly with decreased pain. Dependence only occurs when it is administered for an extended period of time.

Ans: D Feedback: Postoperatively, medications are administered to relieve pain and maintain comfort without increasing the risk of inadequate air exchange. In the responses by the nurse, (response D) addresses the patient's concerns about drug dependency and the nurse's need to increase the patient's ability to move and recover from surgery. The other responses offer incorrect information, such as increasing the patient's ability to breathe or specifying the time needed to take the medication. Opioids will cause respiratory depression.

The nurse is planning patient teaching for a patient who is scheduled for an open hemicolectomy. The nurse intends to address the topics of incision splinting and leg exercises during this teaching session. When is the best time for the nurse to provide teaching? A) Upon the patient's admission to the postanesthesia care unit (PACU) B) When the patient returns from the PACU C) During the intraoperative period D) As soon as possible before the surgical procedure

Ans: D Feedback: Teaching is most effective when provided before surgery. Preoperative teaching is initiated as soon as possible, beginning in the physician's office, clinic, or at the time of preadmission testing when diagnostic tests are performed. Upon admission to the PACU, the patient is usually drowsy, making this an inopportune time for teaching. Upon the patient's return from the PACU, the patient may remain drowsy. During the intraoperative period, anesthesia alters the patient's mental status, rendering teaching ineffective.

A 90-year-old female patient is scheduled to undergo a partial mastectomy for the treatment of breast cancer. What nursing diagnosis should the nurse prioritize when planning this patient's postoperative care? A) Risk for Delayed Growth and Development related to prolonged hospitalization B) Risk for Decisional Conflict related to discharge planning C) Risk for Impaired Memory related to old age D) Risk for Infection related to reduced immune function

Ans: D Feedback: The lessened physiological reserve of older adults results in an increased risk for infection postoperatively. This physiological consideration is a priority over psychosocial considerations, which may or may not be applicable. Impaired memory is always attributed to a pathophysiological etiology, not advanced age.

The nurse is caring for a 78-year-old female patient who is scheduled for surgery to remove her brain tumor. The patient is very apprehensive and keeps asking when she will get her preoperative medicine. The medicine is ordered to be given on call to OR. When would be the best time to give this medication? A) As soon as possible, in order to alleviate the patient's anxiety B) As the patient is transferred to the OR bed C) When the porter arrives on the floor to take the patient to surgery D) After being notified by the OR and before other preoperative preparations

Ans: D Feedback: The nurse can have the medication ready to administer as soon as a call is received from the OR staff. It usually takes 15 to 20 minutes to prepare the patient for the OR. If the nurse gives the medication before attending to the other details of preoperative preparation, the patient will have at least partial benefit from the preoperative medication and will have a smoother anesthetic and operative course.

The nurse admitting a patient who is insulin dependent to the same-day surgical suite for carpal tunnel surgery. How should this patient's diagnosis of type 1 diabetes affect the care that the nurse plans? A) The nurse should administer a bolus of dextrose IV solution preoperatively. B) The nurse should keep the patient NPO for at least 8 hours preoperatively. C) The nurse should initiate a subcutaneous infusion of long-acting insulin. D) The nurse should assess the patient's blood glucose levels vigilantly.

Ans: D Feedback: The patient with diabetes who is undergoing surgery is at risk for hypoglycemia and hyperglycemia. Close glycemic monitoring is necessary. Dextrose infusion and prolonged NPO status are contraindicated. There is no specific need for an insulin infusion preoperatively.

The clinic nurse is caring for a 42-year-old male oncology patient. He complains of extreme fatigue and weakness after his first week of radiation therapy. Which response by the nurse would best reassure this patient? A) These symptoms usually result from radiation therapy; however, we will continue to monitor your laboratory and x-ray studies. B) These symptoms are part of your disease and are an unfortunately inevitable part of living with cancer. C) Try not to be concerned about these symptoms. Every patient feels this way after having radiation therapy. D) Even though it is uncomfortable, this is a good sign. It means that only the cancer cells are dying.

Ans:A

The nurse is doing preoperative patient education with a 61-year-old male patient who has a 40-pack per year history of cigarette smoking. The patient will undergo an elective bunionectomy at a time that fits his work schedule in a few months. What would be the best instruction to give to this patient? A) Reduce smoking by 50% to prevent the development of pneumonia. B) Stop smoking at least 6 weeks before the scheduled surgery to enhance pulmonary function and decrease infection. C) Aim to quit smoking in the postoperative period to reduce the chance of surgical complications D) Stop smoking 4 to 8 weeks before the scheduled surgery to enhance pulmonary function and decrease infection.

Ans: D Feedback: The reduction of smoking will enhance pulmonary function; in the preoperative period, patients who smoke should be urged to stop 4 to 8 weeks before surgery.

While assessing the patient at the beginning of the shift the nurse inspects a surgical dressing covering the operative site after the patients' cervical discectomy. The nurse notes the drainage is serosanguineous. The nurse recognizes that the characteristic of this drainage may indicate what? A) Abnormal finding B) Postnasal drainage C) Hemorrhage D) A dural leak

Ans: D Feedback: After a cervical discectomy, the nurse will monitor the operative site and dressing covering this site. Serosanguineous drainage may indicate a dural leak. This is not an abnormal finding, postnasal drip, or an indication of hemorrhage.

A patient with an inoperable brain tumor has been told he is terminal. A referral is made to Home Health. The Home Health Nurse is making a care plan for this patient. What do home care needs and interventions focus on in the terminal patient? A) Assessing cognitive status B) Improving nutritional status C) Promoting mobility D) Assistance in self-care

Ans: D Feedback: Home care needs and interventions focus on four major areas: palliation of symptoms and pain control, assistance in self-care, control of treatment complications, and administration of specific forms of treatment, such as parenteral nutrition. Assessing cognitive status is not a focus of home care, although it is documented at each visit. Improving nutritional status is a focus with cancer patients who are not terminal. Promoting mobility is not a focus of home care for this patient.

A patient, diagnosed with cancer of the lung, has just been told he has metastases to the brain. The family should be aware that the neurologic signs and symptoms of metastatic brain disease are what? A) Bradycardia B) Temperature greater than 100.5°F C) Increase in diastolic blood pressure D) Personality changes

Ans: D Feedback: Neurologic signs and symptoms include headache, gait disturbances, visual impairment, personality changes, altered mentation (memory loss and confusion), focal weakness, paralysis, aphasia, and seizures. These problems can be devastating to both patient and family. Bradycardia, elevated temperature, and an increase in diastolic blood pressure are not neurologic signs and symptoms of metastatic brain disease.

A nurse is planning discharge teaching for a patient who underwent a cervical diskectomy. Part of the discharge process is for the nurse to assess the patient's understanding of certain management strategies. What strategies would the nurse assess that would aid her in planning discharge teaching? A) Care of the cervical collar B) Regimen for rest C) Signs and symptoms that need to be reported to the home health nurse D) How to assess vital signs

Ans: D Feedback: The nurse assesses the patient's understanding of these management strategies, limitations, and recommendations: strategies for pain management; signs and symptoms that may indicate complications that should be reported to the physician; use and care of the cervical collar; and to alternate tasks that involve minimal body movement (eg, reading) with tasks that require greater body movement. The assessment of vital signs, regimen for rest, and signs and symptoms to report to the home health nurse would not need to be assessed to aid the nurse in planning discharge teaching.

A male patient with a metastatic brain tumor has a generalized seizure. He is vomiting. The family calls for the nurse. What should the nurse do first? A) Obtain supplies to suction. B) Page or call the physician. C) Insert a wooden tongue blade into his mouth. D) Turn him on his side.

Ans: D Feedback: The nurse's first response should be to place the patient on his side to prevent him from aspirating emesis. Inserting something into the seizing patient's mouth is no longer part of a seizure protocol. Obtaining supplies to suction the patient would be a delegated task. Paging or calling the physician would only be necessary if this is the patient's first seizure.

Prior to a patients scheduled surgery, the nurse has described the way that members of diverse health disciplines will collaborate in the patients care. What is the main rationale for organizing perioperative care in this collaborative manner? A) Historical precedence B) Patient requests C) Physicians needs D) Evidence-based practice

Ans: D Feedback: Collaboration of the surgical team using evidence-based practice tailored to a specific case results in optimal patient care and improved outcomes. None of the other listed factors is the basis for the collaboration of the surgical team.

The nurse is caring for a patient who is scheduled to have a needle biopsy of the pleura. The patient has had a consultation with the anesthesiologist and a conduction block will be used. Which local conduction block can be used to block the nerves leading to the chest? A) Transsacral block B) Brachial plexus block C) Peudental block D) Paravertebral block

Ans: D Feedback: Examples of common local conduction blocks include paravertebral anesthesia, which produces anesthesia of the nerves supplying the chest, abdominal wall, and extremities; brachial plexus block, which produces anesthesia of the arm; and transsacral (caudal) block, which produces anesthesia of the perineum and, occasionally, the lower abdomen. A peudental block was used in obstetrics before the almost-routine use of epidural anesthesia.

A nurse is caring for a patient following knee surgery that was performed under a spinal anesthetic. What intervention should the nurse implement to prevent a spinal headache? A) Have the patient sit in a chair and perform deep breathing exercises. B) Ambulate the patient as early as possible. C) Limit the patients fluid intake for the first 24 hours postoperatively. D) Keep the patient positioned supine.

Ans: D Feedback: Measures that increase cerebrospinal pressure are helpful in relieving headache. These include maintaining a quiet environment, keeping the patient lying flat, and keeping the patient well hydrated. Having the patient sit or stand up decreases cerebrospinal pressure and would not relieve a spinal headache. Limiting fluids is incorrect because it also decreases cerebrospinal pressure and would not relieve a spinal headache.

The nurse is packing a patients abdominal wound with sterile, half-inch Iodoform gauze. During the procedure, the nurse drops some of the gauze onto the patients abdomen 2 inches (5 cm) away from the wound. What should the nurse do? A) Apply povidone-iodine (Betadine) to that section of the gauze and continue packing the wound. B) Pick up the gauze and continue packing the wound after irrigating the abdominal wound with Betadine solution. C) Continue packing the wound and inform the physician that an antibiotic is needed. D) Discard the gauze packing and repack the wound with new Iodoform gauze.

Ans: D Feedback: Sterile surfaces or articles may touch other sterile surfaces or articles and remain sterile; contact with unsterile objects at any point renders a sterile area contaminated. The sterile gauze became contaminated when it was dropped on the patients abdomen. It should be discarded and new Iodoform gauze should be used to pack the wound. Betadine should not be used in the wound unless ordered.

As a perioperative nurse, you know that the National Patient Safety Goals have the potential to improve patient outcomes in a wide variety of health care settings. Which of these Goals has the most direct relevance to the OR? A) Improve safety related to medication use B) Reduce the risk of patient harm resulting from falls C) Reduce the incidence of health care-associated infections D) Reduce the risk of fires

Ans: D Feedback: The National Patient Safety Goals all pertain to the perioperative areas, but the one with the most direct relevance to the OR is the reduction of the risk of surgical fires.

The circulating nurse is admitting a patient prior to surgery and proceeds to greet the patient and discuss what the patient can expect in surgery. What aspect of therapeutic communication should the nurse implement? A) Wait for the patient to initiate dialogue. B) Use medically acceptable terms. C) Give preoperative medications prior to discussion. D) Use a tone that decreases the patients anxiety.

Ans: D Feedback: When discussing what the patient can expect in surgery, the nurse uses basic communication skills, such as touch and eye contact, to reduce anxiety. The nurse should use language the patient can understand. The nurse should not withhold communication until the patient initiates dialogue; the nurse most often needs to initiate and guide dialogue, while still responding to patient leading. Giving medication is not a communication skill.

A 21-year-old patient is positioned on the OR bed prior to knee surgery to correct a sports-related injury. The anesthesiologist administers the appropriate anesthetic. The OR nurse should anticipate which of the following events as the teams next step in the care of this patient? A) Grounding B) Making the first incision C) Giving blood D) Intubating

Ans: D Feedback: When the patient arrives in the OR, the anesthesiologist or anesthetist reassesses the patients physical condition immediately prior to initiating anesthesia. The anesthetic is administered, and the patients airway is maintained through an intranasal intubation, oral intubation, or a laryngeal mask airway. Grounding or blood administration does not normally follow anesthetic administration immediately. An incision would not be made prior to intubation.

A patients current antiretroviral regimen includes nucleoside reverse transcriptase inhibitors (NRTIs). What dietary counseling will the nurse provide based on the patients medication regimen? A) Avoid high-fat meals while taking this medication. B) Limit fluid intake to 2 liters a day. Test Bank - Brunner & Suddarth's Textbook of Medical-Surgical Nursing 14e (Hinkle 2017) 698 C) Limit sodium intake to 2 grams per day. D) Take this medication without regard to meals.

Ans: D Feedback: Many NRTIs exist, but all of them may be safely taken without regard to meals. Protein, fluid, and sodium restrictions play no role in relation to these drugs.

The nurse is addressing condom use in the context of a health promotion workshop. When discussing the correct use of condoms, what should the nurse tell the attendees? A) Attach the condom prior to erection. B) A condom may be reused with the same partner if ejaculation has not occurred. C) Use skin lotion as a lubricant if alternatives are unavailable. D) Hold the condom by the cuff upon withdrawal.

Ans: D Test Bank - Brunner & Suddarth's Textbook of Medical-Surgical Nursing 14e (Hinkle 2017) 696 Feedback: The condom should be unrolled over the hard penis before any kind of sex. The condom should be held by the tip to squeeze out air. Skin lotions, baby oil, petroleum jelly, or cold cream should not be used with condoms because they cause latex deterioration/condom breakage. The condom should be held during withdrawal so it does not come off the penis. Condoms should never be reused.

A 16-year-old female patient experiences alopecia resulting from chemotherapy, prompting the nursing diagnoses of disturbed body image and situational low self- esteem. What action by the patient would best indicate that she is meeting the goal of improved body image and self-esteem? A) The patient requests that her family bring her makeup and wig. B) The patient begins to discuss the future with her family. C) The patient reports less disruption from pain and discomfort. D) The patient cries openly when discussing her disease.

Ans:A

A 50-year-old man diagnosed with leukemia will begin chemotherapy. What would the nurse do to combat the most common adverse effects of chemotherapy? A) Administer an antiemetic. B) Administer an antimetabolite. C) Administer a tumor antibiotic. D) Administer an anticoagulant.

Ans:A

A 60-year-old patient with a diagnosis of prostate cancer is scheduled to have an interstitial implant for high-dose radiation (HDR). What safety measure should the nurse include in this patient's subsequent plan of care? A) Limit the time that visitors spend at the patient's bedside. B) Teach the patient to perform all aspects of basic care independently. C) Assign male nurses to the patient's care whenever possible. D) Situate the patient in a shared room with other patients receiving brachytherapy.

Ans:A

A nurse who works in an oncology clinic is assessing a patient who has arrived for a 2- month follow-up appointment following chemotherapy. The nurse notes that the patient's skin appears yellow. Which blood tests should be done to further explore this clinical sign? A) Liver function tests (LFTs) B) Complete blood count (CBC) C) Platelet count D) Blood urea nitrogen and creatinine

Ans:A

The home health nurse is performing a home visit for an oncology patient discharged 3 days ago after completing treatment for non-Hodgkin lymphoma. The nurse's assessment should include examination for the signs and symptoms of what complication? A) Tumor lysis syndrome (TLS) B) Syndrome of inappropriate antiduretic hormone (SIADH) C) Disseminated intravascular coagulation (DIC) D) Hypercalcemia

Ans:A

The hospice nurse has just admitted a new patient to the program. What principle guides hospice care? A) Care addresses the needs of the patient as well as the needs of the family. B) Care is focused on the patient centrally and the family peripherally. C) The focus of all aspects of care is solely on the patient. D) The care team prioritizes the patient's physical needs and the family is responsible for the patient's emotional needs.

Ans:A

The nurse is caring for a patient who is to begin receiving external radiation for a malignant tumor of the neck. While providing patient education, what potential adverse effects should the nurse discuss with the patient? A) Impaired nutritional status B) Cognitive changes C) Diarrhea D) Alopecia

Ans:A

While a patient is receiving IV doxorubicin hydrochloride for the treatment of cancer, the nurse observes swelling and pain at the IV site. The nurse should prioritize what action? A) Stopping the administration of the drug immediately B) Notifying the patient's physician C) Continuing the infusion but decreasing the rate D) Applying a warm compress to the infusion site

Ans:A

You are caring for a patient who has just been told that her stage IV colon cancer has recurred and metastasized to the liver. The oncologist offers the patient the option of surgery to treat the progression of this disease. What type of surgery does the oncologist offer? A) Palliative B) Reconstructive C) Salvage D) Prophylactic

Ans:A

The nurse is describing some of the major characteristics of cancer to a patient who has recently received a diagnosis of malignant melanoma. When differentiating between benign and malignant cancer cells, the nurse should explain differences in which of the following aspects? Select all that apply. A) Rate of growth B) Ability to cause death C) Size of cells D) Cell contents E) Ability to spread

Ans:A, B, E

You are caring for an adult patient who has developed a mild oral yeast infection following chemotherapy. What actions should you encourage the patient to perform? Select all that apply. A) Use a lip lubricant. B) Scrub the tongue with a firm-bristled toothbrush. C) Use dental floss every 24 hours. D) Rinse the mouth with normal saline. E) Eat spicy food to aid in eradicating the yeast.

Ans:A, C, D

A 54-year-old has a diagnosis of breast cancer and is tearfully discussing her diagnosis with the nurse. The patient states, ìThey tell me my cancer is malignant, while my coworker's breast tumor was benign. I just don't understand at all. When preparing a response to this patient, the nurse should be cognizant of what characteristic that distinguishes malignant cells from benign cells of the same tissue type? A) Slow rate of mitosis of cancer cells B) Different proteins in the cell membrane C) Differing size of the cells D) Different molecular structure in the cells

Ans:B

A nurse is creating a plan of care for an oncology patient and one of the identified nursing diagnoses is risk for infection related to myelosuppression. What intervention addresses the leading cause of infection-related death in oncology patients? A) Encourage several small meals daily. B) Provide skin care to maintain skin integrity. C) Assist the patient with hygiene, as needed. D) Assess the integrity of the patient's oral mucosa regularly.

Ans:B

A patient newly diagnosed with cancer is scheduled to begin chemotherapy treatment and the nurse is providing anticipatory guidance about potential adverse effects. When addressing the most common adverse effect, what should the nurse describe? A) Pruritis (itching) B) Nausea and vomiting C) Altered glucose metabolism D) Confusion

Ans:B

A patient with a diagnosis of gastric cancer has been unable to tolerate oral food and fluid intake and her tumor location precludes the use of enteral feeding. What intervention should the nurse identify as best meeting this patient's nutritional needs? A) Administration of parenteral feeds via a peripheral IV B) TPN administered via a peripherally inserted central catheter C) Insertion of an NG tube for administration of feeds D) Maintaining NPO status and IV hydration until treatment completion

Ans:B

A patient's most recent diagnostic imaging has revealed that his lung cancer has metastasized to his bones and liver. What is the most likely mechanism by which the patient's cancer cells spread? A) Hematologic spread B) Lymphatic circulation C) Invasion D) Angiogenesis

Ans:B

An oncology nurse educator is providing health education to a patient who has been diagnosed with skin cancer. The patient's wife has asked about the differences between normal cells and cancer cells. What characteristic of a cancer cell should the educator cite? A) Malignant cells contain more fibronectin than normal body cells. B) Malignant cells contain proteins called tumor-specific antigens. C) Chromosomes contained in cancer cells are more durable and stable than those of normal cells. D) The nuclei of cancer cells are unusually large, but regularly shaped.

Ans:B

The nurse is admitting an oncology patient to the unit prior to surgery. The nurse reads in the electronic health record that the patient has just finished radiation therapy. With knowledge of the consequent health risks, the nurse should prioritize assessments related to what health problem? A) Cognitive deficits B) Impaired wound healing C) Cardiac tamponade D) Tumor lysis syndrome

Ans:B

The nurse is caring for a patient with an advanced stage of breast cancer and the patient has recently learned that her cancer has metastasized. The nurse enters the room and finds the patient struggling to breath and the nurse's rapid assessment reveals that the patient's jugular veins are distended. The nurse should suspect the development of what oncologic emergency? A) Increased intracranial pressure B) Superior vena cava syndrome (SVCS) C) Spinal cord compression D) Metastatic tumor of the neck

Ans:B

The public health nurse is presenting a health-promotion class to a group at a local community center. Which intervention most directly addresses the leading cause of cancer deaths in North America? A) Monthly self-breast exams B) Smoking cessation C) Annual colonoscopies D) Monthly testicular exams

Ans:B

A 58-year-old male patient has been hospitalized for a wedge resection of the left lower lung lobe after a routine chest x-ray shows carcinoma. The patient is anxious and asks if he can smoke. Which statement by the nurse would be most therapeutic? A) Smoking is the reason you are here. B) The doctor left orders for you not to smoke. C) You are anxious about the surgery. Do you see smoking as helping? D) Smoking is OK right now, but after your surgery it is contraindicated.

Ans:C

A public health nurse has formed an interdisciplinary team that is developing an educational program entitled Cancer: The Risks and What You Can Do About Them. Participants will receive information, but the major focus will be screening for relevant cancers. This program is an example of what type of health promotion activity? A) Disease prophylaxis B) Risk reduction C) Secondary prevention D) Tertiary prevention

Ans:C

An oncology patient has just returned from the postanesthesia care unit after an open hemicolectomy. This patient's plan of nursing care should prioritize which of the following? A) Assess the patient hourly for signs of compartment syndrome. B) Assess the patient's fine motor skills once per shift. C) Assess the patient's wound for dehiscence every 4 hours. D) Maintain the patient's head of bed at 45 degrees or more at all times.

Ans:C

The hospice nurse is caring for a patient with cancer in her home. The nurse has explained to the patient and the family that the patient is at risk for hypercalcemia and has educated them on that signs and symptoms of this health problem. What else should the nurse teach this patient and family to do to reduce the patient's risk of hypercalcemia? A) Stool softeners are contraindicated. B) Laxatives should be taken daily C) Consume 2-4L of fluid daily D) Restrict calcium intake

Ans:C

The nurse is caring for a 39-year-old woman with a family history of breast cancer. She requested a breast tumor marking test and the results have come back positive. As a result, the patient is requesting a bilateral mastectomy. This surgery is an example of what type of oncologic surgery? A) Salvage surgery B) Palliative surgery C) Prophylactic surgery D) Reconstructive surgery

Ans:C

The nurse on a bone marrow transplant unit is caring for a patient with cancer who is preparing for HSCT. What is a priority nursing diagnosis for this patient? A) Fatigue related to altered metabolic processes B) Altered nutrition: less than body requirements related to anorexia C) Risk for infection related to altered immunologic response D) Body image disturbance related to weight loss and anorexia

Ans:C

The school nurse is teaching a nutrition class in the local high school. One student states that he has heard that certain foods can increase the incidence of cancer. The nurse responds, research has shown that certain foods indeed appear to increase the risk of cancer. Which of the following menu selections would be the best choice for potentially reducing the risks of cancer? A) Smoked salmon and green beans B) Pork chops and fried green tomatoes C) Baked apricot chicken and steamed broccoli D) Liver, onions, and steamed peas

Ans:C

Traditionally, nurses have been involved with tertiary cancer prevention. However, an increasing emphasis is being placed on both primary and secondary prevention. What would be an example of primary prevention? A) Yearly Pap tests B) Testicular self-examination C) Teaching patients to wear sunscreen D) Screening mammograms

Ans:C

The nurse is performing an initial assessment of an older adult resident who has just relocated to the long-term care facility. During the nurse's interview with the patient, she admits that she drinks around 20 ounces of vodka every evening. What types of cancer does this put her at risk for? Select all that apply. A) Malignant melanoma B) Brain cancer C) Breast cancer D) Esophageal cancer E) Liver cancer

Ans:C, D, E

A 62-year-old woman diagnosed with breast cancer is scheduled for a partial mastectomy. The oncology nurse explained that the surgeon will want to take tissue samples to ensure the disease has not spread to adjacent axillary lymph nodes. The patient has asked if she will have her lymph nodes dissected, like her mother did several years ago. What alternative to lymph node dissection will this patient most likely undergo? A) Lymphadenectomy B) Needle biopsy C) Open biopsy D) Sentinel node biopsy

Ans:D

A nurse provides care on a bone marrow transplant unit and is preparing a female patient for a hematopoietic stem cell transplantation (HSCT) the following day. What information should the nurse emphasize to the patient's family and friends? A) Your family should likely gather at the bedside in case there's a negative outcome. B) Make sure she doesn't eat any food in the 24 hours before the procedure. C) Wear a hospital gown when you go into the patient's room. D) Do not visit if you've had a recent infection.

Ans:D

An oncology nurse is caring for a patient who has developed erythema following radiation therapy. What should the nurse instruct the patient to do? A) Periodically apply ice to the area. B) Keep the area cleanly shaven. C) Apply petroleum jelly to the affected area. D) Avoid using soap on the treatment area.

Ans:D

An oncology nurse is contributing to the care of a patient who has failed to respond appreciably to conventional cancer treatments. As a result, the care team is considering the possible use of biologic response modifiers (BRFs). The nurse should know that these achieve a therapeutic effect by what means? A) Promoting the synthesis and release of leukocytes B) Focusing the patient's immune system exclusively on the tumor C) Potentiating the effects of chemotherapeutic agents and radiation therapy D) Altering the immunologic relationship between the tumor and the patient

Ans:D

An oncology patient has begun to experience skin reactions to radiation therapy, prompting the nurse to make the diagnosis Impaired Skin Integrity: erythematous reaction to radiation therapy. What intervention best addresses this nursing diagnosis? A) Apply an ice pack or heating pad PRN to relieve pain and pruritis B) Avoid skin contact with water whenever possible C) Apply phototherapy PRN D) Avoid rubbing or scratching the affected area

Ans:D

An oncology patient will begin a course of chemotherapy and radiation therapy for the treatment of bone metastases. What is one means by which malignant disease processes transfer cells from one place to another? A) Adhering to primary tumor cells B) Inducing mutation of cells of another organ C) Phagocytizing healthy cells D) Invading healthy host tissues

Ans:D

The nurse is caring for a patient has just been given a 6-month prognosis following a diagnosis of extensive stage small-cell lung cancer. The patient states that he would like to die at home, but the team believes that the patient's care needs are unable to be met in a home environment. What might you suggest as an alternative? A) Discuss a referral for rehabilitation hospital. B) Panel the patient for a personal care home. C) Discuss a referral for acute care. D) Discuss a referral for hospice care.

Ans:D

The nurse is orienting a new nurse to the oncology unit. When reviewing the safe administration of antineoplastic agents, what action should the nurse emphasize? A) Adjust the dose to the patient's present symptoms. B) Wash hands with an alcohol-based cleanser following administration. C) Use gloves and a lab coat when preparing the medication. D) Dispose of the antineoplastic wastes in the hazardous waste receptacle.

Ans:D

The nurse should recognize a patient's risk for impaired immune function if the patient has undergone surgical removal of which of the following? A) Thyroid gland B) Spleen C) Kidney D) Pancreas

B (Feedback: A history of surgical removal of the spleen, lymph nodes, or thymus may place the patient at risk for impaired immune function. Removal of the thyroid, kidney, or pancreas would not directly lead to impairment of the immune system.)

A nurse has administered a child's scheduled vaccination for rubella. This vaccination will cause the child to develop which of the following? A) Natural immunity B) Active acquired immunity C) Cellular immunity D) Mild hypersensitivity

B (Feedback: Active acquired immunity usually develops as a result of vaccination or contracting a disease. Natural immunity is present at birth and provides a nonspecific response to any foreign invader. Immunizations do not activate the process of cellular immunity. Hypersensitivity is not an expected outcome of immunization.)

A nurse is explaining the process by which the body removes cells from circulation after they have performed their physiologic function. The nurse is describing what process? A) The cellular immune response B) Apoptosis C) Phagocytosis D) Opsonization

B (Feedback: Apoptosis, or programmed cell death, is the body's way of destroying worn out cells such as blood or skin cells or cells that need to be renewed. Opsonization is the coating of antigen-antibody molecules with a sticky substance to facilitate phagocytosis. The body does not use phagocytosis or the cellular immune response to remove cells from circulation.)

A patient is undergoing testing to determine the overall function of her immune system. What test can be performed to evaluate the functioning of the patient's cellular immune system? A) Immunoglobulin testing B) Delayed hypersensitivity skin test C) Specific antibody response D) Total serum globulin assessment

B (Feedback: Cellular (cell-mediated) immunity tests include the delayed hypersensitivity skin test, since this immune response is specifically dependent on the cellular immune response. Each of the other listed tests assesses functioning of the humoral immune system.)

A patient was recently exposed to infectious microorganisms and many T lymphocytes are now differentiating into killer T cells. This process characterizes what stage of the immune response? A) Effector B) Proliferation C) Response D) Recognition

B (Feedback: In the proliferation stage, T lymphocytes differentiate into cytotoxic (or killer) T cells, whereas B lymphocytes produce and release antibodies. This does not occur in the response, recognition, or effector stages.)

A 16-year-old has been brought to the emergency department by his parents after falling through the glass of a patio door, suffering a laceration. The nurse caring for this patient knows that the site of the injury will have an invasion of what? A) Interferons B) Phagocytic cells C) Apoptosis D) Cytokines

B (Feedback: Monocytes migrate to injury sites and function as phagocytic cells, engulfing, ingesting, and destroying greater numbers and quantities of foreign bodies or toxins than granulocytes. This occurs in response to the foreign bodies that have invaded the laceration from the dirt on the broken glass. Interferon, one type of biologic response modifier, is a nonspecific viricidal protein that is naturally produced by the body and is capable of activating other components of the immune system. Apoptosis, or programmed cell death, is the body's way of destroying worn out cells such as blood or skin cells or cells that need to be renewed. Cytokines are the various proteins that mediate the immune response. These do not migrate to injury sites.)

The nurse is assessing a client's risk for impaired immune function. What assessment finding should the nurse identify as a risk factor for decreased immunity? A) The patient takes a beta blocker for the treatment of hypertension. B) The patient is under significant psychosocial stress. C) The patient had a pulmonary embolism 18 months ago. D) The patient has a family history of breast cancer.

B (Feedback: Stress is a psychoneuroimmunologic factor that is known to depress the immune response. Use of beta blockers, a family history of cancer, and a prior PE are significant assessment findings, but none represents an immediate threat to immune function.)

A nurse is explaining how the humoral and cellular immune responses should be seen as interacting parts of the broader immune system rather than as independent and unrelated processes. What aspect of immune function best demonstrates this? A) The movement of B cells in and out of lymph nodes B) The interactions that occur between T cells and B cells C) The differentiation between different types of T cells D) The universal role of the complement system

B (Feedback: T cells interact closely with B cells, indicating that humoral and cellular immune responses are not separate, unrelated processes, but rather branches of the immune response that interact. Movement of B cells does not clearly show the presence of a unified immune system. The differentiation between types of T cells and the role of the complement system do not directly suggest a single immune system.)

A man was scratched by an old tool and developed a virulent staphylococcus infection. In the course of the man's immune response, circulating lymphocytes containing the antigenic message returned to the nearest lymph node. During what stage of the immune response did this occur? A) Recognition stage B) Proliferation stage C) Response stage D) Effector stage

B (Feedback: The recognition stage of antigens as foreign by the immune system is the initiating event in any immune response. The body must first recognize invaders as foreign before it can react to them. In the proliferation stage, the circulating lymphocyte containing the antigenic message returns to the nearest lymph node. Once in the node, the sensitized lymphocyte stimulates some of the resident dormant T and B lymphocytes to enlarge, divide, and proliferate. In the response stage, the differentiated lymphocytes function either in a humoral or a cellular capacity. In the effector stage, either the antibody of the humoral response or the cytotoxic (killer) T cell of the cellular response reaches and connects with the antigen on the surface of the foreign invader.)

A patient is admitted with cellulitis and experiences a consequent increase in white blood cell count. The nurse is aware that during the immune response, pathogens are engulfed by white blood cells that ingest foreign particles. What is this process known as? A) Apoptosis B) Phagocytosis C) Antibody response D) Cellular immune response

B (Feedback: During the first mechanism of defense, white blood cells, which have the ability to ingest foreign particles, move to the point of attack, where they engulf and destroy the invading agents. This is known as phagocytosis. The action described is not apoptosis (programmed cell death) or an antibody response. Phagocytosis occurs in the context of the cellular immune response, but it does not constitute the entire cellular response.)

A patient is 2 hours postoperative with a Foley catheter in situ. The last hourly urine output recorded for this patient was 10 mL. The tubing of the Foley is patent. What should the nurse do? A) Irrigate the Foley with 30 mL normal saline. B) Notify the physician and continue to monitor the hourly urine output closely. C) Decrease the IV fluid rate and massage the patients abdomen. D) Have the patient sit in high-Fowlers position.

B) Notify the physician and continue to monitor the hourly urine output closely.

The nurse is creating the plan of care for a patient who is status postsurgery for reduction of a femur fracture. What is the most important short-term goal for this patient? A) Relief of pain B) Adequate respiratory function C) Resumption of activities of daily living (ADLs) D) Unimpaired wound healing

B) Adequate respiratory function

The nurse is preparing to change a patients abdominal dressing. The nurse recognizes the first step is to provide the patient with information regarding the procedure. Which of the following explanations should the nurse provide to the patient? A) The dressing change is often painful, and we will be giving you pain medication prior to the procedure so you do not have to worry. B) During the dressing change, I will provide privacy at a time of your choosing, it should not be painful, and you can look at the incision and help with the procedure if you want to. C) The dressing change should not be painful, but you can never be sure, and infection is always a concern. D) The best time for doing a dressing change is during lunch so we are not interrupted. I will provide privacy, and it should not be painful.

B) During the dressing change, I will provide privacy at a time of your choosing, it should not be painful, and you can look at the incision and help with the procedure if you want to.

The surgeons preoperative assessment of a patient has identified that the patient is at a high risk for venous thromboembolism. Once the patient is admitted to the postsurgical unit, what intervention should the nurse prioritize to reduce the patients risk of developing this complication? A) Maintain the head of the bed at 45 degrees or higher. B) Encourage early ambulation. C) Encourage oral fluid intake. D) Perform passive range-of-motion exercises every 8 hours.

B) Encourage early ambulation.

The nurse admits a patient to the PACU with a blood pressure of 132/90 mm Hg and a pulse of 68 beats per minute. After 30 minutes, the patients blood pressure is 94/47 mm Hg, and the pulse is 110. The nurse documents that the patients skin is cold, moist, and pale. Of what is the patient showing signs? A) Hypothermia B) Hypovolemic shock C) Neurogenic shock D) Malignant hyperthermia

B) Hypovolemic shock

The nurse is caring for a patient who has just been transferred to the PACU from the OR. What is the highest nursing priority? A) Assessing for hemorrhage B) Maintaining a patent airway C) Managing the patients pain D) Assessing vital signs every 30 minutes

B) Maintaining a patent airway

The nursing instructor is discussing the difference between ambulatory surgical centers and hospital- based surgical units. A student asks why some patients have surgery in the hospital and others are sent to ambulatory surgery centers. What is the instructors best response? A) Patients who go to ambulatory surgery centers are more independent than patients admitted to the hospital. B) Patients admitted to the hospital for surgery usually have multiple health needs. C) In most cases, only emergency and trauma patients are admitted to the hospital. D) Patients who have surgery in the hospital are those who need to have anesthesia administered.

B) Patients admitted to the hospital for surgery usually have multiple health needs.

The nurse is caring for a patient who is postoperative day 2 following a colon resection. While turning him, wound dehiscence with evisceration occurs. What should be the nurses first response? A) Return the patient to his previous position and call the physician. B) Place saline-soaked sterile dressings on the wound. C) Assess the patients blood pressure and pulse. D) Pull the dehiscence closed using gloved hands.

B) Place saline-soaked sterile dressings on the wound.

A postoperative patient rapidly presents with hypotension; rapid, thready pulse; oliguria; and cold, pale skin. The nurse suspects that the patient is experiencing a hemorrhage. What should be the nurses first action? A) Leave and promptly notify the physician. B) Quickly attempt to determine the cause of hemorrhage. C) Begin resuscitation. D) Put the patient in the Trendelenberg position.

B) Quickly attempt to determine the cause of hemorrhage.

The nursing instructor is discussing postoperative care with a group of nursing students. A student nurse asks, Why does the patient go to the PACU instead of just going straight up to the postsurgical unit? What is the nursing instructors best response? A) The PACU allows the patient to recover from anesthesia in a stimulating environment to facilitate awakening and reorientation. B) The PACU allows the patient to recover from the effects of anesthesia, and the patient stays in the PACU until he or she is oriented, has stable vital signs, and is without complications. C) Frequently, patients are placed in the medicalsurgical unit to recover, but hospitals are usually short of beds, and the PACU is an excellent place to triage patients. D) Patients remain in the PACU for a predetermined time because the surgeon will often need to reinforce or alter the patients incision in the hours following surgery.

B) The PACU allows the patient to recover from the effects of anesthesia, and the patient stays in the PACU until he or she is oriented, has stable vital signs, and is without complications.

A patient underwent an open bowel resection 2 days ago and the nurses most recent assessment of the patients abdominal incision reveals that it is dehiscing. What factor should the nurse suspect may have caused the dehiscence? A) The patients surgical dressing was changed yesterday and today. B) The patient has vomited three times in the past 12 hours. C) The patient has begun voiding on the commode instead of a bedpan. D) The patient used PCA until this morning.

B) The patient has vomited three times in the past 12 hours.

The PACU nurse is caring for a patient who has arrived from the OR. During the initial assessment, the nurse observes that the patients skin has become blue and dusky. The nurse looks, listens, and feels for breathing, and determines the patient is not breathing. What is the priority intervention? A) Check the patients oxygen saturation level, continue to monitor for apnea, and perform a focused assessment. B) Treat the possible airway obstruction by tilting the head back and pushing forward on the angle of the lower jaw. C) Assess the arterial pulses, and place the patient in the Trendelenburg position. D) Reintubate the patient.

B) Treat the possible airway obstruction by tilting the head back and pushing forward on the angle of the lower jaw

The nurse is caring for a postoperative patient who needs daily dressing changes. The patient is 3 days postoperative and is scheduled for discharge the next day. Until now, the patient has refused to learn how to change her dressing. What would indicate to the nurse the patients possible readiness to learn how to change her dressing? Select all that apply. A) The patient wants you to teach a family member to do dressing changes. B) The patient expresses interest in the dressing change. C) The patient is willing to look at the incision during a dressing change. D) The patient expresses dislike of the surgical wound. E) The patient assists in opening the packages of dressing material for the nurse.

B, C, E

A surgical patient has been in the PACU for the past 3 hours. What are the determining factors for the patient to be discharged from the PACU? Select all that apply. A) Absence of pain B) Stable blood pressure C) Ability to tolerate oral fluids D) Sufficient oxygen saturation E) Adequate respiratory function

B, D, E

Which should a nurse thoroughly evaluate before a bone marrow transplant (BMT) procedure?

Blood studies

A patient's recent diagnostic testing included a total lymphocyte count. The results of this test will allow the care team to gauge what aspect of the patient's immunity? A) Humoral immune function B) Antigen recognition C) Cell-mediated immune function D) Antibody production

C (Feedback: A total lymphocyte count is a test used to determine cellular immune function. It is not normally used for testing humoral immune function and the associated antigen-antibody.)

A gerontologic nurse is caring for an older adult patient who has a diagnosis of pneumonia. What age-related change increases older adults' susceptibility to respiratory infections? A) Atrophy of the thymus B) Bronchial stenosis C) Impaired ciliary action D) Decreased diaphragmatic muscle tone

C (Feedback: As a consequence of impaired ciliary action due to exposure to smoke and environmental toxins, older adults are vulnerable to lung infections. This vulnerability is not the result of thymus atrophy, stenosis of the bronchi, or loss of diaphragmatic muscle tone.)

The nurse knows that the response of natural immunity is enhanced by processes that are inherent in the physical and chemical barriers of the body. What is a chemical barrier that enhances the response of natural immunity? A) Cell cytoplasm B) Interstitial fluid C) Gastric secretions D) Cerebrospinal fluid

C (Feedback: Chemical barriers, such as mucus, acidic gastric secretions, enzymes in tears and saliva, and substances in sebaceous and sweat secretions, act in a nonspecific way to destroy invading bacteria and fungi. Not all body fluids are chemical barriers, however. Cell cytoplasm, interstitial fluid, and CSF are not normally categorized as chemical barriers to infection.)

A nurse is reviewing a patient's medication administration record in an effort to identify drugs that may contribute to the patient's recent immunosuppression. What drug is most likely to have this effect? A) An antibiotic B) A nonsteroidal anti-inflammatory drug (NSAID) C) An antineoplastic D) An antiretroviral

C (Feedback: Chemotherapy affects bone marrow function, destroying cells that contribute to an effective immune response and resulting in immunosuppression. Antibiotics in large doses cause bone marrow suppression, but antineoplastic drugs have the most pronounced immunosuppressive effect. NSAIDs and antiretrovirals do not normally have this effect.)

Diagnostic testing has revealed a deficiency in the function of a patient's complement system. This patient is likely to have an impaired ability to do which of the following? A) Protecting the body against viral infection B) Marking the parameters of the immune response C) Bridging natural and acquired immunity D) Collecting immune complexes during inflammation

C (Feedback: Complement has three major physiologic functions: defending the body against bacterial infection, bridging natural and acquired immunity, and disposing of immune complexes and the byproducts associated with inflammation. Complement does not mark the parameters of the immune response; complement does not collect immune complexes during inflammation.)

A patient's current immune response involves the direct destruction of foreign microorganisms. This aspect of the immune response may be performed by what cells? A) Suppressor T cells B) Memory T cells C) Cytotoxic T cells D) Complement T cells

C (Feedback: Cytotoxic T cells (also called CD8 + cells) participate in the destruction of foreign organisms. Memory T cells and suppressor T cells do not perform this role in the immune response. The complement system does not exist as a type of T cell.)

A nurse is admitting a patient who exhibits signs and symptoms of a nutritional deficit. Inadequate intake of what nutrient increases a patient's susceptibility to infection? A) Vitamin B12 B) Unsaturated fats C) Proteins D) Complex carbohydrates

C (Feedback: Depletion of protein reserves results in atrophy of lymphoid tissues, depression of antibody response, reduction in the number of circulating T cells, and impaired phagocytic function. As a result, the patient has an increased susceptibility to infection. Low intake of fat and vitamin B12 affects health, but is not noted to directly create a risk for infection. Low intake of complex carbohydrates is not noted to constitute a direct risk factor for infection.)

A patient is responding to a microbial invasion and the patient's differentiated lymphocytes have begun to function in either a humoral or a cellular capacity. During what stage of the immune response does this occur? A) The recognition stage B) The effector stage C) The response stage D) The proliferation stage

C (Feedback: In the response stage, the differentiated lymphocytes function in either a humoral or a cellular capacity. In the effector stage, either the antibody of the humoral response or the cytotoxic (killer) T cell of the cellular response reaches and connects with the antigen on the surface of the foreign invader. In the recognition stage, the recognition of antigens as foreign, or non-self, by the immune system is the initiating event in any immune response. During the proliferation stage the circulating lymphocytes containing the antigenic message return to the nearest lymph node.)

A nurse is planning the assessment of a patient who is exhibiting signs and symptoms of an autoimmune disorder. The nurse should be aware that the incidence and prevalence of autoimmune diseases is known to be higher among what group? A) Young adults B) Native Americans C) Women D) Hispanics

C (Feedback: Many autoimmune diseases have a higher incidence in females than in males, a phenomenon believed to be correlated with sex hormones.)

A patient with cystic fibrosis has received a double lung transplant and is now experiencing signs of rejection. What is the immune response that predominates in this situation? A) Humoral B) Nonspecific C) Cellular D) Mitigated

C (Feedback: Most immune responses to antigens involve both humoral and cellular responses, although only one predominates. During transplantation rejection, the cellular response predominates over the humoral response. Neither a mitigated nor nonspecific cell response is noted in this situation.)

A nurse is caring for a patient who has had a severe antigen/antibody reaction. The nurse knows that the portion of the antigen that is involved in binding with the antibody is called what? A) Antibody lock B) Antigenic sequence C) Antigenic determinant D) Antibody channel

C (Feedback: The portion of the antigen involved in binding with the antibody is referred to as the antigenic determinant. This portion is not known as an antibody lock, antigenic sequence, or antibody channel.)

A patient requires ongoing treatment and infection-control precautions because of an inherited deficit in immune function. The nurse should recognize that this patient most likely has what type of immune disorder? A) A primary immune deficiency B) A gammopathy C) An autoimmune disorder D) A rheumatic disorder

C (Feedback: Primary immune deficiency results from improper development of immune cells or tissues. These disorders are usually congenital or inherited. Autoimmune disorders are less likely to have a genetic component, though some have a genetic component. Overproduction of immunoglobulins is the hallmark of gammopathies. Rheumatic disorders do not normally involve impaired immune function)

The nurse just received a postoperative patient from the PACU to the medicalsurgical unit. The patient is an 84-year-old woman who had surgery for a left hip replacement. Which of the following concerns should the nurse prioritize for this patient in the first few hours on the unit? A) Beginning early ambulation B) Maintaining clean dressings on the surgical site C) Close monitoring of neurologic status D) Resumption of normal oral intake

C) Close monitoring of neurologic status

The home health nurse is caring for a postoperative patient who was discharged home on day 2 after surgery. The nurse is performing the initial visit on the patients postoperatative day 2. During the visit, the nurse will assess for wound infection. For most patients, what is the earliest postoperative day that a wound infection becomes evident? A) Day 9 B) Day 7 C) Day 5 D) Day 3

C) Day 5

You are caring for a 71-year-old patient who is 4 days postoperative for bilateral inguinal hernias. The patient has a history of congestive heart failure and peptic ulcer disease. The patient is highly reluctant to ambulate and will not drink fluids except for hot tea with her meals. The nurses aide reports to you that this patients vital signs are slightly elevated and that she has a nonproductive cough. When you assess the patient, you auscultate crackles at the base of the lungs. What would you suspect is wrong with your patient? A) Pulmonary embolism B) Hypervolemia C) Hypostatic pulmonary congestion D) Malignant hyperthermia

C) Hypostatic pulmonary congestion

The nurse is assessing a new client with complaints of acute fatigue and a sore tongue that is visibly smooth and beefy red. This client is demonstrating signs and symptoms associated with what form of hematologic disorder? A) Sickle cell disease B) Hemophilia C) Megaloblastic anemia D) Thrombocytopenia

C) Megaloblastic anemia

The nurse is caring for an 82-year-old female patient in the PACU. The woman begins to awaken and responds to her name, but is confused, restless, and agitated. What principle should guide the nurses subsequent assessment? A) Postoperative confusion in older adults is an indication of impaired oxygenation or possibly a stroke during surgery. B) Confusion, restlessness, and agitation are expected postoperative findings in older adults and they will diminish in time. C) Postoperative confusion is common in the older adult patent, but it could also indicate a significant blood loss. D) Confusion, restlessness, and agitation indicate an underlying cognitive deficit such as dementia.

C) Postoperative confusion is common in the older adult patent, but it could also indicate a significant blood loss.

A young man with a diagnosis of hemophilia A has been brought to emergency department after suffering a workplace accident resulting in bleeding. Rapid assessment has revealed the source of the client's bleeding and established that his vital signs are stable. What should be the nurse's next action? A) Position the client in a prone position to minimize bleeding B) Establish IV access for the administration of vitamin K C) Prepare for the administration of factor VIII D) Administer a normal saline bolus to increase circulatory volume

C) Prepare for the administration of factor VIII

The nurse is caring for a 79-year-old man who has returned to the postsurgical unit following abdominal surgery. The patient is unable to ambulate and is now refusing to wear an external pneumatic compression stocking. The nurse should explain that refusing to wear external pneumatic compression stockings increases his risk of what postsurgical complication? A) Sepsis B) Infection C) Pulmonary embolism D) Hematoma

C) Pulmonary embolism

The nurse is caring for a patient on the medicalsurgical unit postoperative day 5. During each patient assessment, the nurse evaluates the patient for infection. Which of the following would be most indicative of infection? A) Presence of an indwelling urinary catheter B) Rectal temperature of 99.5F (37.5C) C) Red, warm, tender incision D) White blood cell (WBC) count of 8,000/mL

C) Red, warm, tender incision

The nurse is caring for a 78-year-old man who has had an outpatient cholecystectomy. The nurse is getting him up for his first walk postoperatively. To decrease the potential for orthostatic hypotension and consequent falls, what should the nurse have the patient do? A) Sit in a chair for 10 minutes prior to ambulating. B) Drink plenty of fluids to increase circulating blood volume. C) Stand upright for 2 to 3 minutes prior to ambulating. D) Perform range-of-motion exercises for each joint.

C) Stand upright for 2 to 3 minutes prior to ambulating.

The nurse is caring for a patient after abdominal surgery in the PACU. The patients blood pressure has increased and the patient is restless. The patients oxygen saturation is 97%. What cause for this change in status should the nurse first suspect? A) The patient is hypothermic. B) The patient is in shock. C) The patient is in pain. D) The patient is hypoxic.

C) The patient is in pain.

The nurse in the ED is caring for a man who has returned to the ED 4 days after receiving stitches for a knife wound on his hand. The wound is now infected, so the stitches were removed, and the wound is cleaned and packed with gauze. The ED doctor plans to have the man return tomorrow to remove the packing and resuture the wound. You are aware that the wound will now heal by what means? A) Late intention B) Second intention C) Third intention D) First intention

C) Third intention

The dressing surrounding a mastectomy patients Jackson-Pratt drain has scant drainage on it. The nurse believes that the amount of drainage on the dressing may be increasing. How can the nurse best confirm this suspicion? A) Describe the appearance of the dressing in the electronic health record. B) Photograph the patients abdomen for later comparison using a smartphone. C) Trace the outline of the drainage on the dressing for future comparison. D) Remove and weigh the dressing, reapply it, and then repeat in 8 hours.

C) Trace the outline of the drainage on the dressing for future comparison.

A client undergoes a biopsy of a suspicious lesion. The biopsy report classifies the lesion according to the TNM staging system as follows: Tis, N0, M0. What does this classification mean?

Carcinoma in situ, no abnormal regional lymph nodes, and no evidence of distant metastasis

Client with chronic alcoholismThe nurse recognizes that the client most at risk for mortality associated with surgery is the

Client with chronic alcoholism

The nurse is completing a focused assessment addressing a patient's immune function. What should the nurse prioritize in the physical assessment? A) Percussion of the patient's abdomen B) Palpation of the patient's liver C) Auscultation of the patient's apical heart rate D) Palpation of the patient's lymph nodes

D (Feedback: During the assessment of immune function, the anterior and posterior cervical, supraclavicular, axillary, and inguinal lymph nodes are palpated for enlargement. If palpable nodes are detected, their location, size, consistency, and reports of tenderness on palpation are noted. Because of the central role of lymph nodes in the immune system, they are prioritized over the heart, liver, and abdomen, even though these would be assessed.)

A nurse is planning a patient's care and is relating it to normal immune response. During what stage of the immune response should the nurse know that antibodies or cytotoxic T cells combine and destroy the invading microbes? A) Recognition stage B) Proliferation stage C) Response stage D) Effector stage

D (Feedback: In the effector stage, either the antibody of the humoral response or the cytotoxic (killer) T cell of the cellular response reaches and couples with the antigen on the surface of the foreign invader. The coupling initiates a series of events that in most instances results in total destruction of the invading microbes or the complete neutralization of the toxin. This does not take place during the three preceding stages.)

A nurse has admitted a patient who has been diagnosed with urosepsis. What immune response predominates in sepsis? A) Mitigated B) Nonspecific C) Cellular D) Humoral

D (Feedback: Most immune responses to antigens involve both humoral and cellular responses, although only one predominates. For example, during transplantation rejection, the cellular response predominates, whereas in the bacterial pneumonias and sepsis, the humoral response plays the dominant role. Neither mitigated nor nonspecific cell response is noted in this situation.)

A neonate exhibited some preliminary signs of infection, but the infant's condition resolved spontaneously prior to discharge home from the hospital. This infant's recovery was most likely due to what type of immunity? A) Cytokine immunity B) Specific immunity C) Active acquired immunity D) Nonspecific immunity

D (Feedback: Natural immunity, or nonspecific immunity, is present at birth. Active acquired or specific immunity develops after birth. Cytokines are proteins that mediate the immune response; they are not a type of immunity.)

A patient's injury has initiated an immune response that involves inflammation. What are the first cells to arrive at a site of inflammation? A) Eosinophils B) Red blood cells C) Lymphocytes D) Neutrophils

D (Feedback: Neutrophils are the first cells to arrive at the site where inflammation occurs. Eosinophils increase in number during allergic reactions and stress responses, but are not always present during inflammation. RBCs do not migrate during an immune response. Lymphocytes become active but do not migrate to the site of inflammation.)

A nursing student is giving a report on the immune system. What function of cytokines should the student describe? A) Determining whether a cell is foreign B) Determining if lymphokines will be activated C) Determining whether the T cells will remain in the nodes and retain a memory of the antigen D) Determining whether the immune response will be the production of antibodies or a cell-mediated response

D (Feedback: Separate subpopulations of helper T cells produce different types of cytokines and determine whether the immune response will be the production of antibodies or a cell-mediated immune response. Cytokines do not determine whether cells are foreign, determine if lymphokines will be activated, or determine the role of memory T cells.)

A nurse is reviewing the immune system before planning an immunocompromised patient's care. How should the nurse characterize the humoral immune response? A) Specialized cells recognize and ingest cells that are recognized as foreign. B) T lymphocytes are assisted by cytokines to fight infection. C) Lymphocytesare stimulated to become cells that attack microbes directly. D) Antibodies are made by B lymphocytes in response to a specific antigen.

D (Feedback: The humoral response is characterized by the production of antibodies by B lymphocytes in response to a specific antigen. Phagocytosis and direct attack on microbes occur in the context of the cellular immune response.)

A patient is being treated for bacterial pneumonia. In the first stages of illness, the patient's dyspnea was accompanied by a high fever. Currently, the patient claims to be feeling better and is afebrile. The patient is most likely in which stage of the immune response? A) Recognition stage B) Proliferation stage C) Response stage D) Effector stage

D (Feedback: The immune response culminates with the effector stage, during which offending microorganisms are killed by the various actions of the immune system. The patient's improvement in health status is likely the result of this final stage in the immune response.)

A patient is being treated for cancer and the nurse has identified the nursing diagnosis of Risk for Infection Due to Protein Losses. Protein losses inhibit immune response in which of the following ways? A) Causing apoptosis of cytokines B) Increasing interferon production C) Causing CD4+ cells to mutate D) Depressing antibody response

D (Feedback: Depletion of protein reserves results in atrophy of lymphoid tissues, depression of antibody response, reduction in the number of circulating T cells, and impaired phagocytic function. This specific nutritional deficit does not cause T-cell mutation, an increase in the production of interferons, or apoptosis of cytokines.)

Two units of PRBCs have been prescribed for a client who has experienced a GI bleed. The client is highly reluctant to receive a transfusion, stating, "I'm terrified of getting AIDS from a blood transfusion." How can the nurse best address the client's concerns? A) "All donated blood is treated with antiretroviral medications before it is used." B) "That did happen in some high-profile cases in the 20th century, but it is no longer a possibility." C) "HIV was eradicated from the blood supply in the early 2000s." D) "The chances of contracting AIDS from a blood transfusion are exceedingly low."

D) "The chances of contracting AIDS from a blood transfusion are exceedingly low."

The recovery room nurse is admitting a patient from the OR following the patients successful splenectomy. What is the first assessment that the nurse should perform on this newly admitted patient? A) Heart rate and rhythm B) Skin integrity C) Core body temperature D) Airway patency

D) Airway patency

The nursing instructor is talking with a group of medicalsurgical students about deep vein thrombosis (DVT). A student asks what factors contribute to the formation of a DVT. What would be the instructors best response? A) There is a genetic link in the formation of deep vein thrombi. B) Hypervolemia is often present in patients who go on to develop deep vein thrombi. C) No known factors contribute to the formation of deep vein thrombi; they just occur. D) Dehydration is a contributory factor to the formation of deep vein thrombi.

D) Dehydration is a contributory factor to the formation of deep vein thrombi.

The nurse is admitting a patient to the medicalsurgical unit from the PACU. What should the nurse do to help the patient clear secretions and help prevent pneumonia? A) Encourage the patient to eat a balanced diet that is high in protein. B) Encourage the patient to limit his activity for the first 72 hours. C) Encourage the patient to take his medications as ordered. D) Encourage the patient to use the incentive spirometer every 2 hours.

D) Encourage the patient to use the incentive spirometer every 2 hours.

The nurses aide notifies the nurse that a patient has decreased oxygen saturation levels. The nurse assesses the patient and finds that he is tachypnic, has crackles on auscultation, and his sputum is frothy and pink. The nurse should suspect what complication? A) Pulmonary embolism B) Atelectasis C) Laryngospasm D) Flash pulmonary edema

D) Flash pulmonary edema

The nurse is providing teaching about tissue repair and wound healing to a patient who has a leg ulcer. Which of the following statements by the patient indicates that teaching has been effective? A) Ill make sure to limit my intake of protein. B) Ill make sure that the bandage is wrapped tightly. C) My foot should feel cool or cold while my legs healing. D) Ill eat plenty of fruits and vegetables.

D) Ill eat plenty of fruits and vegetables.

The PACU nurse is caring for a male patient who had a hernia repair. The patients blood pressure is now 164/92 mm Hg; he has no history of hypertension prior to surgery and his preoperative blood pressure was 112/68 mm Hg. The nurse should assess for what potential causes of hypertension following surgery? A) Dysrhythmias, blood loss, and hyperthermia B) Electrolyte imbalances and neurologic changes C) A parasympathetic reaction and low blood volumes D) Pain, hypoxia, or bladder distention

D) Pain, hypoxia, or bladder distention

An adult patient is in the recovery room following a nephrectomy performed for the treatment of renal cell carcinoma. The patients vital signs and level of consciousness stabilized, but the patient then complains of severe nausea and begins to retch. What should the nurse do next? A) Administer a dose of IV analgesic. B) Apply a cool cloth to the patients forehead. C) Offer the patient a small amount of ice chips. D) Turn the patient completely to one side.

D) Turn the patient completely to one side.

While healthy bone marrow is growing back after chemotherapy in patients with leukemia, the nursing staff recalls that these patients are at greater risk for: a. Infection and bleeding b. Hypertension and headache c. Oliguria and urinary retention d. Dyspnea and wheezing

a. Infection and bleeding

Your patient has recently completed her first round of chemotherapy in the treatment of lung cancer. When reviewing this morning's blood work, what findings would be suggestive of myelosuppression?

Decreased platelets and red blood cells

While administering cisplatin to a client, the nurse assesses swelling at the insertion site. What is the nurse's first action?

Discontinue the intravenous medication.

A client who is receiving chemotherapy for esophageal cancer complains of "feeling sick to my stomach all the time." What is the best suggestion the nurse can make to help alleviate this client's nausea?

Eat low-fat foods.

A nurse is administering daunorubicin through a peripheral I.V. line when the client complains of burning at the insertion site. The nurse notes no blood return from the catheter and redness at the I.V. site. The client is most likely experiencing which complication?

Extravasation

In which instance may a surgeon operate without informed consent?

Emergency situations

What intervention should the nurse provide to reduce the incidence of renal damage when a patient is taking a chemotherapy regimen?

Encourage fluid intake to dilute the urine.

The nurse is caring for a client undergoing an incisional biopsy. Which statement does the nurse understand to be true about an incisional biopsy?

It removes a wedge of tissue for diagnosis.

What foods should the nurse suggest that the patient consume less of in order to reduce nitrate intake because of the possibility of carcinogenic action?

Ham and bacon

A nurse on the surgical team has been assigned the role of scrub nurse. What action by the scrub nurse is appropriate

Handing instruments to the surgeon and assistants

The nurse assesses that the oxygen saturation is 89% in an unconscious patient who was transferred from surgery to the postanesthesia care unit (PACU) 15 minutes ago. Which action should the nurse take first? a. Elevate the patient's head. b. Suction the patient's mouth. c. Increase the oxygen flow rate. d. Perform the jaw-thrust maneuver.

In an unconscious postoperative patient, a likely cause of hypoxemia is airway obstruction by the tongue, and the first action is to clear the airway by maneuvers such as the jaw thrust or chin lift. Increasing the oxygen flow rate and suctioning are not helpful when the airway is obstructed by the tongue. Elevating the patient's head will not be effective in correcting the obstruction but may help with oxygenation after the patient is awake

Chemotherapeutic agents have which effect associated with the renal system?

Increased uric acid excretion

A patient with uterine cancer is being treated with intracavitary radiation. The patient will emit radiation while the implant is in place. The nurse is aware of the precautions necessary for the provider of care and visitors. Which of the following are appropriate guidelines to follow? Select all that apply.

Lead aprons should be worn to buffer the exposure. Family members should stand about 6 feet from the patient. Visitors may stay for 30 minutes or less.

Which action should not be allowed when wearing masks in the operating room

Letting masks hang around the neck

An oncology nurse educator is providing health education to a client who has been diagnosed with skin cancer. The client's wife has asked about the differences between normal cells and cancer cells. What characteristic of a cancer cell should the educator cite?

Malignant cells contain proteins called tumor-associated antigens.

When caring for a patient with alcoholism, when should the nurse assess for symptoms of alcoholic withdrawal?

On the second or third day

The surgical nurse is preparing to send a client from the presurgical area to the OR and is reviewing the client's informed consent form. What are the criteria for legally valid informed consent? Select all that apply.

Signature must be witnessed by a professional staff member. Consent must be freely given. Consent must normally be obtained by a physician.

The scrub nurse is responsible for:

Preparing the sterile instruments for the surgical procedure

Active acquired immunity occurs when a person __________. a. manufactures antibodies in response to an infection or vaccination b. receives immunity from another person c. gains immunity by a vaccine d. is never sick

a. manufactures antibodies in response to an infection or vaccination

During which step of cellular carcinogenesis do cellular changes exhibit increased malignant behavior?

Progression

The nurse at the clinic explains to the patient that the surgeon will be removing a mole on the patient's back that has the potential to develop into cancer. The nurse informs the patient that this is what type of procedure?

Prophylactic

The immunoglobulin associated with idiopathic thrombocytopenic purpura (ITP) is __________. a. IgA b. IgG c. IgM d. IgE

b. IgG

A patient is admitted for an excisional biopsy of a breast lesion. What intervention should the nurse provide for the care of this patient?

Provide time for the patient to discuss her concerns.

The nursing staff of an oncology unit cautions visitors to be free of infections before visiting patients. The rationale for this precaution is because chemotherapy and decreased bone marrow production can cause: a. Hemorrhage b. Neutropenia c. Edema d. Hypovolemia

b. Neutropenia

A scrub nurse is diagnosed with a skin infection to the right forearm. What is the priority action by the nurse?

Report the infection to an immediate supervisor

After being seen in the oncology clinic, a client with severe bone marrow suppression is admitted to the hospital. The client's cancer therapy consisted of radiation and chemotherapy. When developing the care plan for this client, which nursing diagnosis takes priority?

Risk for infection

The nurse is doing preoperative client education with a 61-year-old male client who has a 40 pack-year history of cigarette smoking. The client will undergo an elective bunionectomy at a time that fits his work schedule in a few months. What would be the best instruction to give to this client?

Stop smoking at least a month before the scheduled surgery to enhance pulmonary function and decrease infection

The nursing instructor is discussing the difference between normal cells and cancer cells with the pre-nursing class in pathophysiology. What would the instructor cite as a characteristic of a cancer cell?

The cell membrane of malignant cells contains proteins called tumor-specific antigens.

What should the nurse tell a client who is about to begin chemotherapy and is anxious about hair loss?

The client should consider getting a wig or cap prior to beginning treatment.

The physician is attending to a 72-year-old client with a malignant brain tumor. The physician recommends immediate radiation therapy. What is a reason for the physician's recommendation?

To prevent the formation of new cancer cells

The nurse evaluates teaching as effective when a female client states that she will

Use sunscreen when outdoors.

A parent of a 16-year-old patient asks the nurse, "How could the surgeon operate without my consent?" What is the best response given by the nurse? a)"Your child had life-threatening injuries that required immediate surgery." b)"We obtained consent from your child after your child requested the surgery." c)"The surgical procedure being performed does not require consent." d)"Two doctors decided your child needed the surgery, therefore we did not need to get consent."

a)"Your child had life-threatening injuries that required immediate surgery." Explanation: In an emergency, it may be necessary for the surgeon to operate as a lifesaving measure without the patient's or parent's informed consent. Informed consent must be obtained before any invasive procedure. A minor cannot consent for a surgical procedure. Two doctors' opinions do not overrule the need to obtain informed consent.

A surgical patient has been transferred to the holding area. What nursing intervention(s) promote safe and effective nursing care? Select all that apply. a)Identify the patient using two identifiers. b)Provide oral fluids to the patient. c)Apply grounding devices to the patient. d)Review the medical records. e)Verify the surgical site and mark it appropriately. f)Maintain an aseptic environment.

a)Identify the patient using two identifiers. e)Verify the surgical site and mark it appropriately. d)Review the medical records. Explanation: Identifying the patient, verifying and marking the surgical site, and reviewing the medical records all promote safe and effective care while the patient is in the holding area. Maintaining an aseptic environment and applying grounding devices are part of the intraoperative phase. Oral fluids should not be provided while the patient is in the holding area.

An anxious preoperative surgical patient is encouraged to concentrate on a pleasant experience or restful scene. What cognitive coping strategy would the nurse document as being used? a)Imagery b)Progressive muscular relaxation c)Distraction d)Optimistic self-recitation

a)Imagery Explanation: Imagery has proven effective for anxiety in surgical patients. Optimistic self-recitation is practiced when the patient is encouraged to recite optimistic thoughts such as, "I know all will go well." Distraction is employed when the patient is encouraged to think of an enjoyable story or recite a favorite poem. Progressive muscular relaxation requires contracting and relaxing muscle groups and is a physical coping strategy as opposed to a cognitive strategy.

A patient continuously states, "I know all will go well." What cognitive coping strategy should the nurse document? a)Optimistic self-recitation b)Imagery c)Music therapy d)Distraction

a)Optimistic self-recitation Explanation: When that patient verbalizes this statement, it is an optimistic response. Imagery occurs when the patient concentrates on a pleasant experience or restful scene. Distraction occurs when the patient thinks of an enjoyable story or recites a favorite poem or song. Music therapy would be an incorrect answer. Reset

The nurse has administered the preanesthetic medication. What action should the nurse take next? a)Place the patient on bed rest with the side rails up. b)Review the patient's list of home medications. c)Obtain the patient's signature on the consent form. d)Educate the patient on discharge instructions.

a)Place the patient on bed rest with the side rails up. Explanation: The preanesthetic medication can make the patient lightheaded and dizzy. Safety is a priority. The consent form should be signed before the patient is medicated. Consents signed after the patient is medicated are not legal. Reviewing the home medications and educating the patient should take place before the patient is medicated.

A patient is undergoing thoracic surgery. What priority education should the nurse provide to assist in preventing respiratory complications? a)Splint the incision site using a pillow during deep breathing and coughing exercises. b)Deep breathing and coughing exercises should be completed every 8 hours. c)Deep breathing and coughing exercises may be used as relaxation techniques. d)Pain medication should be taken before completing deep breathing and coughing exercises.

a)Splint the incision site using a pillow during deep breathing and coughing exercises. Explanation: Splinting the incision site will help decrease pain and support the incision. This will increase compliance with the deep breathing and coughing exercises that assist with the prevention of respiratory complications. Pain medication should be taken regularly and not only before deep breathing and coughing exercises. Deep breathing and coughing exercises should be done at least every 2 hours, more frequently if possible. While some patients will find the exercises relaxing, most patients find it painful to complete the exercises.

The nurse is educating a community group regarding types of surgery. A member of the group asks the nurse to describe a type of surgery that is curative. What response by the nurse is true? a)The excision of a tumor b)A face-lift c)A biopsy d)The placement of gastrostomy tube

a)The excision of a tumor Explanation: An example of a curative surgical procedure is the excision of a tumor. A biopsy, a face-lift, and the placement of a gastrostomy tube are not examples of curative surgical procedures.

The nurse explains that a normal hematocrit value is approximately: a. Three times the hemoglobin value b. The same as the hemoglobin value c. Four times lower than the red blood cell count d. Same as the red blood cell count

a. Three times the hemoglobin value

At the end of a shift, a nurse documents the effectiveness of parent teaching concerning the transmission of hemophilia. Which of the following statements by the mother would best indicate an accurate parental perception? a. "Hemophilia is a genetic disorder and I am a carrier, although I do not have the disease." b. "My son developed hemophilia because I had measles while I was pregnant." c. "Since my husband isn't affected by the disease, our daughter will not be a carrier." d. "I know it is not necessary to have my two daughters tested for the disease."

a. "Hemophilia is a genetic disorder and I am a carrier, although I do not have the disease."

MD writes an order for Valium 1 mg via PEG tube daily. Pharmacy dispenses you with 3 mg/mL. How many mL will you administer per dose? a. 0.3 mL/dose b. 0.6 mL/dose c. 9 mL/dose d. 2 mL/dose

a. 0.3 mL/dose

The patient receiving Epogen asks how soon an increase in his red blood cell count will occur. The nurse's best reply is that the initial increase in red blood cells should be seen in: a. 2 days b. 1 week c. 10 days d. 2 weeks

a. 2 days

When the patient with a platelet count of 20,000/mm3 receives 1 unit of platelets, the platelet count should rise to: a. 25,000 to 30,000/mm3 b. 35,000 to 40,000/mm3 c. 45,000 to 50,000/mm3 d. 55,000 to 100,000/mm3

a. 25,000 to 30,000/mm3

The nurse differentiates that passive-acquired immunity means that the antibodies were: a. Acquired from outside the host and instilled in the host. b. Manufactured in response to a disease in the host. c. Innately acquired because of being born a human being. d. Cell-mediated inside the host.

a. Acquired from outside the host and instilled in the host.

The foods that the nurse would include in a nutrition teaching plan for a patient with iron-deficiency anemia are: a. Beans and dried fruit b. Apples and white rice c. Yogurt and cooked carrots d. Yellow squash and tortillas

a. Beans and dried fruit

The symptom that the nurse would recognize as being pertinent to a possible diagnosis of systemic lupus erythematosus (SLE) is: a. Butterfly rash of the face b. Protruding abdomen c. Thinning hair d. Bloody diarrhea

a. Butterfly rash of the face

The nurse points out that the spleen's primary function in the immune process is to: a. Filter microorganisms from the blood. b. Store lymphocytes used to fight infections. c. Produce additional RBCs (red blood cells). d. Stimulate WBC production.

a. Filter microorganisms from the blood

A 35-year-old man is examined in an urgent care clinic. His presenting symptoms suggest polycythemia vera. The laboratory value that would confirm this possible diagnosis is an extremely: a. High hemoglobin level b. Low white cell count c. Low platelet count d. High iron level

a. High hemoglobin level

The nurse gives an example of innate immunity by comparing it with a human's immunity to: a. Hoof-and-mouth disease b. Measles c. Rabies d. Mange

a. Hoof-and-mouth disease

At 10:00 AM, the nurse receives 2 units of blood from the blood bank, for a patients' transfusion. The nurse should: a. Set up 1 unit for the infusion to start at 10:30 AM, and send the other unit back until the first one has run. b. Set up both units to run at the same time for an infusion at 11:00 AM. c. Set up one unit for infusion, and place the other in the refrigerator for the later infusion d. Send both units back and ask for a reissue of 1 unit only.

a. Set up 1 unit for the infusion to start at 10:30 AM, and send the other unit back until the first one has run

Systemic lupus erythematosus is an autoimmune disease in which __________. a. the immune system mounts an immune response against its own proteins b. the immune system causes excessive clot formation throughout the body c. the immune system destroys the body's platelets d. a type I hypersensitivity reaction develops

a. the immune system mounts an immune response against its own proteins

The pain associated with a sickle cell crisis is caused by __________. a. tissue hypoxia b. bone marrow spasms c. fever and dehydration d. cardiomegaly

a. tissue hypoxia

The nurse is caring for a client in the postanesthesia care unit (PACU). The client has the following vital signs: pulse 115, respirations 20, oral temperature 97.2°F, blood pressure 84/50. What should the nurse do first?

assess for bleeding

The rationale for administering injections of vitamin B12 to patients with pernicious anemia is that: a. The patient's body does not normally manufacture enough vitamin B12. b. The patient may lack the intrinsic factor necessary for vitamin B12 absorption. c. Vitamin B12 is found in very small quantities in the patient's body. d. Vitamin B12 is a mineral necessary to aid in the formation of strong bones.

b. The patient may lack the intrinsic factor necessary for vitamin B12 absorption

The nurse is completing a preoperative assessment. The nurse notices the patient is tearful and constantly wringing hands. The patient states, "I'm really nervous about this surgery. Do you think it will be ok?" What is the nurse's best response? a)"What family support do you have after the surgery?" b)"What are your concerns?" c)"You have nothing to worry about; you have the best surgical team." d)"No one has ever died from the procedure you are having."

b)"What are your concerns?" Explanation: Asking the patient about their concerns is an open-ended therapeutic technique. It allows the patient to guide the conversation and address their emotional state. Asking about family support is changing the subject and is nontherapeutic. Discussing the surgical team and the low death rate associated with a procedure is minimizing the patient's feelings and is nontherapeutic.

A patient is scheduled for an invasive procedure. What is the priority documentation needed regarding the procedure? a)The medication reconciliation form b)A signed consent form from the patient c)Prescriptions for postoperative medications d)A health history obtained by the primary physician

b)A signed consent form from the patient Explanation: A signed consent is required and is important for initiating invasive procedures. The nurse should therefore check for the patient's signed consent form. A health history, medication reconciliation, and postoperative prescriptions are good items to have, but are not required documentation before performing an invasive procedure.

A patient refuses to remove her wedding band when preparing for surgery. What is the best action for the nurse to take? a)Remove the ring once the patient is sedated. b)Allow the ring to stay on the patient and cover it with tape. c)Discuss the risk for infection caused by wearing the ring. d)Notify the surgeon to cancel surgery.

b)Allow the ring to stay on the patient and cover it with tape. Explanation: Most facilities will allow a wedding band to remain on the patient during the surgical procedure. The nurse must secure the ring with tape. Although it is appropriate to discuss the risk for infection, the patient has already refused removal of the ring. The surgery should not be canceled and the ring should not be removed without permission.

A patient is scheduled to have a cholecystectomy. Which of the nurse's finding is least likely to contribute to surgical complications? a)Diabetes b)Osteoporosis c)Urinary tract infection d)Pregnancy

b)Osteoporosis Explanation: Osteoporosis is most likely not going to contribute to complications related to a cholecystectomy. Pregnancy decreases maternal reserves. Diabetes increases wound-healing problems and risks for infection. Urinary tract infection decreases the immune system, increasing the chance for infections.

MD writes an order for Colace 50 mg daily via PEG tube. Pharmacy dispenses you with 100 mg/15 mL. How many mL will you administer per dose? a. 50 mL/dose b. 7.5mL/dose c. 15 mL/dose d. 2.5 mL/dose

b. 7.5mL/dose

A registered nurse who is responsible for coordinating and documenting client care in the operating room is a

circulating nurse

When a patient is taking dexamethasone (Decadron) complains of insomnia, the nurse advises the patient to take the drug: a. With milk b. At breakfast c. Dissolved in fruit juice d. At bedtime

b. At breakfast

True or False: Patients who are Rh positive can only receive Rh positive blood, while patients who are Rh negative can only receive blood from donors who are Rh negative. a. True b. False

b. False rationale: Rh neg can only receive Rh neg, but Rh pos can receive Rh pos or Rh neg. This is why O- is the universal donor and AB+ is the universal recipient.

The presence of Reed-Sternberg cells in the lymph nodes is characteristic of __________. a. non-Hodgkin's lymphoma b. Hodgkin's lymphoma c. multiple myeloma d. singular myeloma

b. Hodgkin's lymphoma

The laboratory results for a patient with acute leukemia that should alert the nurse to the fact that the drug protocols are not effective is: a. Decreased prothrombin time. b. Platelet count lower than 50,000/mm3. c. Negative Western blot. d. Neutrophils 50% to 62%.

b. Platelet count lower than 50,000/mm3.

A client was diagnosed with cancer several weeks ago and family members describe him as "utterly distraught." The client has fully withdrawn from social and family contact. What is the nurse's best action? a. reassure the client and the family that these types of responses to cancer are common b. refer the client to the appropriate mental health provider c. educate the client about the mental health benefits of exercise d. reassure the family that the client is grieving and will eventually come to terms with the diagnosis

b. Refer the client to the appropriate mental health provider.

The primary cells that become infected with HIV are the __________. a. B cells b. T cells c. IgM d. IgG

b. T cells

Cryoprecipitate is administered primarily for __________. a. a decreased hemoglobin and hematocrit b. hemophilia A c. an elevated white blood cell count d. sickle cell anemia

b. hemophilia A

The increased number of red blood cells associated with polycythemia vera causes the blood to increase in __________. a. plasma b. viscosity c. leukocytes d. volume

b. viscosity

A client receiving external radiation to the left thorax to treat lung cancer has a nursing diagnosis of Risk for impaired skin integrity. Which intervention should be part of this client's care plan? a) Removing thoracic skin markings after each radiation treatment b) Applying talcum powder to the irradiated areas daily after bathing c) Avoiding using deodorant soap on the irradiated areas d) Wearing a lead apron during direct contact with the client

c) Avoiding using deodorant soap on the irradiated areas

A patient has been transported to the operating room for emergent surgery. Which statement by the nurse best supports the need for emergent surgery? a)"The patient was tachycardic, had progressive weight loss, and bouts of insomnia as a result of hyperthyroidism." b)"The patient had epigastric abdominal pain, an elevated white blood count, and vomiting for 1 day." c)"The patient was unresponsive, had a distended abdomen, and unstable vital signs following a motor vehicle accident." d)"The patient had severe pain and a laceration to the face with minimal bleeding after being attacked by a dog 1 hour ago." Show Answer

c)"The patient was unresponsive, had a distended abdomen, and unstable vital signs following a motor vehicle accident." Explanation: Emergency surgery means that the patient requires immediate attention and the disorder may be life threatening. The patient with unstable vital signs and a distended abdomen following a motor vehicle accident requires immediate attention. The patient with left sided abdominal pain may not need surgery. Epigastric pain with vomiting for 1 day is usually not an indication for emergent surgery. Lacerations to the face require sutures, not emergent surgery. A thyroidectomy to treat hyperthyroidism is a required surgery, not an emergent one.

The nurse is triaging the surgical patients. Which patient would the nurse document as urgent for surgical care? a)A patient needing cataract surgery b)A patient scheduled for cosmetic surgery c)A patient with an acute gallbladder infection d)A patient with severe bleeding

c)A patient with an acute gallbladder infection Explanation: An acute gallbladder infection is considered an urgent surgical procedure. Cosmetic surgery and cataract surgery are not considered urgent surgical procedures. Severe bleeding could be considered an emergent surgical procedure.

During the preoperative assessment, the patient states he is allergic to avocados, bananas, and hydrocodone (Vicodin). What is the priority action by the nurse? a)Notify the nurse manager to follow up on the procedure. b)Notify the dietary department. c)Notify the surgical team to remove all latex-based items. d)Notify the physician regarding postoperative pain medications.

c)Notify the surgical team to remove all latex-based items. Explanation: Allergies to avocados and bananas may indicate an allergy to latex. Although it is necessary to notify the dietary department and physician, it is not an immediate threat, as the patient is NPO (nothing by mouth) and pain medication will be ordered postoperatively. The nurse manager does not need to be notified of the patient's allergies.

The nurse would identify which of the following vitamin deficiencies to prevent the complication of hemorrhaging during surgery? a)Vitamin A b)Zinc c)Vitamin K d)Magnesium

c)Vitamin K Explanation: Vitamin K is important for normal blood clotting. Vitamin A and zinc deficiencies would affect the immune system, whereas a magnesium deficiency would delay wound healing.

The life of a red blood cell (RBC) is approximately __________. a. 60 days b. 90 days c. 120 days d. 150 days

c. 120 days

Pernicious anemia is related to which vitamin deficiency? a. A b. B6 c. B12 d. C

c. B12

The nurse explains that the inflammatory response is initiated by immunoglobulin E (IgE) and the: a. Eosinophils b. Lymphocytes c. Basophils d. Neutrophils

c. Basophils

The nurse explains that the major difference between fresh frozen plasma (FFP) and cryoprecipitate (CPP) is that FFP: (Select all that apply.) a. Contains more albumin. b. Has a longer infusion time. c. Contains no platelets. d. Has a very high probability of causing an allergic reaction.

c. Contains no platelets

A blood transfusion reaction to donor leukocytes is what type of reaction? a. Hemolytic b. Anaphylactic c. Febrile d. Circulatory overload

c. Febrile

A newborn infant has developed significant jaundice and has a positive Coombs test result from high levels of bilirubin. The nurse has assessed the symptoms as being indicative of: a. Aplastic anemia b. Hemophilia c. Hemolytic anemia d. Sickle cell anemia

c. Hemolytic anemia

The nurse caring for a patient with crushing injuries from an automobile accident is horrified to find the patient bleeding profusely from the nose, mouth, and rectum, as well as from the injuries. The nurse assesses this emergency situation as: a. Hemophilia b. Disseminated intravascular coagulation (DIC) c. Leukemia d. Thrombocytopeni

c. Leukemia

When assessing the patient with thrombocytopenia, the nurse observes for: a. Distended neck veins and skin discoloration b. Discoloration of the nails and sclera c. Petechiae on the skin and bleeding gums d. Enlarged thyroid gland and excitability

c. Petechiae on the skin and bleeding gums

A 35 years old client has been receiving chemotherapy to treat cancer. Which assessment finding suggests that the client has developed stomatitis (inflammation of the mouth)? a. White, cottage cheese-like patches on the tongue b. Yellow tooth discoloration c. Red, open sores on the oral mucosa d. Rust-colored sputum

c. Red, open sores on the oral mucosa

The nurse is preparing to administer premedication. Which of the following actions should the nurse take first? a. have the family present b. ensure that the preoperative shave is complete c. have the patient void d. make sure the patient is covered with a warm blanket

c. have the patient void

In preparing discharge plans for a patient with SLE, the nurse must include: a. Need to consume 2 L of fluid daily b. Close monitoring of daily blood glucose level c. Use of daily sunscreens with a sun protection factor (SPF) higher than 15 d. Careful concern for certain food allergies

c. Use of daily sunscreens with a sun protection factor (SPF) higher than 15

Fresh frozen plasma (FFP) primarily contains __________. a. factors I and III b. factors II, IV, and IX c. all factors except platelets d. platelets

c. all factors except platelets

Colony-stimulating factors are administered to promote __________. a. stronger macrophages b. basophils resistant to inflammatory stress c. bone marrow production of white blood cells d. lymphatic production of B cells

c. bone marrow production of white blood cells

A nurse is performing a home visit for a client who received chemotherapy within the past 24 hours. The nurse observes a small child playing in the bathroom, where the toilet lid has been left up. Based on these observations, the nurse modifies the client's teaching plan to include:

chemotherapy exposure and risk factors.

The nurse is educating patients requiring surgery for various ailments on the perioperative experience. What education provided by the nurse is most appropriate? a)Expected pain levels and narcotic pain medication used to treat the pain b)Intraoperative techniques used to perform the surgery c)Risks and benefits of the surgical procedures d)Three phases of surgery and safety measures for each phase

d) Three phases of surgery and safety measures for each phase Explanation: The perioperative period includes the preoperative, intraoperative, and postoperative phases. Specific safety guidelines are followed for all surgical patients. The information provided should be general enough to be informative about surgery and should not focus on individual surgeries, as all the patients are having different surgeries. Intraoperative techniques, expected pain levels, and pain medication are specific to the patient and type of surgery. The risks and benefits of the surgical procedure should be discussed by the physician.

An obese patient is scheduled for open abdominal surgery. What priority education should the nurse provide this patient? a)Wound care and infection prevention b)Prevention of wound dehiscence c)Venous thromboembolism prevention d)Prevention of respiratory complications

d)Prevention of respiratory complications Explanation: All answers are correct but the obese patient has an increased susceptibility to respiratory complications, and maintaining a patent airway would be the priority.

A patient with a history of alcoholism and scheduled for an urgent surgery asks the nurse, "Why is everyone so concerned about how much I drink?" What is the best response by the nurse? a)"It is a required screening question for all patients having surgery." b)"We can have counselors available after surgery; if it is determined you need help for your drinking." c)"The amount of alcohol you drink will determine the amount of pain medication you will need postoperatively." d)"It is important for us to know how much and how often you drink to help prevent surgical complications."

d)"It is important for us to know how much and how often you drink to help prevent surgical complications." Explanation: Alcohol use and alcoholism can contribute to serious postoperative complications. If the medical and nursing staff is aware of the use or abuse, measures can be implemented proactively to prevent complications. Although alcohol may interfere with a medication's effectiveness, it does not determine the amount of pain medications that are prescribed following surgery. Even though this is a required screening question and counselors can be made available for those who want help, those are not the best responses to answer the patient's question.

What action by the nurse best encompasses the preoperative phase? a)Shaving the patient using a straight razor b)Monitoring vital signs every 15 minutes c)Documenting the application of sequential compression devices (SCD) d)Educating the patients on signs and symptoms of infection

d)Educating the patients on signs and symptoms of infection Explanation: Educating the patient on prevention or recognition of complications begins in the preoperative phase. Applying SCD and frequent vital sign monitoring happens after the preoperative phase. Only electric clippers should be used to remove hair.

A patient with fractured skull after falling from a ladder requires surgery. The nurse should anticipate transporting the patient to surgery during what time frame? a)In 1 week b)In 48-72 hours c)In 1 day d)Immediately

d)Immediately Explanation: Emergent surgery occurs when the patient requires immediate attention. A fractured skull is an indication for emergent surgery. An urgent surgery occurs when the patient requires prompt attention, usually within 24-30 hours. Any surgery scheduled beyond 30 hours is classified as required or elective and a fractured skull does not meet the requirements for elective or required surgery.

A patient asks about the purpose of withholding food and fluid before surgery. Which response by the nurse is appropriate? a)It decreases urine output so that a catheter would not be needed. b)It prevents overhydration and hypertension. c)It decreases the risk of elevated blood sugars and slow wound healing. d)It prevents aspiration and respiratory complications.

d)It prevents aspiration and respiratory complications. Explanation: The major purpose of withholding food and fluid before surgery is to prevent aspiration, which can lead to respiratory complications. Preventing overhydration, decreasing urine output, and decreasing blood sugar levels are not major purposes of withholding food and fluid before surgery.

The nurse is conducting a preoperative assessment on a patient scheduled for gallbladder surgery. The patient reports having a frequent cough producing green sputum for 3 days and denies fever. Upon auscultation, the nurse notes rhonchi throughout the right lung with an occasional expiratory wheeze. Respiratory rate is 20, temperature is 99.8 taken orally, heart rate is 87, and blood pressure is 124/70. What is the nurse's best action? a)Notify the primary physician about the assessment findings. b)Wait 1 hour and complete the assessment again. c)Document the findings and continue the patient through the preoperative phase. d)Notify the surgeon to possibly delay the surgery.

d)Notify the surgeon to possibly delay the surgery. Explanation: A respiratory infection can delay a nonemergent surgical procedure because the infection can increase the risk for respiratory complications. Therefore, the nurse should notify the surgeon about delaying the surgery. The primary physician may be called to care for the assessment findings but that should be done only after the surgeon has been notified. Continuing through the preoperative phase without notifying the surgeon and waiting 1 hour is not appropriate.

A patient is scheduled for elective surgery. To prevent the complication of hypotension and cardiovascular collapse, the nurse should report the use of what medication? a)Warfarin (Coumadin) b)Hydrochlorothiazide (HydroDIURIL) c)Erythromycin (Ery-Tab) d)Prednisone (Deltasone)

d)Prednisone (Deltasone) Explanation: Patients who have received corticosteroids are at risk of adrenal insufficiency. Insufficiency related to corticosteroids can cause circulatory collapse and hypotension. Hydrochlorothiazide and erythromycin can cause respiratory complications. Warfarin will increase the risk of bleeding.

A child with sickle cell anemia is placed prescribed the drug hydroxyurea. The nurse explains that this drug will: a. Increase energy b. Decrease cardiomegaly c. Clean out obstructed vessels d. Produce a hemoglobin that resists sickling

d. Produce a hemoglobin that resists sickling

The nurse is educating a patient scheduled for elective surgery. The patient currently takes aspirin daily. What education should the nurse provide in regard to the medication? a)Take half doses of the aspirin until 1 week after surgery. b)Aspirin should be increased until 3 days before surgery, and then it should be discontinued until 3 days after surgery. c)Continue to take the aspirin as ordered. d)Stop taking the aspirin 7 days before the surgery, unless otherwise directed by your physician.

d)Stop taking the aspirin 7 days before the surgery, unless otherwise directed by your physician. Explanation: Aspirin should be stopped at least 7 to 10 days before surgery. The other directions provided are incorrect.

An 11-year-old girl is diagnosed with idiopathic thrombocytopenic purpura (ITP). The parental statement that helps the nurse evaluate the teaching as successful is: a. "Our daughter can still be involved in gymnastics." b. "When our daughter's hemoglobin falls below 3.5, she'll need blood." c. "Our daughter will need genetic counseling before she marries." d. "Our daughter should avoid drugs containing sulfonamides."

d. "Our daughter should avoid drugs containing sulfonamides."

The nurse caring for a patient who is having radiation treatment for cancer is alert to the threat of thrombocytopenia ______ days after the start of radiation. a. 2 b. 5 c. 9 d. 12

d. 12

When could a "rebound" anaphylactic reaction occur after an initial attack even when epinephrine has been given? a. 1 hour b. 2 hours c. 3 hours d. 4 hours

d. 4 hours

A donor has AB- blood. Which patient or patients below can receive this type of blood safely? a. A patient with O- blood b. A patient with A- blood c. A patient with B- blood d. A patient with AB- blood

d. A patient with AB- blood

In preparing discharge plans for a patient recently diagnosed with pernicious anemia, the nurse must include information regarding: a. Adding daily high-fat, low-fiber supplements. b. Adding a rigorous daily workout. c. Avoiding prolonged exposure to direct sunlight. d. Providing sufficient rest periods throughout the day.

d. Providing sufficient rest periods throughout the day

A patient is receiving 1 unit of packed red blood cells. The unit of blood will be done at 1200. The patient is scheduled to have IV antibiotics at 1000. As the nurse you will: a. Stop the blood transfusion and administer the IV antibiotic, and when the antibiotic is done resume the blood transfusion. b. Administer the IV antibiotic via secondary tubing into the blood transfusion's y-tubing. c. Hold the antibiotic until the blood transfusion is done. d. Administer the IV antibiotic as scheduled in a second IV access site.

d. Administer the IV antibiotic as scheduled in a second IV access site. rationale: The nurse would NEVER administer the IV antibiotic in the same tubing as the blood product or stop the transfusion. Remember blood is time sensitive and must be transfused within 4 hours. Also, holding the antibiotic is not correct because antibiotics are time sensitive as well and must be administered at the scheduled time to maintain therapeutic blood levels.

The nurse explains that the type of bone marrow transplant that uses the patient's own bone marrow is: a. Allergenic b. Allogeneic c. Peripheral blood stem cell d. Autologous

d. Autologous

A 3-year-old African-American child is diagnosed with sickle cell anemia. The parents know that sickle cell anemia is hereditary but do not understand why their child has the disease, because neither of them has it. The nurse explains that: a. At least one of the parents has to have the disease. b. Only one parent has to have the disease or the trait. c. Someone in previous generations had the disease. d. Both parents were carriers of the sickle cell trait.

d. Both parents were carriers of the sickle cell trait

When the nurse prepares to give ferrous sulfate (Feosol) to a home health care patient, the nurse will: a. Mix the drug with a high-protein milkshake. b. Give undiluted with a small snack. c. Mix with coffee or cola to disguise the bitter taste. d. Dilute and offer through a straw and a few crackers.

d. Dilute and offer through a straw and a few crackers.

A nurse caring for a patient in the last stages of leukemia is aware that the patient is at risk from the bacteria of his own body as an infection risk source, such as: a. Beta-hemolytic streptococci b. Streptococcus pneumoniae c. Streptococcus viridans d. Pseudomonas aeruginosa

d. Pseudomonas aeruginosa

When the skin test shows a redness and swelling a few days after injection, the nurse assesses this as a hypersensitivity reaction of type: a. I b. II c. III d. IV

d. IV

A major focus in a teaching plan for a teenager with sickle cell anemia would be: a. Limit tobacco use to no more than two cigarettes a day. b. Eat foods high in iron and vitamin B. c. Maintain environmental temperature at 65° to 68° F d. Maintain adequate hydration.

d. Maintain adequate hydration

The nurse is alarmed by a complete blood count that shows a large shift to the left. The nurse assesses this to mean that the cell level count of: a. Neutrophils have dropped by 10%. b. Basophils have increased by 25%. c. Neutrophils have increased by 25%. d. Neutrophils have increased by 60%.

d. Neutrophils have increased by 60%.

A hospitalized patient has been prescribed dexamethasone (Decadron) for an allergic reaction. The teaching instruction that the patient should be given with discharge, relative to this drug, is: a. Report blurry vision. b. Take the medication on an empty stomach. c. Do not operate heavy machinery. d. Take this medication with meals.

d. Take this medication with meals

Leukemia is a(n) __________. a. low white blood cell count b. elevated neutrophil count c. lymph node malignancy d. cancer of the white blood cells

d. cancer of the white blood cells

The most numerous white blood cells are the __________. a. basophils b. eosinophils c. erythrocytes d. neutrophils

d. neutrophils

Hemorrhaging related to coagulation disorders is commonly due to abnormal clotting factors and ___________. a. leukopenia b. hemorrhagic fever c. anaphylaxis d. thrombocytopenia

d. thrombocytopenia

Plasma is composed primarily of __________. a. bile b. spinal fluid c. joint synovium d. water

d. water

During the first 24 hours after surgery, how often will the nurse evaluate the client's temperature?

every 4 hrs

The nurse is aware that loss of consciousness occurs with which type of anesthesia?

general

The nurse is working with a client who has had an allohematopoietic stem cell transplant (HSCT). The nurse notices a diffuse rash and diarrhea. The nurse contacts the physician to report that the client has symptoms of

graft-versus-host disease.

The nurse is creating the care plan for a 70-year-old obese client who has been admitted to the postsurgical unit following a colon resection. This client's age and increased body mass index mean that she is at increased risk for what complication in the postoperative period?

infection

As a circulating nurse, what task are you solely responsible for?

keeping records

A patient is to undergo surgery on his kidney. The patient would be placed in which position for the surgery?

on side, hairnet, band around knees

A client has been diagnosed with a neoplasm and is seeking further information and possible treatment. The primary care physician described the neoplasm as "insidious." What does the word "insidious" mean?

slow-growing


संबंधित स्टडी सेट्स

Test 1-ch. 9 & 10, generator videos & safety

View Set

Chapter 1: What is interaction design?

View Set

OpenStax US History Ch. 5-8 Review Questions & Terms

View Set

Module 4&5 ATI Questions (Respiratory, GI, and Cardiovascular Systems)

View Set

Intro to Environmental Science Chapter Ch 4, Sect 4-6

View Set

Strategic management: Q9 Assessing firm performance

View Set